You are on page 1of 81

TOPNOTCH MEDICAL BOARD PREP SURGERY SUPPLEMENT HANDOUT - Jules Lopez,MD-MBA,Teddy Carpio,MD-MBA

For inquiries visit www.topnotchboardprep.com.ph or email us at topnotchmedicalboardprep@gmail.com


Table 1. Cytokines And Their Responses to Injury
SURGERY REVIEW Cytokines and their Responses to Injury
Among earliest responders after
injury
This review material is a synthesis of the first few chapters of Tumor Necrosis Factor
Induces muscle breakdown and
Schwartz’s Principles of Surgery, 9th edition, Absite and Board Alpha (TNF- )
cachexia through increased
Review of Schwartz’s Principles of Surgery, 9th edition, 2009 Absite catabolism
Clinical Review of Surgery and RUSH integrated review of surgery. Induces fever through
Integration with other basic subjects as well as review questions prostaglandin activity in anterior
per section are also included to facilitate mastery of the course. Interleukin 1 (IL-1) hypothalamus
Promotes -endorphin release from
Ready? God bless and enjoy Surgery! pituitary
Promotes lymphocyte
- Dr. Jules Lopez and Dr. Teddy Carpio proliferation, immunoglobulin
production, gut barrier integrity
Interleukin 2 (IL-2)
P.S. Attenuated production after major
blood loss leads to
Ophthalmology, ENT, Orthopedics, and Gynecology topics are not immunocompromise
included in this surgery handout. Elicited by all immunogenic cells as
mediator of acute phase
Interleukin 6 (IL-6) response
Any statements, tables, figures marked with a means that the
Prolongs activated neutrophil
information highlighted was previously tested in previous board exams. survival
You should master/memorize those. Pay close attention to those in bold,
Chemoattractant for neutrophils,
italicized, underlines as these are very important facts to remember for Interleukin 8 (IL-8)
basophils, eosinophils, lymphocytes
the subject. Master topics written in our quick review, subject cross overs
and end of review question boxes because they provide high yield Activates macrophages via TH1
information, not just for surgery but for the rest of the other subjects as cells that demonstrate enhanced
well! Interferon (IFN- ) phagocytosis and microbial killing
Found in wounds 5-7 days after
injury
General Outline:
I. Basic Principles in Surgery
2. Heat Shock Proteins
a. Systemic Response to Injury and Metabolic support
Group of intracellular proteins that are increasingly
b. Fluid and Electrolyte management of the Surgical
expressed during times of stress
Patient
c. Hemostasis, Surgical bleeding and transfusion Bind both autologous and foreign proteins and thereby
function as intracellular chaperones for ligands such as
d. Surgical Infections and Shock
bacterial DNA and endotoxin
e. Trauma
f. Burns Protect cells from the deleterious effects of traumatic
g. Wound healing stress and, when released by damaged cells, alert the
II. Organ System Pathologies immune system of the tissue damage
a. Skin and soft tissues
b. Breast 3. Reactive Oxygen Species (ROS)
c. Head & Neck Small molecules that are highly reactive due to the
d. Esophagus presence of unpaired outer orbit electrons
e. Stomach Cause cellular injury to both host cells and invading
f. Small Intestine pathogens through the oxidation of unsaturated fatty
g. Appendix acids within cell membranes
h. Colon, Rectum & Anus
i. Abdominal Wall & Hernia 4. Eicosanoids (Refer to Figure 1 )
j. Liver, Portal Venous System & Gallbladder Derived primarily by oxidation of membrane
phospholipid,arachidonic acid
Composed of subgroups including prostaglandins,
PART I: BASIC PRINCIPLES IN SURGERY prostacyclins, hydroxyeicosatetraenoic acid,
thromboxanes, and leukotrienes
Generate a proinflammatory response
SYSTEMIC RESPONSE TO INJURY AND
METABOLIC SUPPORT Figure 1. Arachidonic acid metabolism . Cycloxygenase catalyzes the
formation of PG and TXA2from arachidonic acid. LT = leukotriene;
A. Mediators of Inflammation PG = prostaglandin; TXA2 = thromboxane A2
B. Central Nervous System Regulation of Inflammation
C. Hormonal Response to Injury
D. Surgical Metabolism
E. Nutrition in the Surgical Patient

A. MEDIATORS OF INFLAMMATION
1. Cytokines(Refer to Table 1 )
Protein signaling compounds that are essential for
both innate and adaptive immunity
Mediate cellular responses, including cell migration and
turnover, DNA replication, and immunocyte
proliferation

TOPNOTCH MEDICAL BOARD PREP SURGERY SUPPLEMENT HANDOUT Page 1 of 81


For inquiries visit www.topnotchboardprep.com.ph or email us at topnotchmedicalboardprep@gmail.com
TOPNOTCH MEDICAL BOARD PREP SURGERY SUPPLEMENT HANDOUT - Jules Lopez,MD-MBA,Teddy Carpio,MD-MBA
For inquiries visit www.topnotchboardprep.com.ph or email us at topnotchmedicalboardprep@gmail.com

PHARMACOLOGY a Wound healing is impaired because it reduces


transforming growth factor-beta (TGF-B) and insulin-
Aspirin (Acetylsalicylic acid, ASA) is one of the non- like growth factor I (IGF-I) in the wound
steroidal anti-inflammatory drug (NSAID) prototypes
Figure 2. Steroid synthesis from cholesterol . ACTH is a principal
Mechanism of action ||
regulator of steroid synthesis. The end products are mineralocorticoids,
o Nonselective AND irreversible inhibitor of glucocorticoids, and sex steroids.
the enzyme cyclooxygenase (COX), inhibiting
both cylooxygenase-1 (COX-1) and
cyclooxygenase-2 (COX-2) isoenzymes
o Anti-inflammatory effect is mediated by COX-2
inhibition via decreasing platelet production of
TXA2, a potent stimulator of platelet
aggregation
Side effect ||Gastrointestinal (GI) toxicity due to
inhibition of COX-1 and therebyPG synthesis
Notes ||Uncoupler of oxidative phosphorylation and
is associated with Reye syndrome in children
Ketorolac and Indomethacin are bothnonselective
BUT reversible COX inhibitors
Celecoxib is a selective COX-2 inhibitor with a reduced
risk of GI toxicity

BIOCHEMISTRY a
5. Fatty Acid Metabolites
Omega-3 fatty acids have anti-inflammatory effects, Cholesterol is a steroid alcohol
including inhibition of TNF release from hepatic It is a precursor of the following:
Kupffer cells, leukocyte adhesion and migration 1. Cell membranes
6. Kallikrein-Kinin System 2. Vitamin D (7-dehydrocholesterol)
Group of proteins that contribute to inflammation, 3. Bile salts (cholic and chenodeoxycholic acid)
BP control, coagulation, and pain responses 4. Adrenal hormones (aldosterone and cortisol)
Kallikrein levels are increased during gram negative 5. Sex hormones (testosterone and estradiol)
bacteremia, hypotension, hemorrhage, endotoxemia, It is very hydrophobic (which means, it can cross lipid
and tissue injury predominant barriers), composed of 4 fused
Kinis mediate vasodilation, increased capillary hydrocarbon rings (A-D) and 8-membered branched
permeability, tissue edema, pain pathway activation, hydrocarbon chain (20-27) attached to the D-ring
inhibition of gluconeogenesis, and increased It has a single hydroxyl group located at carbon 3 of the
bronchoconstriction A-ring to which a fatty acid can be attached to form
Elevated levels of both has been associated with the cholesterol esters
magnitude of injury and mortality

7. Serotonin
Released at the site of injury, primarily by platelets
Stimulates vasoconstriction, bronchoconstriction, and
platelet aggregation
Ex vivo study showed that serotonin receptor blockade
is associated with decreased production of TNF andIL-1
in endotoxin-treated monocytes

8. Histamine (H4)
Associated with eosinophil and mast cell chemotaxis
Increased release has been documented in hemorrhagic
shock, trauma, thermal injury, endotoxemia, and sepsis
2. Macrophage Migration-Inhibiting Factor
B. CNS REGULATION OF INFLAMMATION Neurohormone that is stored and secreted by the
Vagus nerve is highly influential in mediating afferent anterior pituitary and by intracellular pools within
sensory input to the CNS macrophages
o Parasympathetic nervous system transmits A counter regulatory mediator that potentially reverses
its efferent signals via acetylcholine the anti-inflammatory effects of cortisol
o Exerts homeostatic influences such as
enhancing gut motility, reducing heart rate, 3. Growth Hormones (GH)
and regulating inflammation Neurohormone expressed primarily by the pituitary
o Allows for a rapid response to inflammatory gland that has both metabolic and immunomodulatory
stimuli and also for the potential regulation of effects
early proinflammatory mediator release, Exerts its downstream effects through direct interaction
specifically tumor necrosis factor (TNF) with GH receptors and secondarily through the
o Inhibit cytokine activity and reduce injury enhanced hepatic synthesis of IGF-I
from disease process GH and IGF-I promote protein synthesis and insulin
resistance, and enhances mobilization of fat stores
C. HORMONAL RESPONSE TO INJURY
1. Cortisol(Refer to Figure2 ) 4. Catecholamines
Glucocorticoid steroid hormone released by the adrenal Include epinephrine, norepinephrine, and dopamine,
cortex in response to adrenocorticotropic hormone which have metabolic, immunomodulatory, and
(ACTH) vasoactive effects
Release is increased during times of stress and may be After severe injury, plasma catecholamine levels are
chronically elevated in certain disease processes (e.g. increased threefold to fourfold, with elevations lasting
burn-injured patients may exhibit elevated levels for 24 to 48 hours before returning to baseline levels
4 weeks)

TOPNOTCH MEDICAL BOARD PREP SURGERY SUPPLEMENT HANDOUT Page 2 of 81


For inquiries visit www.topnotchboardprep.com.ph or email us at topnotchmedicalboardprep@gmail.com
TOPNOTCH MEDICAL BOARD PREP SURGERY SUPPLEMENT HANDOUT - Jules Lopez,MD-MBA,Teddy Carpio,MD-MBA
For inquiries visit www.topnotchboardprep.com.ph or email us at topnotchmedicalboardprep@gmail.com
5. Insulin
Mediates an overall host anabolic state Lactate production is insufficient to maintain systemic
Insulin resistance and hyperglycemia are hallmarks glucose needs during short-term fasting; therefore,
of critical illness due to the catabolic effects of significant amounts of protein must be degraded
circulating mediators, including catecholamines, cortisol, daily (75 g/d for a 70 kg adult) to provide the amino
glucagon, and GH acid substrate for hepatic gluconeogenesis
Hyperglycemia during critical illness has Proteolysis during starvation, which results from
immunosuppressive effects, and thus is associated decreased insulin and increased cortisol release, is
with an increased risk for infection associated with elevated urinary nitrogen excretion
Insulin therapy (to manage hyperglycemia) decreased from the normal 7-10 g/day up to 30 g or more/day
mortality and reduced in infectious complications in
select patient populations 2. Metabolism During Prolonged Fasting
Systemic proteolysis is reduced approximately 20 g/d
QUICK REVIEW a and urinary nitrogen excretion stabilizes at 2 to 5 g/d
due to adaptation by vital organs (e.g. myocardium,
Burn patients may exhibit elevated levels of cortisol for brain, renal cortex, and skeletal muscle) to using
4 weeks ketone bodies as their principal fuel source
Plasma catecholamine levels are increased 3-4x lasting Ketone bodies become an important fuel source for the
for 24 to 48 hours before returning to baseline brain after 2 days and gradually become the principal
fuel source by 24 days

D. SURGICAL METABOLISM 3. Metabolism After Injury


To maintain basal metabolic needs (i.e. at rest and Injuries or infections induce unique neuroendocrine
fasting), a normal healthy adult requires ~22 to 25 and immunologic responses that differentiate injury
kcal/kg/dayfrom carbohydrate, lipid, and protein metabolism from that of unstressed fasting
sources Magnitude of metabolic expenditure appears to be
Initial hours after surgical or traumatic injury are directly proportional to the severity of insult, with
metabolically associated with a reduced total body thermal injuries and severe infections having the
energy expenditure and urinary nitrogen wasting highest energy demands (Refer to Figure 4)

Figure 4. Influence of injury severity on resting metabolism


1. Metabolism During Short-term Fasting (<5 days)
(resting energy expenditure or REE)
In the healthy adult, principal sources of fuel are
derived from muscle protein and lipids, with lipids
being the most abundant source of energy(40% or
more of caloric expenditure)
Hepatic glycogen stores are rapidly and preferentially
depleted fall of serum glucose concentration within
hours (<16 hours)
Hepatic gluconeogenesis is then activated using lactate
from skeletal muscle as the main precursor(Refer to
Figure 3 )

Figure 3. The recycling of peripheral lactate and pyruvate for hepatic


gluconeogenesis is accomplished by the Cori cycle. Alanine within skeletal
muscles can also be used as a precursor for hepatic gluconeogenesis .

Lipids(Triglyceride) become the primary source of


energy (50-80%) during critical illness and stressed
states
o Minimize protein catabolism in the injured
patient
o Lipolysis occurs mainly in response to
catecholamine stimulus of the hormone-
BIOCHEMISTRY a sensitive triglyceride lipase
Injury and severe infections induce a state of
Lactate is generated from the skeletal muscle during peripheral glucose intolerance (insulin resistance),
anaerobic metabolism despite ample insulin production at levels several-fold
Muscle CANNOT reconvert lactate to glucose above baseline
Lactate MUST first be transported to the liver, where it is o Occur in part due to reduced skeletal muscle
converted to glucose via hepatic gluconeogenesis pyruvate dehydrogenase activity after injury,
Glucose is then brought back to musclesfor usage which diminishes the conversion of pyruvate
This metabolic pathway involving conversion of lactate to acetyl-CoA and subsequent entry into the
to glucose; which, in turn, is brought back to the muscle TCA cycle
for utilization is the Cori cycle o Increase in plasma glucose levels is
Energy expense: 4 ATP molecules proportional to the severity of injury, and this
net hepatic gluconeogenic response is under
the influence of glucagon
After injury, the initial systemic proteolysis, mediated
primarily by cortisol, increases urinary nitrogen
excretion to levels in excess of 30 g/d, which roughly

TOPNOTCH MEDICAL BOARD PREP SURGERY SUPPLEMENT HANDOUT Page 3 of 81


For inquiries visit www.topnotchboardprep.com.ph or email us at topnotchmedicalboardprep@gmail.com
TOPNOTCH MEDICAL BOARD PREP SURGERY SUPPLEMENT HANDOUT - Jules Lopez,MD-MBA,Teddy Carpio,MD-MBA
For inquiries visit www.topnotchboardprep.com.ph or email us at topnotchmedicalboardprep@gmail.com
corresponds to a loss in lean body mass of 1.5%/d consumption, increased CO2 production and
(Refer to Figure 5) prolonged need for ventilatory support,
suppression of leukocyte function,
Figure 5. Effect of injury severity on nitrogen wasting hyperglycemia, and increased risk of infection

1. Enteral Nutrition
Generally preferred over parenteral nutrition due to:
o Lower cost
o Associated risks of the intravenous route
o Beneficial effects of luminal nutrient contact as
it reduces intestinal mucosal atrophy
Initiation should occur immediately after adequate
resuscitation (adequate urine output)
Presence of bowel sounds and the passage of flatus or
stool are NOT absolute prerequisites to start enteral
nutrition, EXCEPT in the setting of gastroparesis,
feedings should be administered distal to the pylorus
Gastric residuals of 200 ml or more in a 4 to 6 hour
period or abdominal distention requires cessation of
feeding and adjustment of infusion rate
The following are options for enteral feeding access
QUICK REVIEW a (Refer to Table 3 ):

Normal energy requirement: 22 to 25 kcal/kg/day Table 3. Options for Enteral Feeding Access
Initial hours after surgical or traumatic injury results to a Options for Enteral Feeding Access
reduced total body energy expenditure and urinary Short-term use
nitrogen wasting Nasogastric tube Aspiration risks
Fat/lipid is the primary source of calories during acute (NGT) Nasopharyngeal trauma
Frequent dislodgement
starvation (<5 days fasting) and after acute injury
Short-term use
Ketone bodies is the primary fuel source in prolonged
Nasoduodenal / Lower aspiration risks in jejunum
starvation Nasojejunal tube Placement challenges (radiographic
Ketone bodies becomes an important fuel source for assistance often necessary)
brain after 2 days and eventually become the principal Endoscopy skills required
fuel source by 24 days May be used for gastric decompression or
Percutaneous
bolus feeds
Endoscopic
Aspiration risks
Gastrostomy
E. NUTRITION IN THE SURGICAL PATIENT Can last 12-24 months
(PEG)
Goals of nutritional support in the surgical patient are Slightly higher complication rates with
as follows: placement and site leaks
Requires general anesthesia and small
o To meet the energy requirements for
laporotomy
metabolic processes, core temperature Surgical
Procedure may allow placement of
maintenance, and tissue repair Gastrostomy
extended duodenal/jejunal feeding ports
o To meet the substrate requirements for Laparoscopic placement possible
protein synthesis Commonly carried out during laparotomy
Energy requirement may be measured by indirect General anesthesia, laparoscopic
calorimetry and trends in serum markers (e.g. Surgical placement usually requires assistant to
prealbumin level) and estimation from urinary nitrogen Jejunostomy thread catheter
excretion, which is proportional to resting energy Laparoscopy offers direct visualization of
expenditure catheter placement
Basal energy expenditure (BEE) may also be Jejunal placement with regular endoscope
is operator dependent
estimated using Harris-Benedict equations, adjusted for
Jejunal tube often dislodges retrograde
the type of surgical stress (Refer to Table 2) PEG-jejunal tube Two-stage procedure with PEG
o BEE (men) = 66.47 + 13.75 (weight in kg) + 5 placement, followed by fluoroscopic
(height in cm) – 6.76 (age in years) kcal/d conversion with jejunal feeding tube
o BEE (women) = 655.1 + 9.56 (weight in kg) + through PEG
1.85 (height in cm) – 4.68 (age in years) kcal/d
The BEE is then multiplied by the type of surgical stress 2. Parenteral Nutrition
(Refer to Table 2 ) that the patient has to determine Continuous infusion of hyperosmolar solution
the total daily caloric need containing carbohydrates, proteins, fat, and other
necessary nutrients through an indwelling catheter
Table 2. Caloric Adjustment Above BEE in Hypermetabolic Conditions inserted into the superior vena cava
Caloric Adjustments Above Basal Energy Expenditures in Principal indications include malnutrition, sepsis, or
Hypermetabolic Conditions
surgical or traumatic injury in seriously ill patients for
Normal or Moderate Malnutrition 25-30 kcal/kg/day
whom use of the gastrointestinal tract for feedings is
Mild Stress 25-30
not possible
Moderate Stress 30
Severe Stress 30-35 Total (Central) Parenteral Nutrition (TPN)requires
Burns 35-40 access to a large-diameter vein to deliver the nutritional
requirements of the individual
Provision of 30 kcal/kg/d will adequately meet energy o Dextrose content of the solution is high (15-
requirements in most postsurgical patients, with low 25%)
risk of overfeeding o All other macronutrients and micronutrients
o Overfeedingusually results from are deliverable by this route
overestimation of caloric needs because actual Peripheral Parenteral Nutrition (PPN) uses lower
body weight is used to calculate BEE, osmolarity of the solution to allow its administration via
especially in special patients (e.g. critically ill peripheral veins
with significant fluid overload and the obese) o Reduced levels of dextrose (5-10%) and
o Overfeeding may contribute to clinical protein (3%)
deterioration via the following: increased O2
TOPNOTCH MEDICAL BOARD PREP SURGERY SUPPLEMENT HANDOUT Page 4 of 81
For inquiries visit www.topnotchboardprep.com.ph or email us at topnotchmedicalboardprep@gmail.com
TOPNOTCH MEDICAL BOARD PREP SURGERY SUPPLEMENT HANDOUT - Jules Lopez,MD-MBA,Teddy Carpio,MD-MBA
For inquiries visit www.topnotchboardprep.com.ph or email us at topnotchmedicalboardprep@gmail.com
o Some nutrients cannot be supplemented c. 75%
because they cannot be concentrated into d. 100%
small volumes
o Not appropriate for repleting patients with Answer: B
severe malnutrition
o Used for short periods (<2 weeks); beyond Sepsis increases metabolic needs to approximately
this, TPN should be instituted 150-160% of resting energy expenditure, or 50%
Complications are as follows (Refer to Table 4 ): above normal (Refer to Figure 4). This is mediated in
part by sympathetic activation and catecholamine
Table 4. Complication of Parenteral Nutrition release.
Complications of Parenteral Nutrition
Rare occurrences if IV vitamin 3. Which of the following is the initial enteric formula for
preparations are used the majority of surgical patients?
However, Vitamin K is not part of any a. Low-residue isotonic formula
Vitamin Deficiencies
commercially prepared vitamin
b. Elemental formula
solution so it should be supplemented
on a weekly basis c. Calorie dense formula
Clinically apparent during prolonged d. High protein formula
parenteral nutrition with fat-free
solutions Answer: A
Essential Fatty Acid Manifests as dry, scaly dermatitis and
(EFA) Deficiency loss of hair Most low-residue isotonic formulas provide a caloric
Prevented by periodic infusion of a fat density of 1.0 kcal/ml, and approximately 1500 to
emulsion at a rate equivalent to 10 to 1800 ml are required to meet daily requirements.
15% of total calories
These provide baseline carbohydrates, protein,
Essential trace minerals may be
electrolytes, water, fat, and fat-soluble vitamins. These
required after prolonged TPN
Zinc deficiency is the most common
solutions usually are considered to be the standard or
that manifests as diffuse eczematoid first-line formulas for stable patients with an intact GI
Trace Mineral tract.
rash at intertriginous areas
Deficiencies
Copper deficiency is associated with
Microcytic anemia
Chromium deficiency is associated with FLUID AND ELECTROLYTE MANAGEMENT
Glucose intolerance
OF THE SURGICAL PATIENT
May occur after initiation of parenteral
nutrition
Manifests as glycosuria A. Body Fluids and Compartments
If blood glucose levels remain elevated B. Body Fluid Changes
or glycosuria persists, dextrose C. Fluid Therapy
Relative Glucose
concentration may be decreased, D. Special Case: Refeeding Syndrome
infusion rate slowed, or regular insulin E. Electrolyte Abnormalities
Intolerance
added to each bottle F. Acid-Base Disorders
Rise in blood glucose may be
temporary, as the normal pancreas
increases its output of insulin in
response to the continuous A. BODY FLUIDS AND COMPARTMENTS
carbohydrate infusion Water constitutes ~50-60% of total body weight
Due to large glucose infusion, a Relationship between total body weight and total body
significant shift of potassium from water (TBW) is relatively constant for an individual
Hypokalemia
extracellular to intracellular space may and is primarily a reflection of body fat
(and Metabolic
take place
Acidosis) o Lean tissues (e.g. muscle and solid organs)
Manifests as glycosuria, which is
treated with potassium, NOT insulin
have higher water content than fat and bone
Lack of intestinal stimulation is o TBW of average young adult male and
associated with intestinal mucosal female is 60% and 50%, respectively of
atrophy, diminished villous height, total body weight
Intestinal Atrophy bacterial overgrowth, reduced Estimates of %TBW should be adjusted downward
lymphoid tissue size, reduced ~10-20% for obese individuals and upward by ~10%
immunoglobulin A production, and for malnourished individuals
impaired gut immunity
Highest percentage of TBW is found in newborns
(~80%)
REVIEW QUESTIONS a
QUICK REVIEW a
1. Prostacyclin has which of the following effects in
systemic inflammation?
a. Inhibition of platelet aggregation TBW is ~50-60% of total body weight
b. Vasoconstriction TBW (Male): 60%of total body weight
c. Increased adhesion molecules TBW (Female): 50%of total body weight
d. Decreased cardiac output Young lean males have a higher proportion of TBW than
elderly or obese individuals
Answer: A Lower percentage of TBW in females generally
correlates with a higher percentage of adipose tissue and
Prostacyclin is a member of the eicosanoid family and lower percentage of muscle mass
is primarily produced by endothelial cells. It is an
effective vasodilator and also inhibits platelet
aggregation. During systemic inflammation, TBW is divided into 3 functional fluid compartments
prostacyclin expression is impaired and thus the (Refer to Table 5 ):
endothelium favors a more procoagulant profile. o Plasma (extracellular)
o Interstitial fluid (extracellular)
2. Sepsis increases metabolic needs by approximately o Intracellular fluid
what percentage?
a. 25%
b. 50%
TOPNOTCH MEDICAL BOARD PREP SURGERY SUPPLEMENT HANDOUT Page 5 of 81
For inquiries visit www.topnotchboardprep.com.ph or email us at topnotchmedicalboardprep@gmail.com
TOPNOTCH MEDICAL BOARD PREP SURGERY SUPPLEMENT HANDOUT - Jules Lopez,MD-MBA,Teddy Carpio,MD-MBA
For inquiries visit www.topnotchboardprep.com.ph or email us at topnotchmedicalboardprep@gmail.com
Table 5. Functional Body Fluid Compartments Table 6. Normal Fluid Balance
Extracellular fluid PLASMA (1) Water Gain Water Loss
Total (1/3 of TBW or (5% of total body weight) Urine
Body 20% of total body INTERSTITIAL FLUID (2) Oral fluids 800-1200 ml
Water weight) (15% of total body weight) 1,500 ml Stool
(TBW) INTRACELLULAR FLUID (3) Sensible
250 ml
(2/3 of TBW or 40% of total body weight) Solid fluids Sweat
500 ml 0 ml
Water of oxidation Skin
Extracellular fluid compartment (ECF) is balanced Insensible
250 ml 450 ml
between sodium (Na2+), the principal cation, and Water of solution Lungs
chloride (Cl-) and bicarbonate (HCO3-), the principal 0 ml 150 ml
anions(Refer to Figure 6)
o Composition of the plasma and interstitial fluid
differs only slightly in ionic composition 1. Extracellular Volume Deficit
o Slightly higher protein content (anions) in Most common fluid disorder in surgical patients
plasma results in a higher plasma cation Can either be acute or chronic (Refer to Table 7)
composition relative to the interstitial fluid o Acute volume deficit is associated with
Intracellular fluid compartment (ICF) is comprised cardiovascular and central nervous system
ofcations, potassium (K+) and magnesium (Mg2+), and signs
theanions, phosphate(HPO4-) and proteins o Chronic deficit displays tissue signs such as
Concentration gradient between compartments is decrease in skin turgor and sunken eyes, in
maintained by adenosine triphosphate (ATP) driven addition to acute signs
sodium-potassium pumps located with the cell
Table 7. Signs and Symptoms of Volume Disturbances
membranes
System Volume Deficit Volume Excess
Weight loss Weight gain
Figure 6. Chemical composition of body fluid compartments General
Decreased skin turgor Peripheral edema
Tachycardia Increased cardiac output
Orthostasis / Increased central venous
Cardio Hypotension pressure
Collapsed neck veins Distended neck veins
Murmur
Oliguria --
Renal
Azotemia
GI Ileus Bowel edema
Pulmo -- Pulmonary edema

Most common cause of volume deficit in surgical


patients is a loss of GI fluids from nasogastric suction,
vomiting, diarrhea, or enterocutanous fistula
(Refer to Table 8 )
Third-space or nonfunctional ECF losses that occur with
sequestration secondary to soft tissue
injuries/infections, burns, and intraabdominal
processes such as peritonitis, obstruction, or prolonged
surgery can also lead to massive volume deficits

Table 8. Composition of GI Secretions


Volume Volume Na+ K+ Cl- HCO3
(ml/24h) (ml/24h) (mEq/L) (mEq/L) (mEq/L) (mEq/L)
Saliva 1000 10 26 10 30
Water is freely diffusible and distributed evenly Stomach 1000-2000 60-90 10 130 0
throughout all fluid compartments of the body Duodenum 1500 120-140 5-10 90-120 0
Sodiumis confined to ECF and is associated with water Ileum 3000 140 5 104 30
Colon 750 60 30 40 0
o Sodium-containing fluids are distributed
Pancreas 600-800 135-145 5-10 70-90 115
throughout the ECF and add to bothplasma
Bile 300-800 135-145 5-10 90-110 30-40
(intravascular) and interstitial spaces
o Sodium-containing fluids expand the
2. Extracellular Volume Excess
interstitial space by ~3x as much as the plasma
May be iatrogenic or secondary to renal dysfunction,
congestive heart failure, or cirrhosis
B. BODY FLUID CHANGES
Both plasma and interstitial volumes are increased
A healthy person consumes water an average of 2L/d,
~75% from oral intake and the rest extracted from solid Symptoms are primarily pulmonary and cardiovascular
foods (Refer to Table 6) (Refer to Table 7)
Daily water losses include 800-1200 ml in urine, 250 ml In healthy patients, edema and hyperdynamic
in stool, and 600 ml in insensible losses through both circulation are common and well tolerated
the skin (75%) and lungs (25%) However, the elderly and patients with cardiac disease
may quickly develop congestive heart failure and
Sensible water losses such as sweating or pathologic
loss of GI fluids vary widely, but these include loss of pulmonary edema in response to only a moderate
electrolytes as well volume excess
o Sweat is hypotonic and sweating usually
results in only a small sodium loss C. FLUID THERAPY
o Pathologic GI losses are isotonic to slightly Most commonly used solutions are as follows:
hypotonic and contribute little to net gain or (Refer to Table 9)
loss of free water

TOPNOTCH MEDICAL BOARD PREP SURGERY SUPPLEMENT HANDOUT Page 6 of 81


For inquiries visit www.topnotchboardprep.com.ph or email us at topnotchmedicalboardprep@gmail.com
TOPNOTCH MEDICAL BOARD PREP SURGERY SUPPLEMENT HANDOUT - Jules Lopez,MD-MBA,Teddy Carpio,MD-MBA
For inquiries visit www.topnotchboardprep.com.ph or email us at topnotchmedicalboardprep@gmail.com
Table 9. Electrolyte Solutions for Parenteral Administration o Patients whose volume deficit is not corrected
Solution Na+ Cl- K+ Ca2+ Other mOsm after initial volume challenge and those with
ECF 142 103 4 27 -- 280 impaired renal function and the elderly should
Lactated Lactate be considered for more intensive monitoring
130 109 4 28 280
Ringer’s (LR) 28 mEq/l of central venous pressure or cardiac output in
0.9% Sodium
an ICU setting
chloride 154 154 0 0 -- 308
(PNSS) o If symptomatic electrolyte abnormalities
Dextrose accompany volume deficit, the abnormality
D5 Lactated should be corrected to the point that the acute
50 g/l
Ringer’s 130 109 4 3 560
Lactate symptom is relieved before surgical
(D5LR)
28 mEq/l intervention.
D5 Sodium
Dextrose
chloride 154 154 0 0 588
50 g/l QUICK REVIEW a
(D5NS)
D5 0.45%
Dextrose Extracellular volume deficit is the most common fluid
Sodium 77 77 0 0 434
50 g/l disorder in surgical patients
chloride
D5 0.25% Most common cause of volume deficit in surgical
Dextrose
Sodium 34 34 0 0 357 patients is a loss of GI fluids
50 g/l
chloride Both PLR and PNSS are considered isotonic and are
useful in replacing GI losses and correcting extracellular
volume deficits
Type of fluid administered depends on the patient’s Hypertonic saline solution is used as a treatment
volume status and the type of concentration or modality in patients with closed head injuries
composition abnormality present (Refer to Table 10)

Table 10. Fluid Therapy


2. Intraoperative Fluid Therapy
Solution Description
Considered isotonic BUT it is slightly
With the induction of anesthesia, compensatory
Lactated hypotonic due to lactate mechanisms are lost, and hypotension will develop if
Ringer’s (PLR) Useful in replacing GI losses and correcting volume deficits are not appropriately managed
extracellular volume deficits To avoid hemodynamic instability intraoperatively, the
Considered isotonic BUT it is mildly following should be ensured:
0.9% Sodium
hypertonic o Known fluid losses corrected
Also useful in replacing GI losses and preoperatively
chloride
correcting extracellular volume deficits, o Adequate maintenance fluid therapy
(PNSS)
especially those associated with hyponatremia, provided
hypochloremia, and metabolic alkalosis
o Ongoing losses replaced intraoperatively
Useful for replacement of ongoing GI losses as
well as for maintenance fluid therapy in the Among the ongoing losses during surgery include
D5 0.45% distributional shifts via third space or nonfunctional ECF
postoperative period
Sodium losses seen in the following:
Provides sufficient free water for insensible
chloride
losses and enough sodium to aid the kidneys in o Major open abdominal surgeries in the form of
adjustment of serum sodium levels bowel wall edema, peritoneal fluid, and the
D5 3.5-5% wound edema during surgery
Hypertonic saline solution
Sodium o Large soft tissue wounds, complex fractures
Used for correction of severe sodium deficits
chloride with associated soft tissue injury, and burns
Hypertonic saline solution
Replacement of ECF losses during surgery often
Used as a treatment modality in patients with
closed head injuries
requires 500 to 1000 ml/hr of a balanced salt solution
D5 7% Shown to increase cerebral perfusion and to support homeostasis
Sodium decrease intracranial pressure, thus decreasing Addition of albumin or other colloid-containing
chloride brain edema solutions to intraoperative fluid therapy is NOT
However, there also have been concerns of necessary
increased bleeding, because hypertonic saline is
an arteriolar vasodilator 3. Postoperative Fluid Therapy
Should be based on the patient’s current estimated
volume status and projected ongoing fluid losses
1. Preoperative Fluid Therapy Any deficits from either preoperative or intraoperative
Preoperative evaluation of a patient’s volume status and losses should be corrected and ongoing requirements
pre-existing electrolyte abnormalities is an important should be included along with maintenance fluids
part of overall preoperative care In the initial postoperative period, an isotonic solution
Administration of maintenance fluidsis required in an should be administered
otherwise healthy individual on NPO before surgery o Adequacy of resuscitation should be based on
The following is the formula used for calculating vital signs and urine output
maintenance fluids in the absence of pre-existing o All measured losses, including losses through
abnormalities (Refer to Table 11): vomiting, NGT, drains, and urine output as
well as insensible losses should be replaced
Table 11. Maintenance Fluid Computation
After the initial 24 to 48 hours, fluids can be changed to
First 0-10 kg Give 100 ml/kg/dor 4 ml/kg/hr 5% dextrose to 0.45% saline in patients unable to
Next 10-20 kg Give additional 50 ml/kg/dor 2 ml/kg/hr tolerate enteral nutrition
Weight >20 kg Give additional 20 ml/kg/dor 1 ml/kg/hr If normal renal function and adequate urine output are
present, potassium may be added to the IV fluids
However, may surgical patients have volume and/or
electrolyte abnormalities associated with their surgical D. SPECIAL CASE: REFEEDING SYNDROME
disease Refeeding syndrome potentially lethal condition that
o Acute volume deficits should be corrected as can occur with rapid and excessive feeding of
much as possible patients with severe underlying malnutrition due to
o Once a volume deficit is diagnosed, prompt starvation, alcoholism, delayed nutritional support,
fluid replacement should be instituted, usually anorexia nervosa, or massive weight loss in obese
with an isotonic crystalloid patients

TOPNOTCH MEDICAL BOARD PREP SURGERY SUPPLEMENT HANDOUT Page 7 of 81


For inquiries visit www.topnotchboardprep.com.ph or email us at topnotchmedicalboardprep@gmail.com
TOPNOTCH MEDICAL BOARD PREP SURGERY SUPPLEMENT HANDOUT - Jules Lopez,MD-MBA,Teddy Carpio,MD-MBA
For inquiries visit www.topnotchboardprep.com.ph or email us at topnotchmedicalboardprep@gmail.com
Shift in metabolism from fat to carbohydrate substrate Figure 8. Etiology of Hyponatremia
stimulates insulin release, which results in the cellular
uptake of electrolytes, particularly phosphate,
magnesium, potassium, and calcium
Severe hyperglycemia may result from blunted basal
insulin secretion
To prevent its development, the following measures
should be done:
o Underlying electrolyte and volume deficits
should be corrected
o Thiamine should be administered before the
initiation of feeding
o Caloric repletion should be instituted slowly,
at 20 kcal/kg per day, and should gradually
increase over the first week

E. ELECTROLYTE ABNORMALITIES Treatment ||Water restriction and, if severe, the


1. Hypernatremia administration of sodium
Results from either a loss of free water or a gain of If symptomatic, 3% normal saline should be used to
sodium in excess of water increase the sodium by no more than 1 mEq/l/huntil
Associated with either an increased, normal, or the serum sodium reaches 130 mEq/l or symptoms are
decreased extracellular volume (Refer to Figure 7) improved
Symptoms are rare until serum sodium concentration If asymptomatic, correction should increase the sodium
exceeds 160 mEq/l level by no more than 0.5 mEq/l/hr to a maximum
Clinical manifestations || Mostly central nervous increase of 12 mEq/l/d
system in nature (restlessness, irritability, seizures,
coma) due tohyperosmolarity INTERNAL MEDICINE a
May lead to subarachnoid hemorrhage and death
Central Pontine Myelinosisis a consequence of rapid
Figure 7. Etiology of Hypernatremia correction of hyponatremia
Characterized with seizures, weakness, paresis, akinetic
movements, and unresponsiveness
May result in permanent brain damage and death
MRI may assist in the diagnosis

3. Hyperkalemia
Serum K+ concentration above the normal range of
3.5-5 mEq/l
Caused by excessive K+ intake, increased release of K+
from cells, or impaired K+ excretion by the kidneys
(Refer to Table 12)
Clinical manifestations || Mostly GI (nausea/vomiting,
diarrhea), neuromuscular (weakness, paralysis), and
cardiovascular (arrhythmia, arrest)
ECG changes ||High peaked T waves (early),
widened QRS complex, flattened P wave, prolonged PR
interval (first-degree block), sine wave formation and
Treatment ||Management of water deficit ventricular fibrillation
In hypovolemic patients, volume should be restored Treatment ||Reducing total body K+, shifting K+ from
with normal saline before concentration abnormality is extracellular to intracellular space, and protecting cells
addressed from the effects of increased K+
Once adequate volume is achieved, water deficit is Exogenous sources of potassium should be removed,
replaced using a hypotonic fluid including K+ supplementation in IV fluids
Rate of fluid administration should be titrated to K+ can be removed from the body using a cation-
achieve a decrease in serum sodium concentration exchange resin such as Kayexalate that binds K+ in
of no more than 1 mEq/l/h exchange for Na+
Overly rapid correction can lead to cerebral edema Immediate measures also should include attempts to
and herniation shift K+ intracellularly with glucose, insulin and
bicarbonate infusion and nebulized salbutamol(10-
2. Hyponatremia 20 mg)
Occurs when there is an excess of extracellular water When ECG changes are present, calcium chloride or
relative to sodium calcium gluconate (5-10 ml of 10% solution) should be
Extracellular volume can be high, normal, or low (Refer administered immediately
to Figure 8) All measures are temporary, lasting from 1 to 4 hours
In most cases, sodium concentration is decreased as a Dialysis should be considered in severe hyperkalemia
consequence of either sodium depletion or dilution when conservative measures fail
Symptomatic hyponatremia does not occur until serum
sodium level is 20 mEq/l 4. Hypokalemia
Clinical manifestations || Primarily central nervous More common than hyperkalemia in the surgical patient
system in origin (headache, confusion, seizures, coma) Caused by inadequate K+ intake, excessive renal K+
associated increases in intracranial pressure excretion, K+ loss in pathologic GI secretions, or
intracellular shifts from metabolic alkalosis or insulin
therapy (Refer to Table 12)
Clinical manifestations || Primarily related to failure of
normal contractility of GI smooth muscle (ileus,

TOPNOTCH MEDICAL BOARD PREP SURGERY SUPPLEMENT HANDOUT Page 8 of 81


For inquiries visit www.topnotchboardprep.com.ph or email us at topnotchmedicalboardprep@gmail.com
TOPNOTCH MEDICAL BOARD PREP SURGERY SUPPLEMENT HANDOUT - Jules Lopez,MD-MBA,Teddy Carpio,MD-MBA
For inquiries visit www.topnotchboardprep.com.ph or email us at topnotchmedicalboardprep@gmail.com
constipation), skeletal muscle (decreased reflexes,
weakness, paralysis), and cardiac muscle (arrest) QUICK REVIEW a
ECG changes || U waves, T-wave flattening, ST-
segment changes, and arrhythmias (with digitalis Normal Na+: 135-145 mEq/l
therapy) Symptomatichypernatremia are rare until serum
sodium exceeds 160 mEq/l
Table 12. Etiology of Potassium Abnormalities
Symptomatic hyponatremia does not occur until serum
Etiology of Potassium Abnormalities
sodium level is 20 mEq/l
Increased Intake
Potassium supplementation
Blood transfusions Normal K+: 3.5-5 mEq/l
Endogenous load/destruction: hemodialysis, Peaked T waves are the first ECG change seen in most
rhabdomyolysis, crush injury, GI hemorrhage patients with hyperkalemia
Increased Release T-wave flattening is seen in hypokalemia
Hyperkalemia
Acidosis Hypokalemia causes decreased deep tendon reflexes
Rapid rise of extracellular osmolality while hypomagnesemia and hypocalcemia causes
(hyperglycemia or mannitol) increased deep tendon reflexes
Impaired Excretion
Potassium-sparing diuretics
Renal insufficiency/failure
Inadequate Intake 5. Hypercalcemia
Dietary, potassium-free IV fluids Serum calcium level above the normal range of 8.5-
Potassium-deficient TPN 10.5 mEq/l or an increase in ionized calcium above
Excessive Potassium Excretion 4.2-4.8 mg/dl
Hyperaldosteronism Caused by primary hyperparathyroidism in the
Hypokalemia
Medications (Non-K+ sparing diuretics) outpatient setting and malignancy in hospitalized
GI losses patients
Direct loss of potassium from GI fluid Clinical manifestations || Neurologic impairment,
(diarrhea) musculoskeletal weakness and pain, renal dysfunction,
Renal loss of potassium
and GI symptoms (Refer to Table 13)
ECG changes || Shortened QT interval, prolonged PR
Treatment || Potassium repletion, the rate is
and QRS intervals, increased QRS voltage, T-wave
determined by the symptoms
flattening and widening, and atrioventricular block
Mild, asymptomatic hypokalemia: oral repletion is
Treatment is required when hypercalcemia is
adequate (KCl 40 mEq per enteral access x 1 dose)
symptomatic, which usually occurs when the serum
Asymptomatic hypokalemia, not tolerating enteral
level exceeds 12 mEq/l
nutrition: KCl 20 mEq IV q2h x 2 doses
Critical level for serum calcium is 15 mEq/l, when
If IV repletion is required, usually no more than 10
symptoms noted earlier may rapidly progress to death
mEq/h is advisable in an unmonitored setting
Treatment || Aimed at repleting the associated volume
K+ supplementation can be increased to 40 mEq/h
deficit and then inducing a brisk diuresis with normal
when accompanied by continuous ECG monitoring, and
saline
even more in the case of imminent cardiac arrest from a
malignant arrhythmia associated hypokalemia 6. Hypocalcemia
Caution should be done when oliguria or impaired renal Serum calcium level below 8.5 mEq/l or a decrease in
function is coexistent the ionized calcium level below 4.2 mg/dl
Causes include pancreatitis, malignancies associated
PHARMACOLOGY a with increased osteoclastic activity (breast and prostate
cancer), massive soft tissue infections such as
K+sparing Diuretics are competitive antagonists that necrotizing fasciitis, renal failure, pancreatic and small
either block the actions of aldosterone at the distal bowel fistulas, hypoparathyroidism, toxic shock
convoluted tubule, or directly inhibit sodium channels syndrome, and tumor lysis syndrome
o Aldosterone antagonists: Spironolactoneand Transient hypocalcemia also occurs after removal of a
Eplerenone parathyroid adenoma due to atrophy of the remaining
o Epithelial sodium channel blockers: Amiloride gland and avid bone remineralization
and Triamterene Neuromuscular and cardiac symptoms do not occur
Non K+-sparing Diureticsinclude loop diuretics and until the ionized fraction falls below 2.5 mg/dl
thiazides, which both inhibit Na+ and Cl- reabsorption Clinical manifestations || Neuromuscular symptoms
o Loop diuretics (Furosemide) inhibit the Na+- with decreased cardiac contractility (Refer to Table 13)
K+-2Cl- cotransporter in the thick ascending ECG changes || Prolonged QT interval, T-wave
limb of the loop of Henle inversion, heart block and ventricular fibrillation
o Thiazidesinhibit the Na+-Cl-transporter in the
distal tubule
MICROBIOLOGY a

Toxic Shock Syndromeis due to the Staphylococcus


aureus toxin, Toxic shock syndrome toxin (TSST-1)
Clinical manifestations ||Fever, hypotension, sloughing
of the filiform papillae (strawberry tongue),
desquamating rash, andmulti-organ involvement
Usually no site of pyogenic inflammation blood CS
negative
Common in tampon-using menstruating women or in
patients with nasal packing for epistaxis
Treatment || Remove the offending agent and to start
antibiotics (Clindamycin and Vancomycin)

TOPNOTCH MEDICAL BOARD PREP SURGERY SUPPLEMENT HANDOUT Page 9 of 81


For inquiries visit www.topnotchboardprep.com.ph or email us at topnotchmedicalboardprep@gmail.com
TOPNOTCH MEDICAL BOARD PREP SURGERY SUPPLEMENT HANDOUT - Jules Lopez,MD-MBA,Teddy Carpio,MD-MBA
For inquiries visit www.topnotchboardprep.com.ph or email us at topnotchmedicalboardprep@gmail.com

PATHOLOGY a Abdominal pain


Weakness Weakness
Bone pain Lethargy
Tumor Lysis Syndromeconsists of multiple electrolyte Neuromuscular
Confusion Decreased reflexes
abnormalities that may be seen after initiation of Coma
cancer treatment Hypertension Hypotension Arrest
Chemotherapy causes release of break-down products of Arrhythmia
Cardiovascular
dying cancer cells Worsening of digitalis
Among the electrolyte abnormalities include toxicity
hyperkalemia, hyperphosphatemia, hyperuricemia, Renal Polyuria -
and hypocalcemia Decreased Serum Levels
System Calcium Magnesium
Clinical consequences are acute uric acid nephropathy
Hyperactive reflexes Hyperactive reflexes
and acute renal failure Paresthesias Muscle tremors
Muscle cramps Tetany
Carpopedal spasm Positive Chvostek’s
Neuromuscular
Treatment || Calcium supplementation and correction Seizures and Trousseau’s
of other metabolic derangements Tetany signs
Asymptomatic hypocalcemia can be treated with oral Trousseau’s sign1 Delirium and
Chvostek’s sign2 seizures (severe)
or IV calcium
Cardiovascular Heart failure Arrhythmia
Acute symptomatic hypocalcemia should be treated 1Spasm resulting from pressure applied to the nerves and vessels of the
with IV 10% calcium gluconate to achieve a serum upper extremity with a blood pressure cuff
concentration of 7-9 mg/dl 2 Spasm resulting from tapping over the facial nerve
Associated deficits in magnesium, potassium, and pH
must also be corrected Treatment || Magnesium supplementation
Hypocalcemia will be refractory to treatment if Correction of magnesium depletion can be oral if
coexisting hypomagnesemia is not corrected first asymptomatic and mild or IV if symptomatic and severe
For those with severe deficits (<1 mEq/L) or those who
7. Hypermagnesemia
are symptomatic, 1 to 2 g of magnesium sulfate may
Rare but can be seen with severe renal insufficiency and
be administered IV over 15 minutes or 2 minutes if
parallel changes in potassium excretion
under ECG monitoring to correct torsades
o Magnesium-containing antacids and laxatives
To counteract the adverse side effects of a rapidly rising
can produce toxic levels in patients with renal
magnesium level and correct hypocalcemia (frequently
insufficiency/failure
associated with hypomagnesemia), simultaneous
o Excess intake in conjunction with TPN, or
administration of calcium gluconateis done
rarely massive trauma, thermal injury, and
severe acidosis, may be associated with
symptomatic hypermagnesemia QUICK REVIEW a
Clinical manifestations || Mainly GI with
neuromuscular dysfunction and impaired cardiac Normal Ca2+: 8.5-10.5 mEq/l
conduction (Refer to Table 13) Normal ionized Ca2+: 4.2-4.8 mg/dl
ECG changes || (similar to hyperkalemia) Increased PR Treatment is required when hypercalcemia is
interval, widened QRS complex, elevated T waves symptomatic, when the serum level exceeds 12 mEq/l
Treatment || Eliminate exogenous sources of Symptomatic hypocalcemia do not occur until the
magnesium, correct concurrent volume deficits and ionized fraction falls below 2.5 mg/dl
correct acidosis if present Hypocalcemia will be refractory to treatment if
To manage acute symptoms, calcium chloride(5-10ml) coexisting hypomagnesemia is not corrected first
should be administered to immediately antagonize the
cardiovascular effects F. ACID-BASE DISORDERS
If persistently elevated or with symptoms, dialysis may 1. Metabolic Acidosis
be necessary Results from an increased intake of acids, an increased
generation of acids, or an increased loss of bicarbonate
8. Hypomagnesemia Body compensates by producing buffers (extracellular
Magnesium depletion is a common problem in bicarbonate and intracellular buffers from bone and
hospitalized patients, particularly in the critically ill muscle), increasing ventilation (Kussmaul's
Result from alterations of intake, renal excretion and respirations),increasing renal reabsorption and
pathologic losses generation of bicarbonate, and increasing renal secretion
o Poor intake may occur in cases of starvation, of hydrogen
alcoholism, prolonged IV fluid therapy, and TPN Evaluation of a patient with metabolic acidosis includes
with inadequate supplementation of determination of the anion gap (AG), an index of
Magnesium unmeasured anions
o Losses are seen in cases of increased renal o AG = Na+ – (Cl- + HCO3-)
excretion from alcohol abuse, diuretic use, o Normal: <12 mmol/l
administration of amphotericin B, and primary Etiology of metabolic acidosis is listed inTable 14
aldosteronism, as well as GI losses from
diarrhea, malabsorption, and acute pancreatitis
Clinical manifestations || Neuromuscular and central Table 14. Etiology of Metabolic Acidosis
nervous system hyperactivity, similar to those of High Anion Gap Metabolic Acidosis (HAGMA)
calcium deficiency
ECG changes || Prolonged QT and PR intervals, ST- Exogenous acid ingestion Mnemonic: “MUDPILES”
Ethylene glycol Methanol
segment depression, flattening or inversion of P waves,
Salicylate Uremia (Renal failure)
torsades de pointes, and arrhythmias
Methanol Diabetic ketoacidosis
Can produce hypocalcemia and lead to persistent
Propylene glycol
hypokalemia
Paraldehyde
Endogenous acid production Infection, Iron, Isoniazid
Table 13. Clinical Manifestations of Abnormalities in Ca2+ and Mg+
Ketoacidosis Lactic acidosis
Increased Serum Levels
Lactic acidosis Ethylene glycol
System Calcium Magnesium
Renal insufficiency Salicylates
Anorexia Nausea/vomiting
Gastrointestinal
Nausea/vomiting
TOPNOTCH MEDICAL BOARD PREP SURGERY SUPPLEMENT HANDOUT Page 10 of 81
For inquiries visit www.topnotchboardprep.com.ph or email us at topnotchmedicalboardprep@gmail.com
TOPNOTCH MEDICAL BOARD PREP SURGERY SUPPLEMENT HANDOUT - Jules Lopez,MD-MBA,Teddy Carpio,MD-MBA
For inquiries visit www.topnotchboardprep.com.ph or email us at topnotchmedicalboardprep@gmail.com
Norma Anion Gap Metabolic Acidosis (NAGMA) Table 16. Etiology of Respiratory Acidosis
Etiology of Respiratory Acidosis
Acid administration (HCl)
Narcotics
Mnemonic: “HARD UP” Central nervous system injury
Loss of bicarbonate Hyperalimentation Pulmonary (secretions, atelectasis, mucus plug, pneumonia, pleural
Acetazolamide (Carbonic effusion)
GI losses (diarrhea, fistulas) anhydrase inhibitor) Pain from abdominal or thoracic injuries or incisions
Renal tubular acidosis Limited diaphragmatic excursion from intra-abdominal pathology
Ureterosigmoidoscopy Diarrhea (abdominal distention, abdominal compartment syndrome, ascites)
Ureteroenteric fistula
Renal tubular acidosis Pancreticoduodenal
fistula 4. Respiratory Alkalosis
Carbonic anhydrase inhibitor In the surgical patient, most cases are acute and
secondary to alveolar hyperventilation
Causes include pain, anxiety, neurologic disorders
Lactic acidosis is a common cause of severe metabolic (central nervous system injury and assisted ventilation),
acidosis in surgical patients drugs (salicylates), fever, gram-negative bacteremia,
In circulatory shock, lactate is produced in the presence thyrotoxicosis, and hypoxemia
of hypoxia from inadequate tissue perfusion Acute hypocapnia can cause an uptake of potassium and
Treatment || Restore perfusion with volume phosphate into cells and increased binding of calcium to
resuscitation rather than to attempt to correct with albumin, leading to symptomatic hypokalemia,
exogenous bicarbonate hypophosphatemia, and hypocalcemia with
With adequate perfusion, lactate is rapidly metabolized subsequent arrhythmias, paresthesias, muscle cramps,
by the liver and the pH level returns to normal and seizures
Administration of bicarbonate for the treatment of Treatment || Directed at the underlying cause
metabolic acidosis is controversial Direct treatment of the hyperventilation using
o Overzealous administration of bicarbonate can controlled ventilation may also be required
lead to metabolic alkalosisand can be
associated with arrhythmias
QUICK REVIEW a
o An additional disadvantage is that sodium
bicarbonate actually can exacerbate
Evaluation of a patient with metabolic acidosis includes
intracellular acidosis
determination of the anion gap (AG) to differentiate
HAGMA from NAGMA (TIP: Memorize the mnemonics!)
2. Metabolic Alkalosis
Normal AG is <12 mmol/l
Results from the loss of fixed acids orgain of
Treatment of metabolic acidosis is to restore perfusion
bicarbonate(Refer to Table 15)
with volume resuscitation rather than exogenous
Majority of patients will have hypokalemia, because
bicarbonate
extracellular potassium ions exchange with intracellular
Metabolic alkalosis is associated with hypokalemia
hydrogen ions and allow the hydrogen ions to buffer
excess HCO3
Treatment || Includes replacement of the volume
deficit with isotonic saline and then potassium
replacement once adequate urine output is achieved REVIEW QUESTIONS a

Table 15. Etiology of Metabolic Alkalosis 1. A patient develops a high output fistula following
Increased bicarbonate generation abdominal surgery. The fluid is sent for evaluation with
Chloride losing (urinary chloride > 20 mEq/l) the following results: Na+ 135, K+ 5, Cl- 70. Which of the
Mineralocorticoid excess following is the most likely source of the fistula?
Profound potassium depletion a. Stomach
Chloride sparing (urinary chloride < 20 mEq/l) b. Small bowel
Loss from gastric secretions (emesis or nasogastric suction) c. Pancreas
Diuretics d. Biliary tract
Excess administration of alkali
Acetate in parenteral nutrition Answer: C
Citrate in blood transfusions The composition of pancreatic secretions is marked by
Antacids
high level of bicarbonate (Refer to Table 8), compared
Bicarbonate
to other GI secretions. In this example, the patient has a
Milk-alkali syndrome
Impaired bicarbonate excretion
total of 140 mEq of cation (Na+ + K+) and only 70 mEq of
Decreased glomerular filtration anion (Cl-). The remaining 70 mEq (to balance the 140
Increased bicarbonate reabsorption (hypercarbia or potassium mEq of cation) must be bicarbonate.
depletion)
2. A postoperative patient with a potassium of 2.9 is given
1 mEq/kg replacement with KCl (potassium chloride).
3. Respiratory Acidosis Repeat tests after the replacement show the serum K to
Associated with retention of CO2 secondary to be 3.0. The most likely diagnosis is:
decreased alveolar ventilation a. Hypomagnesemia
Principal causes are listed in Table 16 b. Hypocalcemia
Because compensation is primarily a renal mechanism, c. Metabolic acidosis
it is a delayed response d. Metabolic alkalosis
In the chronic form, partial pressure of arterial CO2
remains elevated and the bicarbonate concentration Answer: A
rises slowly as renal compensation occurs In cases in which potassium deficiency is due to
Treatment || Directed at the underlying cause magnesium depletion, potassium repletion is difficult
Measures to ensure adequate ventilation through unless hypomagnesemia is first corrected.
bilevel positive airway pressure or endotracheal Alkalosis will change serum potassium (a decrease in 0.3
intubationare also initiated mEq/l for every 0.1 increase in pH above normal). This is
not enough to explain the lack of response to repletion in
the patient. Metabolic acidosis would not decrease
potassium. Calcium does not play a role in potassium
metabolism.
TOPNOTCH MEDICAL BOARD PREP SURGERY SUPPLEMENT HANDOUT Page 11 of 81
For inquiries visit www.topnotchboardprep.com.ph or email us at topnotchmedicalboardprep@gmail.com
TOPNOTCH MEDICAL BOARD PREP SURGERY SUPPLEMENT HANDOUT - Jules Lopez,MD-MBA,Teddy Carpio,MD-MBA
For inquiries visit www.topnotchboardprep.com.ph or email us at topnotchmedicalboardprep@gmail.com
Extent of vasoconstriction varies with the degree of
3. Which of the following is a cause of acute vessel injury (more pronounced in vessels with medial
hypophosphatemia? smooth muscles)
a. Chronic ingestion of magnesium containing
laxatives 2. Platelet Plug Formation
b. Insulin coma Platelets do not normally adhere to each other or to the
c. Refeeding syndrome vessel wall but during vascular disruption, they form a
d. Rhabdomyolosis hemostatic plugthat aids in cessation of bleeding
Injury to the intimal layer in the vascular wall exposes
Answer: C von Willebrand's factor (vWF), a subendothelial
Acute hypophosphatemia is usually caused by an protein, where platelets adhere via glycoprotein I/IX/V
intracellular shift of phosphate in association with After adhesion, platelets initiate a release reaction that
respiratory alkalosis, insulin therapy, refeeding recruits other platelets to seal the disrupted vessel
syndrome, and hungry bone syndrome. Clinical The aforementioned process, mediated by adenosine
manifestations include cardiac dysfunction or muscle diphosphate (ADP) and serotonin,is reversible and is
weakness but are usually absent until levels fall known as primary hemostasis
significantly. Refer to page 8 for a discussion on In the second wave of platelet aggregation,
refeeding syndrome. anotherrelease reaction occurs that results in
Magnesium containing laxatives can cause compaction of the platelets viaglycoprotein IIb/IIIa into
hypermagnesemia in patients with renal failure but does a plug
not affect phosphorous. Patients with insulin coma With fibrinogen as a cofactor, this process, mediated by
(hypoglycemia) are not at risk for hypophosphatemia. ADP, Ca2+, serotonin, TXA2, is irreversible
Rhabdomyolosis is associated with hyperkalemia and
hyperphosphatemia 3. Fibrin Formation / Coagulation
As a consequence of the release reaction, alterations
occur in the phospholipids of the platelet membrane
HEMOSTASIS, SURGICAL BLEEDING, that initiates coagulation
AND TRANSFUSION Coagulation cascade typically has been depicted as two
intersecting pathways
A. Hemostasis o Intrinsic pathway begins with factor XII and
B. Evaluation of Hemostatic Risk through a series of enzymatic reactions, which
C. Surgical Bleeding is intrinsic to the circulating plasma and no
D. Special Cases surface is required to initiate the process
E. Transfusion o Extrinsic pathwayrequires exposure of
tissue factor on the surface of the injured
vessel wall to initiate the arm of the cascade
A. HEMOSTASIS beginning with factor VII
Function is to limit blood loss from an injured vessel o The two arms of the coagulation cascade
Four major physiologic events participate in the merge to a common pathway at factor X, and
hemostatic process(Refer to Figure 9): activation of factors II (prothrombin) and I
o Vascular constriction (fibrinogen)proceeds in sequence
o Platelet plug formation Secondary hemostasis or fibrin clot formation
o Fibrin formation occurs after conversion of fibrinogen to fibrin
o Fibrinolysis
Figure 9. Biology of Hemostasis 4. Fibrinolysis
During the wound-healing process, the fibrin clot
undergoes fibrinolysis, which permits restoration of
blood flow
This is initiated at the same time as the clotting
mechanism under the influence of circulating kinases,
tissue activators, and kallikrein, which are present in
the vascular endothelium
Plasmindegrades the fibrin mesh at various places,
which leads to the production of circulating fragments
that are cleared by proteases or by the kidney and liver

B. EVALUATION OF HEMOSTATIC RISK


1. Preoperative Evaluation of Hemostasis
Most important component of the bleeding risk
assessment is a directed bleeding history
When history is unreliable or incomplete or when
abnormal bleeding is suggested, a formal evaluation of
hemostasis should be performed before surgery
o Hemoglobin levels below 7 or 8 g/dl appear to
be associated with significantly more
perioperative complications
o Determination of the need for preoperative
1. Vascular Constriction transfusion must consider factors other than
Initial response to vessel injury the absolute hemoglobin level, including the
Dependent on local contraction of smooth muscle presence of cardiopulmonary disease, type of
o Thromboxane A2 (TXA2) ,potent constrictor surgery, and likelihood of surgical blood loss
of smooth muscle,is produced locally at the Laboratory tests of hemostatic parameters in patients
site of injury with low risk of bleeding are NOT required
o Endothelin ,also a potent vasoconstrictor, is
synthesized by injured endothelium and 2. Evaluation of Intraoperative or Postoperative Bleeding
serotonin Excessive bleeding during or after a surgical procedure
o Bradykinin and fibrinopeptidesare capable may be the result of ineffective hemostasis, blood
of contracting vascular smooth muscle. transfusion, undetected hemostatic defect, disseminated
TOPNOTCH MEDICAL BOARD PREP SURGERY SUPPLEMENT HANDOUT Page 12 of 81
For inquiries visit www.topnotchboardprep.com.ph or email us at topnotchmedicalboardprep@gmail.com
TOPNOTCH MEDICAL BOARD PREP SURGERY SUPPLEMENT HANDOUT - Jules Lopez,MD-MBA,Teddy Carpio,MD-MBA
For inquiries visit www.topnotchboardprep.com.ph or email us at topnotchmedicalboardprep@gmail.com
intravascular coagulation (DIC) or consumptive
coagulopathy, and/or fibrinolysis Treatment || Depends on the extent and cause of
platelet reduction
C. SURGICAL BLEEDING o Platelets are given preoperatively to rapidly
1. Systemic Bleeding Disorders increase the count in surgical patients
Thrombocytopenia, secondary to any platelet o A count of >50,000/L generally requires no
pathology, is the most common abnormality of specific therapy
hemostasis that results in bleeding in surgical patients o One unit of platelet concentrate is expected
Systemic causes of surgical bleeding can either be to increase the circulating platelet count by
inherited or acquired (Refer to Table 17) ~10,000/L in the average 70-kg person
Inherited platelet functional defects include o In patients whose thrombocytopenia is
abnormalities of platelet surface proteins, abnormalities refractory to standard platelet transfusion,
of platelet granules, and enzyme defects the use of human leukocyte antigen (HLA)
Acquired abnormalities of platelets may be quantitative compatible plateletshas proved effective
or qualitative, although some patients have both types
o Quantitative defects may be a result of failure 2. Local Hemostasis
of production due to bone marrow disorders, Significant surgical bleeding usually is caused by
shortened survival, or sequestration ineffective local hemostasis
o Qualitative defects include massive Goal is to prevent further blood loss from a disrupted
transfusionand drugs that interfere with vessel that has been incised or transected
platelet function Hemostasis may be accomplished by interrupting the
flow of blood to the involved area or by direct closure of
Table 17. Etiology of Surgical Bleeding the blood vessel wall defect
Congenital Factor Deficiencies o Mechanical procedure: When pressure is
Coagulation Factor Deficiencies applied (whether through direct digital
von Willibrand’s Disease pressure, hemostatic clamp, or tourniquet) to
Platelet Functional Defects an artery proximal to an area of bleeding,
Acquired Hemostatic Defects
profuse bleeding may be reduced so that more
Platelet Abnormalities: Quantitative
definitive action is permitted
Leukemia
Myeloproliferative disorders
o Thermal agents: Heat (via cautery or
Failure of Vitamin B12 or Folate deficiency harmonic scalpel) achieves hemostasis by
Production Chemotherapy or radiation therapy denaturation of protein that results in
Acute alcohol intoxication coagulation of large areas of tissue
Viral infections o Topical hemostatic agents: Include physical
Immune-mediated disorders (Idiopathic or mechanical, caustic, biologic, and
thrombocytopenia, Heparin-induced physiologic agents that works either by
thrombocytopenia, Autoimmune disorders or B- inducing protein coagulation and precipitation
Decreased cell maligancies, Secondary thrombocytopenia) or activating biologic responses to bleeding
Survival Disseminated intravascular coagulation
Disorders related to platelet thrombi D. SPECIAL CASES
(Thrombocytopenic purpura, Hemolytic uremic
syndrome)
1. Disseminated Intravascular Coagulation (DIC)
Portal hypertension An acquired syndrome characterized by intravascular
Sarcoid activation of coagulation
Sequestration Can originate from and cause damage to the
Lymphoma
Gaucher’s disease microvasculature, which if sufficiently severe, can
Platelet Abnormalities: Qualitative produce organ dysfunction
Massive transfusion Excessive thrombin generation leads to microthrombus
Therapeutic administration of platelet inhibitors formation, followed by consumption and depletion of
Myeloproliferative disorders coagulation factors and platelets, which leads to the
Disease states Monoclonal gammopathies classic picture of diffuse bleeding
Liver disease
Causes include the following:
o Central nervous system injuries with
embolization of brain matter
PATHOLOGY a o Fractures with embolization of bone marrow
o Malignancy
Bernard-Soulier Syndromeis caused by a defect in the o Organ injury (severe pancreatitis, liver failure)
glycoprotein Ib/IX/V receptor for vWF, leading to o Certain vascular abnormalities (aneurysms)
defective platelet adhesion o Others: snakebites, illicit drugs, transfusion
o Decreased platelet count reactions, transplant rejection, and sepsis
o Treatment || Platelet transfusion Diagnosis is made on the basis of an inciting cause with
Glanzmann Thrombastheniais caused by a defect in associated thrombocytopenia, prolonged PT, low
the platelet glycoprotein IIb/IIIa complex , leading to fibrinogen level, and elevated levels of fibrin
defective platelet aggregation markers (fibrin degradation products, D-dimer, soluble
o Normal platelet count fibrin monomers)
o Treatment || Platelet transfusion Treatment || Relieving the patient's causative primary
medical or surgical problem and maintaining adequate
perfusion
If there is active bleeding, hemostatic factors should be
replaced using fresh frozen plasma (FFP), which
generally is sufficient to correct the hypofibrinogenemia

2.Anticoagulation and Bleeding


Spontaneous bleeding can be a complication of
anticoagulant therapy with either heparin, warfarin,or
low molecular weight heparin
o Risk of spontaneous bleeding with heparinis
relatively high but reduced with continuous
infusion technique
TOPNOTCH MEDICAL BOARD PREP SURGERY SUPPLEMENT HANDOUT Page 13 of 81
For inquiries visit www.topnotchboardprep.com.ph or email us at topnotchmedicalboardprep@gmail.com
TOPNOTCH MEDICAL BOARD PREP SURGERY SUPPLEMENT HANDOUT - Jules Lopez,MD-MBA,Teddy Carpio,MD-MBA
For inquiries visit www.topnotchboardprep.com.ph or email us at topnotchmedicalboardprep@gmail.com
o Therapeutic anticoagulation is more reliably o Clopidogrelinhibit platelet function through
achieved withlow molecular weight selective irreversible inhibition of ADP-
heparinbecause laboratory testing is not induced platelet aggregation
routinely done, which makes them attractive General recommendation is that a period of ~7 days is
options for outpatient anticoagulation required from the time the drug is stopped until an
o Warfarin is used for long-term outpatient elective procedure can be performed
anticoagulation in various clinical conditions Timing of urgent and emergent surgeries is unclear
including deep vein thrombosis, valvular heart Preoperative platelet transfusions may be beneficial
disease (with or without prosthetic valves),
atrial fibrillation, and recurrent myocardial 3.Coagulopathy of Liver Disease
infarction Liver plays a key role in hemostasis because it
synthesizes manycoagulation factors
PHARMACOLOGY a Most common coagulation abnormalities associated
with liver dysfunction are thrombocytopenia and
Warfarin inhibits vitamin K epoxide reductase and impaired humoral coagulation function manifested
thereby interferes with production of functional vitamin as prolonged PT and increase in the International
K-dependent clotting and anticlotting factors Normalized Ratio (INR)
Side effect||bleeding, warfarin-induced skin necrosis Thrombocytopenia is related to hypersplenism, reduced
production of thrombopoietin, and immune-mediated
Mnemonic: “Ethel Booba takes Phen- destruction of platelets
phen and Refuses Greasy Carb Shakes” Before any therapy for thrombocytopenia is initiated,
Cytochrome P450 Ethanol the actual need for correction should be strongly
Inducers considered
Barbiturates
increase clearance Treatment || Platelet transfusions; however, the
Phenytoin
and reduce the effect typically lasts only several hours
Rifampicin
anticoagulant Potential alternative strategy is administration of
effect of warfarin Griseofulvin
Carbamazepine interleukin-11, a cytokine that stimulates proliferation
St. John’s Wort / Smoking of hematopoietic stem cells and megakaryocyte
Mnemonic: “Inhibitors Stop Cyber Kids progenitors
from Eating GRApefruit Q” Less well accepted option is splenectomy or splenic
Isoniazid embolization to reduce hypersplenism but reduced
Cytochrome P450 Sulfonamides splenic blood flow can reduce portal vein flow with
inhibitors subsequent development of portal vein thrombosis.
Cimetidine
reduce clearance
Ketoconazole
and increase the 4. Coagulopathy of Trauma
Erythromycin
anticoagulant Recognized causes of traumatic coagulopathy include
effect of warfarin Grapefruit juice
acidosis, hypothermia, and dilution of coagulation factors
Ritonavir
Significant proportion of trauma patients arrive at the
Amiodarone
ER coagulopathic, and this early coagulopathy is
Quinidine associated with increased mortality
Shockhas been postulated to induce coagulopathy
Bleeding complications can be manifested through systemic activation of anticoagulant and
throughhematuria, soft tissue bleeding, intracerebral fibrinolytic pathways
bleeding, skin necrosis, and abdominal bleeding Hypoperfusion causes activation of thrombomodulin(on
Bleeding into the abdominal cavity is the most the surface of endothelial cells), which complexes with
common complication of warfarin therapy circulating thrombin thereby inducing not only an
Intramural bowel hematoma is the most common anticoagulant statebut also enhancing fibrinolysis
cause of abdominal pain in patients receiving
anticoagulation therapy 5. Massive Transfusion
Certain surgical procedures should not be performed Well-known cause of thrombocytopenia due to
such as procedures involving the central nervous hypothermia, dilutional coagulopathy, platelet
system or the eye dysfunction, fibrinolysis, or hypofibrinogenemia
Impaired ADP-stimulated aggregation occurs with
Table 18. Reversal of anticoagulation for patients undergoing surgery massive transfusion(>10 units of packed RBC) leading to
Reversal of Heparin Therapy surgical bleeding
Not indicated when aPTT is <1.3 times the control value
Emergency Discontinue drug and use ofprotamine sulfate for D. TRANSFUSION
surgery more rapid reversal of anticoagulation
Reversal of Warfarin Therapy
General indications for transfusion is listed in Table 19
Not indicated when the INR is <1.5
Table 19. Indications for Replacement of Blood and its Elements
Discontinue drug several days before the operation
with monitoring of prothrombin concentration General Indications for Transfusion
(>50% is safe) Improvement in Oxygen-carrying capacity is primarily a
Parenteral administration of vitamin K is indicated Oxygen function of RBC
Elective Carrying Therefore, transfusion of RBC should
in patients with biliary obstruction or malabsorption
surgery Capacity augment oxygen-carrying capacity
who may be vitamin K deficient
Low molecular weight heparin should be Critically ill patients frequently receive
Treatment of
administered while the INR is decreasing in patients transfusions at a hemoglobin level
Anemia
with high risk of thrombosis approaching 9 g/dl
Emergency Rapid reversal of anticoagulation can be Most common indication for blood
surgery accomplished with FFP transfusion in surgical patients is
thereplenishment of the blood volume
Measurements of hemoglobin levels or
hematocrit are frequently used to assess
Other drugs that interfere with platelet function are
Volume blood loss misleading in acute loss,
aspirin, clopidogrel, dipyridamole, and glycoprotein Replacement because levels can be normal in spite of
IIb/IIIa inhibitors severely contracted blood volume
o Aspirin inhibit platelet function through Estimated total blood volume is 7-8% of
irreversible acetylation of platelet TBW
prostaglandin synthase Blood loss of up to 20% of total blood volume:
Replaced with crystalloid solution
TOPNOTCH MEDICAL BOARD PREP SURGERY SUPPLEMENT HANDOUT Page 14 of 81
For inquiries visit www.topnotchboardprep.com.ph or email us at topnotchmedicalboardprep@gmail.com
TOPNOTCH MEDICAL BOARD PREP SURGERY SUPPLEMENT HANDOUT - Jules Lopez,MD-MBA,Teddy Carpio,MD-MBA
For inquiries visit www.topnotchboardprep.com.ph or email us at topnotchmedicalboardprep@gmail.com
Blood loss >20% of total blood Treatment ||Administration of antihistamines or in
volume:Addition of packed RBC, and in the more serious cases, use of epinephrine or steroids may
case of massive transfusion, the addition of be indicated
FFP
3. Respiratory Complications
Circulatory overload can occur with rapid infusion of
QUICK REVIEW a blood, plasma expanders, and crystalloids, particularly
in older patients with underlying heart disease
Thrombocytopenia is the most common abnormality of Clinical manifestations ||dyspnea, rales,and cough
hemostasis Treatment || Initiate diuresis, slow the rate of blood
Significant surgical bleeding usually is caused by administration, and minimize delivery of fluids while
ineffective local hemostasis blood products are being transfused
Most important management of DIC is treatment of the Another significant respiratory complication is
underlying cause Transfusion-related Acute Lung Injury (TRALI)
Bleeding into the abdominal cavity is the most o Defined as noncardiogenic pulmonary edema
common complication of warfarin therapy related to transfusion
Intramural bowel hematoma is the most common o Can occur with the administration of any
cause of abdominal pain in patients receiving plasma-containing blood product
anticoagulation therapy o Clinical manifestations ||similar to those of
A period of ~7 days is required from the time aspirin circulatory overload and often accompanied
and/or clopidogrel is stopped until an elective procedure by fever, rigors, and bilateral pulmonary
can be performed infiltrates on chest radiograph
Most common coagulation abnormalities associated with o Most commonly occurs within 1 to 2 hours
liver dysfunction are thrombocytopenia and impaired after the onset of transfusion, but virtually
humoral coagulation function always before 6 hours
Most common indication for blood transfusion in o Etiology is not well established, but TRALI is
surgical patients is volume replacement thought to be related to anti-HLA or anti-
human neutrophil antigen antibodies in
transfused blood that primes neutrophils in
Complications of transfusionis primarily related to the pulmonary circulation.
blood-induced proinflammatory responses o Treatment || Discontinuation of any
Complications (discussed below) occur in transfusion, notification of the transfusion
approximately 10% of all transfusions, but <0.5% are service, and provision of pulmonary support
serious (from supplemental oxygen to mechanical
ventilation)
1. Febrile Non-hemolytic Reactions
Defined as an increase in temperature [>1°C (1.8°F)] 4. Hemolytic Reactions
associated with a transfusion Can be classified as either acute ordelayed(Refer to
Approximately 1% of all transfusions Table 20)
Preformed cytokines in donated blood and recipient
antibodies reacting with donated antibodies are Table 20. Classification of Hemolytic Reactions
postulated causes Classification of Hemolytic Reactions
Occur with the administration of ABO-
Can be reduced by the use of leukocyte-reduced blood
incompatible blood
products with
Fatal in up to 6% of cases
Pretreatment with paracetamol reduces the severity of Contributing factors include technical or clerical
the reaction errors in the laboratory andadministration of
Rare but potentially lethal febrile reaction is secondary wrong blood type
to bacterial contamination of infused blood Characterized by intravascular hemolysis and
o Gram-negative organisms, especially consequent hemoglobinemia and hemoglobinuria
Yersinia enterocolitica and Pseudomonas Clinical manifestations ||pain at the site of
species are the most common cause transfusion, facial flushing, and back and chest
Acute
pain, associated with fever, respiratory distress,
o Most cases are associated with the Hemolytic
hypotension, and tachycardia
administration of platelets Reaction
In anesthetized patients, diffuse bleeding and
o Pathogenesis is related to lability of factor V, hypotension are the hallmarks
which appears necessary for this interaction Positive Coombs' test is diagnostic
o Results in sepsis and death in 25% of patients Treatment || stop transfusion, get a sample of the
o Clinical manifestations ||fever and chills, recipient's blood and send along with the suspect
tachycardia, and hypotension, GI symptoms unit to the blood bank for comparison with the
(abdominal cramps, vomiting, and diarrhea), pretransfusion samples
and hemorrhagic manifestations such as Urine output should be monitored and adequate
hemoglobinemia, hemoglobinuria, and DIC hydration maintained to prevent precipitation of
hemoglobin within the tubules
o If suspected, transfusion should be
Reactions occur 2 to 10 days after transfusion
discontinued and the blood cultured
Occur when an individual has a low antibody titer
o Treatment ||Administration of oxygen, at the time of transfusion
adrenergic blocking agents, and antibiotics Characterized by extravascular hemolysis, mild
Delayed anemia, indirect hyperbilirubinemia, decreased
2. Allergic Reaction Hemolytic haptoglobin levels, low-grade hemoglobinemia
Occurs in ~1% of all transfusions Reaction and hemoglobinuria
Reactions usually are mild Clinical manifestations ||fever and jaundice
Clinical manifestations ||rash, urticaria, and fever Coombs' test usually yields a positive result
Treatment || Do not usually require specific
within 60 to 90 minutes of the start of the transfusion
intervention
In rare instances, anaphylactic shock develops
Caused by transfusion of antibodies from
hypersensitive donors or the transfusion of antigens to 5. Transmission of Disease
which the recipient is hypersensitive Among the diseases that have been transmitted by
Can occur after the administration of any blood product transfusion are malaria, Chagas' disease, brucellosis,
and, very rarely, syphilis

TOPNOTCH MEDICAL BOARD PREP SURGERY SUPPLEMENT HANDOUT Page 15 of 81


For inquiries visit www.topnotchboardprep.com.ph or email us at topnotchmedicalboardprep@gmail.com
TOPNOTCH MEDICAL BOARD PREP SURGERY SUPPLEMENT HANDOUT - Jules Lopez,MD-MBA,Teddy Carpio,MD-MBA
For inquiries visit www.topnotchboardprep.com.ph or email us at topnotchmedicalboardprep@gmail.com
Transmission of hepatitis C virus and HIV-1has been SURGICAL INFECTIONS AND SHOCK
dramatically minimized by the introduction of better
antibody and nucleic acid screening for these pathogens A. Definitions
Hepatitis B virus transmission may still occur in about 1 B. Surgical Wounds Classification
in 100,000 transfusions in nonimmune recipients C. Prevention and Treatment of Surgical Infections
D. Infections of Significance in Surgical Patients
QUICK REVIEW a E. Shock

Gram-negative organismsare the most common cause


of bacterial contamination of infused blood, especially A. DEFINITIONS
with platelet administration 1. Infection (Refer to Figure 10 )
Identifiable source of microbial insult
Transfusion-related Acute Lung Injury (TRALI) most 2. Systemic Inflammatory Response Syndrome (SIRS)
commonly occurs within 1 to 2 hours after the onset of Two or more of the following criteria met:
transfusion, but virtually always before 6 hours o Temperature 38 C or 36 C
o Heart rate 90 beats per minute
o Respiratory rate 20 breaths per minute or
Acute hemolytic reaction is characterized by PaCO2 32 mmHg or mechanical ventilation
intravascular hemolysis while delayed reaction is o White blood cell count 12,000/uL or
characterized by extravascular hemolysis
4,000/uL or 10% band forms
3. Sepsis
SIRS + Identifiable source of infection
4. Severe Sepsis
REVIEW QUESTIONS a
Sepsis + Organ dysfunction
5. Septic Shock
1. What percentage of platelets can be sequestered in the
Sepsis + Cardiovascular collapse (needs vasopressors)
spleen?
a. 15% Figure 10. Relationship between infection and SIRS. Sepsis is the
b. 30% presence both of infection and SIRS, shown here as the intersection of
c. 45% these two areas. Other conditions may cause SIRS as well (trauma,
d. 60% aspiration, etc.). Severe sepsis (and septic shock) are both subsets of
sepsis.
Answer: B
Platelets are anucleate fragments of megakaryoctes. The
normal circulating number of platelets ranges between
150,000 and 400,000/L. Up to 30% of circulating
platelets may be sequestered in the spleen. If not
consumed in a clotting reaction, platelets are normally
removed by the spleen and have an average life span of 7
to 10 days.

2. A patient on chronic warfarin therapy presents with


acute appendicitis. INR is 1.4. Which of the following is
the most appropriate management?
a. Proceed immediately with surgery without
stopping the warfarin
b. Stop the warfarin, give FFP, and proceed with B. SURGICAL WOUNDS CLASSIFICATION
surgery *Based on the magnitude of bacterial load at the time of surgery
c. Stop the warfarin and proceed with surgery in **IR = Infection rate
8-12 hours 1. Clean (Class I)
d. Stop the warfarin and proceed with surgery in Include those in which no infection is present
24-36 hours Only skin microflora potentially contaminate the wound
No hollow viscus (that contains microbes) is entered
Answer: A Class ID wounds are similar except that a prosthetic
When the INR <1.5 in a patient taking warfarin, reversal device (e.g. mesh or valve) is inserted
of anticoagulation therapy may not be necessary. (Refer Example: Hernia repair, Breast biopsy(IR: 1-5.4%)
to Table 18). However, meticulous surgical technique is
mandatory, and the patient must be observed closely 2. Clean/Contaminated (Class II)
throughout the postoperative period. Include those in which a hollow viscus such as
respiratory, GI, or GU tracts with inherent bacterial flora
3. What percent of the population is Rh negative? is opened under controlled circumstances without
a. 5% significant spillage of contents
b. 15% Example: Cholecystectomy, Elective GI surgery(not colon)
c. 25% (IR: 2.1-9.5%), Colorectal surgery(IR: 9.4-25%)
d. 35%
3. Contaminated (Class III)
Answer: B Include open accidental wounds encountered early
Rh negative recipients should receive transfusion only of after injury, those with extensive introduction of
Rh negative blood. However, this groups represents only bacteria into a normally sterile area of the body due
15% of the population. Therefore, the administration of to major breaks in sterile technique (e.g. open cardiac
Rh positive blood is acceptable if Rh negative blood is massage), gross spillage of viscus contents such as
not available. However, Rh positive blood should not be from the intestine, or incision through inflamed albeit
transfused to Rh negative females who are of nonpurulent tissue
childbearing age. Example: Penetrating abdominal trauma, large tissue
injury, enterotomy (IR: 3.4-13.2%)

TOPNOTCH MEDICAL BOARD PREP SURGERY SUPPLEMENT HANDOUT Page 16 of 81


For inquiries visit www.topnotchboardprep.com.ph or email us at topnotchmedicalboardprep@gmail.com
TOPNOTCH MEDICAL BOARD PREP SURGERY SUPPLEMENT HANDOUT - Jules Lopez,MD-MBA,Teddy Carpio,MD-MBA
For inquiries visit www.topnotchboardprep.com.ph or email us at topnotchmedicalboardprep@gmail.com
4. Dirty (Class IV) preparation or considerable spillage of colon contents)
Include traumatic wounds in which a significant o Prophylaxis merges into empiric therapy in
delay in treatment has occurred and in which situations in which the risk of infection
necrotic tissue is present, those created in the increases markedly because of intraoperative
presence of overt infection as evidenced by the findings
presence of purulent material, and those created to o Limited to a short course of drug (3-5 days),
access a perforated viscus accompanied by a high and should be curtailed based on
degree of contamination microbiologic data (i.e. culture and sensitivity
Example: Perforated diverticulitis, necrotizing soft tissue pattern) coupled with improvements in the
infections (IR: 3.1-12.8%) clinical course of the patient
o Manner in which therapy is used differs
C. PREVENTION AND TREATMENT OF SURGICAL INFECTIONS depending on whether the infection is
Resident microflora of the skin and other barrier monomicrobial or polymicrobial
surfaces represent a potential source of microbes that
can invade the body during trauma, thermal injury, or Table 22. General principles in empiric therapy
elective or emergent surgical intervention Empiric Therapy
Maneuvers to diminish the presence of exogenous Frequently are nosocomial infections
occurring in postoperative patients, such
(surgeon and operating room environment) and asUTIs, pneumonia, or bacteremia
endogenous (patient) microbes consist of the use of Therapy should be initiated in patients with
mechanical, chemical, and antimicrobial modalities, or a evidence of SIRS, coupled with evidence of local
combination of these methods infection (e.g., an infiltrate on chest X-ray plus a
These modalities are NOT capable of sterilizing the positive Gram's stain in BAL sample)
hands of the surgeon or the skin or epithelial surfaces of Within 24 to 72 hours, culture and sensitivity
the patient BUT the inoculum can be reduced Monomicrobial reports will allow directed antibiotic regimen
considerably Empiric regimen for common infections are as
follows:
Thus, entry through the skin, into the soft tissue, and
o UTI: 3-5 days
into a body cavity or hollow viscus invariably is o Pneumonia: 7-10 days
STILLassociated with the introduction of some o Bacteremia: 7-14 days
degree of microbial contamination o Osteomyelitis, endocarditis, or
Therefore, antimicrobial agents should be given in prosthetic infections: 6-12
patients who undergo procedures that may be associated weeks
with the ingress of significant numbers of microbes(e.g., Primary therapeutic modality is source
colonic resection) or in whom the consequences of any control(discussed below) but antimicrobial
agents play an important role as well
type of infection due to said process would be dire (e.g.,
Culture results are of lesser importance in
prosthetic vascular graft infection)
managing these infections, as it has been
demonstrated that only a limited group of
1. Appropriate Use of Antimicrobial Agents Polymicrobial
microbes predominate in the established
Prophylaxis is the administration of an antimicrobial infection, selected from a large number present
agent(s)before and during the operative procedure at the time of initial contamination
to reduce the number of microbes that enter the tissue As such, antibiotic regimen should NOT be
or body cavity modified solely on the basis of culture
o Only a single dose of antibiotic is required, information
and only for certain types of surgical
procedures (Refer to Table 21)
2. Source Control
o Patients who undergo complex, prolonged
procedures in which the duration of the Primary precept of surgical infectious disease therapy
operation exceeds the serum drug half-life consists of drainage of all purulent material,
should receive an additional dose(s) debridement of all infected, devitalized tissue, and debris,
o Administration of postoperative doses and/or removal of foreign bodies at the site of infection,
DOES NOT provide additional benefit, and plus remediation of the underlying cause of infection
should be discouraged, as it is costly and is o Discrete, walled-off purulent fluid collection
associated with increased rates of microbial (abscess) requires drainage via percutaneous
drug resistance drain insertion or an incision and drainage
o Ongoing source of contamination (e.g. bowel
Table 21. Prophylactic therapy perforation) or presence of an aggressive,
Site Antibiotic Alternative rapidly-spreading infection (e.g. necrotizing
Cardiovascular Cefazolin or Cefuroxime Vancomycin soft tissue infection) requires aggressive
Cefazolin, Cefotetan operative intervention, both to remove
Gastroduodenal Cefoxitin Fluoroquinolone contaminated material and infected tissue (e.g.
Ampicillin-sulbactam radical debridement or amputation) and to
Biliary tract with Ampicillin-sulbactam Fluoroquinolone + remove the initial cause of infection (e.g.
active infection Ticarcillin-clavulanate Clindamycin or bowel resection)
(cholecystitis) Piperacillin-tazobactam Metronidazole
Cefazolin+Metronidazole Gentamicin or
Colorectal , D. INFECTIONS OF SIGNIFICANCE IN SURGICAL PATIENTS
Ertapenem Fluoroquinolone
Obstructed small 1. Surgical Site Infection (SSI)
Ticarcillin-clavulanate plus Clindamycin
bowel Infections of the tissues, organs, or spaces exposed
Piperacillin-tazobactam or Metronidazole
Aminoglycoside + by surgeons during surgery
Head and neck Cefazolin
Clindamycin Development of SSI is related to three factors
Neurosurgery Cefazolin Vancomycin (Refer to Table 23):
Cefazolin o Patient factors
Orthopedics Vancomycin
Ceftriaxone o Local factors
Breast, Hernia Cefazolin Vancomycin o Microbial factors
Treatment || Prophylactic antibiotics reduce the
incidence of SSI during certain types of procedures
Empiric therapycomprises the use of an antimicrobial o Single dose of an antimicrobial agent
agent(s)when the risk of a surgical infection is high,
should be administered immediately before
based on the underlying disease process (e.g. ruptured
commencing surgery for class ID, II, III, and
appendicitis), or when significant contamination
IV types of wounds
during surgery has occurred (e.g. inadequate bowel
TOPNOTCH MEDICAL BOARD PREP SURGERY SUPPLEMENT HANDOUT Page 17 of 81
For inquiries visit www.topnotchboardprep.com.ph or email us at topnotchmedicalboardprep@gmail.com
TOPNOTCH MEDICAL BOARD PREP SURGERY SUPPLEMENT HANDOUT - Jules Lopez,MD-MBA,Teddy Carpio,MD-MBA
For inquiries visit www.topnotchboardprep.com.ph or email us at topnotchmedicalboardprep@gmail.com
Surgical management of the wound is also a critical Occurs due to contamination of the peritoneal
determinant of the propensity to develop an SSI cavity due to perforation or severe
o Class I and II woundsmay be closed primarily inflammation and infection of an intra-
o Class III and IV wounds areallowed to heal by abdominal organ
Secondary Examples: Appendicitis, perforation of any
secondary intention where superficial aspects
Microbial portion of the GI tract, or diverticulitis
of these wounds should be packed open only Peritonitis Treatment ||Effective therapy requires source
control to resect or repair the diseased organ,
Table 23. Risk factors for development of surgical site infections debridement of necrotic, infected tissue and
Patient Factors debris, and administration of antimicrobial
Older age agents directed against aerobes and anaerobe
Immunosuppression Develops by leakage from a GI anastomosis or
Obesity intra-abdominal abscess in patients in whom
Diabetes Mellitus standard therapy fails
Chronic inflammatory process Common in immunosuppressed patients
Malnutrition Microbes such as E. faecalis and faecium, S.
Peripheral vascular disease epidermidis, C. albicans, and P. aeruginosa can be
Anemia identified
Radiation Abscess is diagnosed via abdominal CT
Chronic skin disease Treatment || CT-guided percutaneous drainage
Carrier state (e.g. chronic staphylococcus carriage) for intra-abdominal abscess
Recent operation Surgical intervention is reserved for patients
Local Factors with multiple abscesses, those with abscesses in
Poor skin penetration proximity to vital structures such that
Tertiary
Contamination of instruments percutaneous drainage would be hazardous, and
(persistent)
Inadequate antibiotic prophylaxis those in whom an ongoing source of
Peritonitis or
Prolonged procedure contamination (e.g., enteric leak) is identified
Postoperative
Local tissue necrosis Peritonitis Necessity of antimicrobial agent therapy and
Hypoxia, hypothermia precise guidelines that dictate duration of
catheter drainage have NOT been established
Microbial Factors
Short course (3 to 7 days) of antibiotics that
Prolonged hospitalization (leading to nosocomial organisms)
covers for aerobic and anaerobic bacteria can be
Toxin secretion
given
Resistance to clearance (e.g. capsule formation)
Unfortunately, even with effective antimicrobial
agent therapy, this disease process is associated
Surgical site infections are classified into with mortality rates of more than 50%
incisionaland organ/space infections Drainage catheter is left in situ until the abscess
Incisional infections are further subclassified into cavity collapse, its output is less than 10-20 ml/d
superficial (limited to skin and subcutaneous tissue) with no evidence of an ongoing source of
and deep incisional categories contamination and the patient's clinical
condition has improved
o Treatment ||Effective therapy for incisional
SSIs consists of incision and drainage
without the addition of antibiotics
3. Postoperative Nosocomial Infections
o Antibiotic therapy is reserved for patients in
Include SSIs, UTIs, pneumonia, and bacteremia
whom evidence of significant cellulitis is
Most infections are related to prolonged use of
present, or who manifest concurrent SIRS
indwelling tubes and catheters for the purpose of
o Open wound often is allowed to heal by
urinary drainage, ventilation, and venous and arterial
secondary intention, with dressings being
access, respectively (Refer to Table 25).
changed twice a day
o Use of topical antibiotics and antiseptics to
Table 25. Postoperative nosocomial infections
further wound healing remains unproven
Postoperative nosocomial infections
o Vacuum-assisted closure is increasingly used Should be considered based on urinalysis with
in management of problem wounds and can be WBCs or bacteria, a positive test for leukocyte
applied to complex wounds in difficult esterase, or a combination of these elements
locations Diagnosis is established after more than 104
Treatment of organ/space infections is discussed in CFU/ml of microbes are identified by culture
Intra-Abdominal Infections section Postoperative techniques in symptomatic patients, or more
Urinary Tract than 105 CFU/ml in asymptomatic individuals
Infection (UTI) Treatment ||Single antibiotic therapy for 3 to
2.Intra-Abdominal Infections/Peritonitis
5 days
Microbial contamination of the peritoneal cavity Indwelling urinary catheters should be
Classified according to etiology (Refer to Table 24) removed as quickly as possible, typically
within 1 to 2 days, as long as the patients are
mobile
Table 24. Intra-abdominal infections Associated with prolonged mechanical
Intra-abdominal Infections ventilation and is frequently due to pathogens
Occurs when microbes invade the normally common in the nosocomial environment
sterile peritoneal cavity via hematogenous Diagnosis should be made using the presence
dissemination from a distant source of infection of a purulent sputum, elevated leukocyte
or direct inoculation count, fever, and new chest x-ray abnormality
More common among patients with ascites, and Presence of two of the clinical findings, plus
in those individuals who are undergoing Pneumonia chest x-ray findings, significantly increases the
peritoneal dialysis likelihood of ventilator-associated
Often monomicrobial and rarely require pneumonia
Primary surgical intervention Treatment ||Antibiotic therapy for 7 to 10
Microbial Diagnosis is established based on physical days
Peritonitis examination that reveals diffuse tenderness and Surgical patients should be weaned from
guardingwithout localized findings, absence of mechanical ventilation as soon as feasible,
pneumoperitoneum, presence of >100 WBCs/ml, based on oxygenation and inspiratory effort
and microbes with a single morphology on Gram’s
stain on fluid obtained via paracentesis
Treatment || Antibiotic therapy for 14 to 21 E. SHOCK
days and removal of indwelling devices (e.g., Failure to meet the metabolic needs of the cell and
peritoneal dialysis catheter or peritoneovenous
shunt)
the consequences that ensue

TOPNOTCH MEDICAL BOARD PREP SURGERY SUPPLEMENT HANDOUT Page 18 of 81


For inquiries visit www.topnotchboardprep.com.ph or email us at topnotchmedicalboardprep@gmail.com
TOPNOTCH MEDICAL BOARD PREP SURGERY SUPPLEMENT HANDOUT - Jules Lopez,MD-MBA,Teddy Carpio,MD-MBA
For inquiries visit www.topnotchboardprep.com.ph or email us at topnotchmedicalboardprep@gmail.com
Consists of inadequate tissue perfusion marked by If the patient's blood pressure has not
decreased delivery of required metabolic substrates responded, dopamine may be used
and inadequate removal of cellular waste products Results from failure of the heart as a pump,
(Refer to Figure 11) as in arrhythmias or acute myocardial
infarction (most common)
Initial cellular injury that occurs is reversible; however,
Hemodynamic criteria include sustained
injury will become irreversible if tissue perfusion is hypotension (i.e. SBP <90 mmHg for at least 30
prolonged or severe enough such that, at the cellular minutes), reduced cardiac index (<2.2
level, compensation is no longer possible L/min/m2), and elevated pulmonary artery
wedge pressure (>15 mmHg)
Figure 11. Pathways leading to decreased tissue perfusion and shock Treatment ||Ensure adequate airway is
Cardiogenic present and ventilation is sufficient
Treatment of cardiac dysfunction includes
maintenance of adequate oxygenation to
ensure adequate myocardial O2 delivery and
judicious fluid administration to avoid fluid
overload and development of cardiogenic
pulmonary edema
Significant dysrhythmias and heart block must
be treated with antiarrhythmic drugs, pacing,
or cardioversion

Form of cardiogenic shock that results from


mechanical impediment to circulation
leading to depressed cardiac output rather
than primary cardiac failure
Causes include cardiac tamponade, pulmonary
Obstructive
embolism, tension pneumothorax, IVC
obstruction (DVT, gravid uterus), increased
intrathoracic pressure (neoplasm)
Treatment || Dependent on the etiology of the
obstructive shock
Soft tissue and bony injury lead to the
activation of inflammatory cells and the release
of circulating factors that modulate the
Clinical manifestations of several physiologic responses immune response
are most often what lead practitioners to the diagnosis These effects of tissue injury are combined
of shock as well as guide the management of patients with the effects of hemorrhage, creating a more
Shock is classified into six types (Refer to Table 26) complex and amplified deviation from
homeostasis.
Table 26. Types of Shock Traumatic
Postoperative nosocomial infections Treatment || Correction of the individual
Most common type elements to diminish the cascade of
Results from loss of circulating blood proinflammatory activation, and includes
volumedue to loss of whole blood prompt control of hemorrhage, adequate
(hemorrhagic shock), plasma, interstitial fluid volume resuscitation to correct O2 debt,
(bowel obstruction) debridement of nonviable tissue, stabilization
Clinical and physiologic response is classified of bony injuries, and appropriate treatment of
according to the magnitude of volume loss soft tissue injuries
(Refer to Table 27)
Hypovolemic Treatment || Instituted with diagnostic
evaluation to identify a bleeding source Table 27. Signs and symptoms of advancing stages of hypovolemic shock
Appropriate priorities are secure the airway,
control source of blood loss, and IV volume Class I Class II Class III Class IV
resuscitation Blood loss
Up to 750 750-1,500 1,500-2,000 >2,000
Patients who fail to respond to initial (ml)
resuscitative efforts should be assumed to have Blood loss
ongoing active hemorrhage and require (%blood Up to 15% 15-30% 30-40% >40%
prompt operative intervention volume)
Results from decreased resistance within Pulse rate <100 >100 >120 >140
capacitance vessels Blood
Normal Normal Decreased Decreased
Evaluationbegins with an assessment of the pressure
adequacy of airway and ventilation Pulse Normal or
Decreased Decreased Decreased
Treatment ||Fluid resuscitation and pressure increased
restoration of circulatory volume Respiratory
14-20 20-30 30-40 >35
Vasogenic Empiric antibiotics must be chosen carefully rate
(Septic) (gram-negative rods, gram-positive cocci, and Urine
anaerobes) output >30 20-30 5-15 Negligible
However, IV antibiotics without source control (ml/h)
will be insufficient to adequately treat patients Confused
CNS/mental Slightly Mildly Anxious and
with infected fluid collections, infected foreign and
status anxious anxious confused
bodies, and devitalized tissue lethargic
Vasopressors may be necessary as well
Form of vasogenic shock in which spinal cord Ultimate goal in the treatment of shock is restoration of
injury or spinal anesthesia causes adequate organ perfusion and tissue oxygenation
vasodilation due to acute loss of Endpoints in resuscitation can be divided into systemic
sympathetic vascular tone
or global parameters ,tissue-specific parameters, and
Treatment ||After airway is secured and
ventilation is adequate, fluid resuscitation and
cellular parameters (Refer to Table 28)
Neurogenic Global endpoints include vital signs, cardiac output,
restoration of intravascular volume often will
improve perfusion pulmonary artery wedge pressure, O2 delivery and
Administration of vasoconstrictors will consumption, lactate, and base deficit
improve peripheral vascular tone, decrease
vascular capacitance, and increase venous
return

TOPNOTCH MEDICAL BOARD PREP SURGERY SUPPLEMENT HANDOUT Page 19 of 81


For inquiries visit www.topnotchboardprep.com.ph or email us at topnotchmedicalboardprep@gmail.com
TOPNOTCH MEDICAL BOARD PREP SURGERY SUPPLEMENT HANDOUT - Jules Lopez,MD-MBA,Teddy Carpio,MD-MBA
For inquiries visit www.topnotchboardprep.com.ph or email us at topnotchmedicalboardprep@gmail.com
Table 28. Endpoints in resuscitation preventing the typical reflex tachycardia that occurs with
Systemic/Global hypovolemia.
Lactate The classic description of neurogenic shock consist of
Base deficit decreased blood pressure associated with
Cardiac output
bradycardia (absence of reflex tachycardia due to
Oxygen delivery and consumption
disrupted sympathetic discharge), warm extremities
Tissue Specific
(loss of peripheral vasoconstriction), motor and sensory
Gastric tonometry
Tissue pH, Oxygen, Carbon dioxide levels deficits indicative of a spinal cord injury, and
Near infrared spectroscopy radiographic evidence of a vertebral column fracture.
Cellular
Membrane potential
Adenosine triphosphate TRAUMA

A. General Principle
REVIEW QUESTIONS a B. Primary Survey
C. Resuscitation
1. Which of the following is the most effective dosing of D. Secondary Survey
antibiotics in a patient undergoing elective colon E. Diagnostic Evaluation
resection? F. Definitive Care
a. A single dose given within 30 min prior to skin
incision
b. A single doe given at the time of skin incision A. GENERAL PRINCIPLE
c. A single preoperative dose + 24 hours of
Trauma or injury is a cellular disruption caused by
postoperative antibiotics an exchange with environmental energy that is
d. A single preoperative dose + 48 hours of beyond the body’s resilience
postoperative antibiotics
Most common cause of death for all individuals
between the ages of 1 and 44 years
Answer: A
Third most common cause of death regardless of age
Prophylaxis is the administration of an antimicrobial
Most common cause of years of productive life lost
agent(s) before and during the operative procedure to
reduce the number of microbes that enter the tissue or Initial management of seriously injured patients
body cavity. Only a single dose of antibiotic is required, according to the Advanced Trauma Life Support (ATLS)
and only for certain types of surgical procedures. There consists of the following:
is no evidence that administration of postoperative o Primary survey
doses provides additional benefit. o Concurrent resuscitation
o Secondary survey
2. What percentage of the blood volume is normally in the o Diagnostic evaluation
splanchnic circulation? o Definitive care
a. 10%
b. 20% B. PRIMARY SURVEY
c. 30% Goal is to identify and treat conditions that constitute an
d. 40% immediate threat to life (Refer to Table 29)
Assessment of the “ABCDE” (Airway with cervical
Answer: B spine protection, Breathing, Circulation, Disability,
Most alterations in cardiac output in the normal heart and Exposure)
are related to changes in preload. Increases in
Table 29. Life-threatening injuries identified during the primary survey
sympathetic tone have a minor effect on skeletal muscle
Airway
beds but produce a dramatic reduction in splanchnic
Airway obstruction
blood volume, which holds 20% of the blood volume.
Airway injury
Breathing
3. Which of the following best describes the hemodynamic Tension pneumothorax
response to neurogenic shock? Open pneumothorax
a. Increased cardiac index, unchanged venous Flail chest with underlying pulmonary contusion
capacitance Circulation
b. Increased cardiac index, decreased venous Massive hemothorax or hemoperitoneum
capacitance
c. Variable change in cardiac index (can increase Hemorrhagic shock Mechanically unstable pelvis fracture
or decrease), increased venous capacitance Extremity losses
d. Variable change in cardiac index (can increase Cardiogenic shock: Cardiac tamponade
or decrease), decreased venous capacitance Neurogenic shock: Cervical spine injury
Disability
Answer: A Intracranial hemorrhage/mass lesion
Choice B and D are most commonly associated with
septic shock. Choice C, on the other hand, is most likely 1. Airway management with cervical spine protection
seen in cardiogenic shock. Ensuring a patent airway is the first priority in the
primary survey
4. An unconscious patient with a systolic BP of 80 and a HR Efforts to restore cardiovascular integrity will be futile
of 80 most likely has? unless the oxygen content of the blood is adequate
a. Cardiogenic shock All patients with blunt trauma require cervical spine
b. Hemorrhagic shock immobilization (hard collar or placing sandbags on
c. Neurogenic shock both sides of the head with the patient’s forehead taped
d. Obstructive shock across bags to the backboard) until injury is excluded
Patients who are conscious, do not show tachypnea, and
Answer: C have a normal voice do not require early attention to the
Sympathetic input to the heart, which normally increases airway EXCEPT the following:
heart rate and cardiac contractility, and input to the o Patients with penetrating injuries to the neck
adrenal medulla, which increases catecholamine release, and an expanding hematoma
may also be disrupted (with spinal cord injury),

TOPNOTCH MEDICAL BOARD PREP SURGERY SUPPLEMENT HANDOUT Page 20 of 81


For inquiries visit www.topnotchboardprep.com.ph or email us at topnotchmedicalboardprep@gmail.com
TOPNOTCH MEDICAL BOARD PREP SURGERY SUPPLEMENT HANDOUT - Jules Lopez,MD-MBA,Teddy Carpio,MD-MBA
For inquiries visit www.topnotchboardprep.com.ph or email us at topnotchmedicalboardprep@gmail.com
o Evidence of chemical or thermal injury to the
mouth, nares, or hypopharynx
o Extensive subcutaneous air in the neck Figure 12. Tension Pneumothorax (inset) with Needle Thoracostomy
o Complex maxillofacial trauma
o Airway bleeding
Elective intubation should be performed on the cases
above before evidence of airway compromise
Altered mental status is the most common
indication for intubation
Options for endotracheal intubation include
nasotracheal, orotracheal, or surgical routes
o Nasotracheal: Only done in patients, who are
breathing spontaneously, requiring emergent
airway support in whom chemical paralysis
cannot be used
o Orotracheal: most common technique used
to establish a definitive airway
o Surgical (cricothyroidotomy): Done in
2nd ICS
patients in whom attempts at intubation have
failed or who are precluded from intubation
due to extensive facial injuries
o Surgical (emergent tracheostomy): Indicated Figure 13. Closed Tube Thoracostomy (CTT).A. Performed in the MAL at
in patients with laryngotracheal separation or the 4th-5th ICS to avoid iatrogenic injury to the liver or spleen. B. Heavy
laryngeal fractures, in whom scissors are used to cut through the intercostal muscle into the pleural
cricothyroidotomy may cause further damage space done on top of the rib to avoid injury to the intercostal bundle
or result in complete loss of airway located just beneath the rib. C. Incision is digitally explored to confirm
intrathoracic location and identify pleural adhesions. D. A 36F chest tube
is directed superiorly and posteriorly with the aid of a large clamp.
2. Breathing and Ventilation
Once a secure airway is obtained, adequate oxygenation
and ventilation must be assured
All injured patients should receive supplemental oxygen
and be monitored by pulse oximetry
The following conditions constitute an immediate threat
to life due to inadequate ventilation (Refer to Table 30)

Table 30. Life-threatening injury identified due to inadequate ventilation


Inadequate Ventilation
Diagnosis is implied by respiratory distress
and hypotension in combination with any of
the following physical signs in patients with
chest trauma: tracheal deviation away from
the affected side, lack of or decreased breath
sounds on the affected side, and
subcutaneous emphysema on the affected
Tension
side(Refer to Figure 10inset)
Pneumothorax
Treatment ||Needle thoracostomy
decompressionin the 2nd ICS in the MCL
may be indicated in the acute setting(Refer
to Figure 10)
Closed tube thoracostomy should be
performed immediately before a chest
radiograph is obtained
Occurs with full thickness loss of the chest
wall, permitting free communication
between the pleural space and the
atmosphere
Compromises ventilation due to
equilibration of atmospheric and pleural
Figure 14. Mechanism of a Flail Chest.Paradoxical movement of the flail
pressures, which prevents lung inflation
Open chest during inspiration and expiration.
and alveolar ventilation, and results in
Pneumothorax
hypoxia and hypercarbia
(Sucking chest
Complete occlusion of the chest wall defect
wound)
WITHOUT a tube thoracostomy may
convert an open pneumothorax to a
tension pneumothorax
Treatment ||Definitive treatment is closure
of the chest wall defect and closed tube
thoracostomy remote from the wound
(Refer to Figure 11)
Occurs when 3 or more contiguous ribs
are fractured in at least 2 locations
(Refer to Figure 12)
Paradoxical movement of this free floating
segment of chest wall may be evident in
Flail chest with patients with spontaneous ventilation, due
underlying to the negative intrapleural pressure of
pulmonary inspiration
contusion Associated pulmonary contusion is
typically the source of postinjury 3. Circulation with hemorrhage control
pulmonary dysfunction (Decreased
compliance and increased shunt fraction) Initial approximation of the patient’s cardiovascular
Treatment ||May require presumptive status can be obtained by palpating peripheral pulses
intubation and mechanical ventilation o Carotid pulse: 60 mmHg systolic BP
TOPNOTCH MEDICAL BOARD PREP SURGERY SUPPLEMENT HANDOUT Page 21 of 81
For inquiries visit www.topnotchboardprep.com.ph or email us at topnotchmedicalboardprep@gmail.com
TOPNOTCH MEDICAL BOARD PREP SURGERY SUPPLEMENT HANDOUT - Jules Lopez,MD-MBA,Teddy Carpio,MD-MBA
For inquiries visit www.topnotchboardprep.com.ph or email us at topnotchmedicalboardprep@gmail.com
o Femoral pulse: 70 mmHg
o Radial pulse: 80 mmHg to be palpable Figure 16. Pericardiocentesis. Access to the pericardium is obtained
Any hypotensive episode (SBP <90 mmHg) is assumed through a subxiphoid approach, with the needle angled 45 degrees up
from the chest wall and toward the left shoulder.
to be caused by hemorrhage until proven otherwise
IV access for fluid resuscitation is obtained with 2
peripheral catheters, 16-gauge or larger in adults
In patients under 6 years old, an intraosseus needle can
be placed in the proximal tibia (preferred) or distal
femur of an unfractured extremity
External control of hemorrhage should be achieved
promptly while circulating volume is restored
The following conditions constitute an immediate threat
to life due to inadequate circulation (Refer to Table 31)

Table 31. Life-threatening injury identified due to inadequate circulation


Inadequate Circulation
Defined as >1,500 ml of blood or, in the
pediatrics, 1/3 of the patient’s blood
volume in the pleural space
After a blunt trauma, hemothorax is usually
due to multiple rib fractures with severed
Massive
intercostal arteries, but occasionally
Hemothorax
bleeding isfrom lacerated lung parenchyma
After a penetrating trauma, a systemic or
pulmonary hilar vessel injury should be
presumed Table 32. Emergency Department Thoracotomy (EDT) Indications and
Treatment || Operative intervention Contraindications. CPR = Cardiopulmonary resuscitation
Occurs most commonly after penetrating Indications
thoracic injuries, although occasionally blunt Patients sustaining witnessed penetrating
rupture of the heart, particularly the atrial Salvegeable trauma with <15 min of prehospital CPR
appendage, is seen postinjury cardiac Patients sustaining witnessed blunt trauma
<100 ml of pericardial blood may cause arrest with <5 min of
pericardial tamponade prehospital CPR
(Refer to Figure 13 right) Persistent severe Cardiac tamponade
Beck’s triad (dilated neck veins, muffled postinjury Hemorrhage (intrathoracic, intra-
heart tones, and a decline in arterial pressure) hypotension abdominal, extremity, cervical)
is NOT often observed (SBP 60 mmHg) Air embolism
Diagnosis is best achieved by ultrasound of
Contraindications
the pericardium
Penetrating trauma: CPR >15 min and no signs of life (papillary
(Refer to Figure 13 left)
response, respiratory effort, motor activity)
Early in the course of tamponade, blood
Blunt trauma: CPR >5 min and no signs of life or asystole
pressure and cardiac output will transiently
improve with fluid administration
Cardiac Figure 17. Emergency department thoracotomy (EDT) is performed
Treatment ||Pericardiocentesis is successful
Tamponade through the 5th ICS using the anterolateral approach. Pericardium is
in decompressing tamponade in
opened anterior to the phrenic nerve, and the heart is rotated out for
approximately 80% of cases (Refer to
repair
Figure 14)
Removing as little as 15 to 20 ml of blood will
often temporarily stabilize the patient’s
hemodynamic status, prevent subendocardial
ischemia, and associated lethal arrhythmias,
and allow transport to the OR for sternotomy
Patients with a SBP <70 mmHg warrant
emergency department thoracotomy(EDT)
with opening of the pericardium to address
the injury (Refer to Table 31)
EDT is best accomplished using aleft
anterolateral thoracotomy, with the
incision started to the right of the sternum
(Refer to Figure 15)

4. Disability and Exposure


Glasgow Coma Scale (GCS) score should be determined
for all injured patients
Scores of 13 to 15 indicate mild head injury, 9 to 12
moderate injury, and <9 severe injury
Abnormal mental status should prompt an immediate ANATOMY a
re-evaluation of the ABCs and consideration of central
nervous system injury Closed Tube Thoracostomy (CTT)is done on the
superior border of the lower ribon the 4th-5th ICS MAL
Figure 15. Cardiac o Directed superiorly for air drainage
Tamponade with ultrasound findings (*) on the left.
o Directed inferiorly for fluid drainage
o Tube passes through the following: Skin
Superficial fascia Serratus anterior
External intercostals Internal intercostals
Innermost intercostals Endothoracic fascia
Parietal pleura

TOPNOTCH MEDICAL BOARD PREP SURGERY SUPPLEMENT HANDOUT Page 22 of 81


For inquiries visit www.topnotchboardprep.com.ph or email us at topnotchmedicalboardprep@gmail.com
TOPNOTCH MEDICAL BOARD PREP SURGERY SUPPLEMENT HANDOUT - Jules Lopez,MD-MBA,Teddy Carpio,MD-MBA
For inquiries visit www.topnotchboardprep.com.ph or email us at topnotchmedicalboardprep@gmail.com

QUICK REVIEW a high-riding prostate, which is particularly critical in


patients with suspected spinal cord injury, pelvic
Primary survey consists of the assessment of the fracture, or transpelvic gunshot wounds
“ABCDE” (Airway with cervical spine protection, Vaginal examination with a speculum also should be
Breathing, Circulation, Disability, and Exposure) performed in women with pelvic fractures to exclude an
Ensuring a patent airway is the first priority in the open fracture
primary survey
Altered mental status is the most common indication E. DIAGNOSTIC EVALUATION
for intubation Selective radiography and laboratory tests are done
Massive hemothorax is defined as >1,500 ml of blood early in the evaluation after the primary survey
or, in the pediatrics, 1/3 of the patient’s blood volume For patients with severe blunt trauma, lateral cervical
in the pleural space spine, chest, and pelvic radiographs should be
Tension pneumothorax is the most common cause of obtained, often termed the big three
cardiogenic shock in trauma patients For patients with truncal gunshot wounds,
anteroposterior and lateral radiographs of the chest
and abdomen are warranted
C. RESUSCITATION In critically injured patients, blood samples for a routine
Quantity of acute blood loss correlates with trauma panel (type and cross-match, complete blood
physiologicabnormalities (Refer to Table 26) count, blood chemistries, coagulation studies,
o Tachycardia is often the earliest sign of lactate level, and arterial blood gas analysis) should
ongoing blood loss but watch out for relative be sent to the laboratory
tachycardia (HR<90 in patients with a resting For less severely injured patients only a complete blood
pulse rate in the 50s) count and urinalysis may be required
o Bradycardia, an ominous sign, occurs with
severe blood loss, often heralding impending F. DEFINITIVE CARE
cardiovascular collapse All injured patients undergoing an operation should
o Hypotension is NOT a reliable early sign of receive preoperative antibiotics
hypovolemia, because blood volume must Extended postoperative antibiotic therapy is
decrease by >30% before hypotension occurs administered only for open fractures or significant intra-
Goal is to re-establish tissue perfusion abdominal contamination
o Urine output is a quantitative, reliable Tetanus prophylaxis is administered to all patients
indicator of organ perfusion Trauma patients particularly (a) those with multiple
o Adequate urine output is 0.5 ml/kg/hr in fractures of the pelvis and lower extremities, (b) those
an adult, 1 ml/kg/hr in a child, and 2 with coma or spinal cord injury, and (c) those requiring
ml/kg/hr in an infant <1 year of age ligation of large veins in the abdomen and lower
o Fluid resuscitation begins with a 2L (adult) or extremitiesare at risk for venous thromboembolism
20 ml/kg (child) IV bolus of isotonic and its associated complications
crystalloid, typically Ringer’s lactate o Low molecular weight heparin is initiated as
o For persistent hypotension, this is repeated soon as bleeding has been controlled and
once in adult and twice in a child before RBCs there is no intracranial pathology
are administered o In high-risk patients, removable inferior
Based on the initial response to fluid resuscitation, vena caval filters should be considered if
hypovolemic injured patients can be separated into there are contraindications to administration
three broad categories: responders, transient responders, of low molecular weight heparin
and nonresponders o Pulsatile compression stockings or
o Responders:Individuals who are stable or sequential compression devices are used
have a good response to the initial fluid routinely unless there is a fracture
therapy as evidenced by normalization of vital Another prophylactic measure is thermal protectionby
signs, mental status, and urine output are maintaining a comfortable ambient temperature,
unlikely to have significant ongoing covering stabilized patients with warm blankets, and
hemorrhage, and further diagnostic evaluation administering warmed IV fluids and blood products.
for occult injuries can proceed in an orderly o Hemorrhagic shock impairs perfusion and
fashion (Secondary survey) metabolic activity throughout the body, with
o Transient Responders: Those who respond resultant decrease in heat production and
initially to volume loading by an increase in body temperature
blood pressure only to then hemodynamically o Hypothermia causes coagulopathy and
deteriorate once more myocardial irritability
o Nonresponders: These patients have PRBC transfusion should occur once the patient's
persistent hypotension despite aggressive hemoglobin level is <7 g/dl, in the acute phase of
resuscitation resuscitation the endpoint is 10 g/dl
Patients with ongoing hemodynamic instability, FFP is transfused to keep theINR <1.5 and PTT <45 sec
whether nonresponders or transient responders, require Target of 100,000/l is the target platelet count with
systematic evaluation and prompt intervention massive transfusion

D. SECONDARY SURVEY 1. Neck


Once the immediate threats to life have been addressed, Divided into three distinct zones that is important in the
a thorough history is obtained and the patient is management of neck injuries (Refer to Figure 18 )
examined in a systematic fashion
Patient (or surrogate) should be queried to obtain an
“AMPLE” (Allergies, Medications, Past illnesses or
Pregnancy, Last meal, and Events related to the
injury)
Physical examination should be head to toewith special
attention to the patient's back, axilla, and perineum,
because injuries here are easily overlooked
All potentially seriously injured patients should
undergo digital rectal examination to evaluate for
sphincter tone, presence of blood, rectal perforation, or a
TOPNOTCH MEDICAL BOARD PREP SURGERY SUPPLEMENT HANDOUT Page 23 of 81
For inquiries visit www.topnotchboardprep.com.ph or email us at topnotchmedicalboardprep@gmail.com
TOPNOTCH MEDICAL BOARD PREP SURGERY SUPPLEMENT HANDOUT - Jules Lopez,MD-MBA,Teddy Carpio,MD-MBA
For inquiries visit www.topnotchboardprep.com.ph or email us at topnotchmedicalboardprep@gmail.com
Figure 18. For the purpose of evaluating penetrating injuries, the neck is Figure 20. Algorithm for the evaluation of penetrating abdominal
divided into three zones. Zone I is up to the level of the cricoid and is also injuries. AASW = anterior abdominal stab wound (from costal margin to
known as the thoracic outlet.Zone II is located between the cricoid inguinal ligament and bilateral MAL); CT = computed tomography; DPL =
cartilage and the angle of the mandible. Zone III is above the angle of the diagnostic peritoneal lavage; GSW = gunshot wound; LWE = local wound
mandible. exploration; RUQ = right upper quadrant; SW = stab wound.

Imaging options include CT scan or five plain Anterior abdominal stab wounds (AASW) should be
radiograph views of the cervical spine: lateral view explored under local anesthesia in the ED to determine
with visualization of C7-T1, anteroposterior view, if the fascia has been violated
transoral odontoid views, and bilateral oblique views o Injuries that do not penetrate the peritoneal
Identification of penetrating injuries to the neck with cavity do not require further evaluation, and
exsanguination, expanding hematomas, and airway the patient is discharged from the ED
obstruction is a priority during the primary survey o Patients with fascial penetration must be
Management algorithm for penetrating neck injury further evaluated for intra-abdominal injury,
patients is based on the presenting symptoms and because there is up to a 50% chance of
anatomic location of injury (Refer to Figure 19) requiring laparotomy
All blunt trauma patients should be assumed to have o Debate remains over whether the optimal
cervical spine injuries until proven otherwise diagnostic approach is serial examination,
diagnostic peritoneal lavage
Figure 19. Algorithm for the selective management of penetrating neck (Refer to Figure 20), or CT scanning
injuries. CT = computed tomography; CTA = computed tomographic o Values representing positive findings for
angiography; GSW = gunshot wound; IR Embo = interventional radiology diagnostic peritoneal lavage are summarized
embolization in Table 33

Figure 21. Diagnostic peritoneal lavage (DPL) is performed through an


infraumbilical incision unless the patient has a pelvic fracture or is
pregnant. Linea alba is sharply incised, and the catheter is directed into
the pelvis. Abdominal contents (diagnostic peritoneal aspiration) is
considered positive if >10 ml of blood is aspirated. If <10 ml is obtained, a
liter of NSS is instilled. Effluent is withdrawn via siphoning and sent to
the laboratory for analysis.

2. Abdomen
Diagnostic approach differs for penetrating trauma
(i.e. gun shot/stab wound) and blunt abdominal trauma
Management algorithm for penetrating abdominal
injury patients is primarily based on the anatomic
location of injury (Refer to Figure 18)
As a rule, minimal evaluation is required before
laparotomy for abdominal gunshot or shotgun wounds
because over 90% of patients have significant internal
injuries EXCEPT those isolated in the liver by CT scan;
in hemodynamically stable patients where
nonoperative observation may be considered
Abdominal stab wounds are less likely to injure intra-
abdominal organs and thus, diagnostic evaluation can
be afforded

TOPNOTCH MEDICAL BOARD PREP SURGERY SUPPLEMENT HANDOUT Page 24 of 81


For inquiries visit www.topnotchboardprep.com.ph or email us at topnotchmedicalboardprep@gmail.com
TOPNOTCH MEDICAL BOARD PREP SURGERY SUPPLEMENT HANDOUT - Jules Lopez,MD-MBA,Teddy Carpio,MD-MBA
For inquiries visit www.topnotchboardprep.com.ph or email us at topnotchmedicalboardprep@gmail.com
Table 33. Criteria for positive finding on diagnostic peritoneal lavage.
Between 1,000-10,000/ml, do laparoscopy/thoracoscopy REVIEW QUESTIONS a
Anterior Anterior Abdominal Thoracoabdominal
Abdominal Stab Wound Stab Wound
1. Which of the following trauma patients with airway
Red blood cell
(RBC) count
>100,000/ml >10,000/ml compromise and failed endotracheal intubation should
White blood cell undergo emergency tracheostomy (rather than a
>500/ml cricothyroidotomy)?
(WBC) count
Amylase a. 84 y/o male with blunt trauma to the neck
>19 IU/l b. 65 y/o female with a stab wound to the
level
Alkaline submandibular region
>2 IU/l
phosphatase level c. 16 y/o male with a gun shot wound to the neck
Bilirubin d. 6 y/o female with a crush injury to the face
>0.01 mg/dl
level
Answer: D
Blunt abdominal trauma initially is evaluated by FAST In patients under the age of 8, cricothyroidotomy is
(Refer to Figure 22) exam in major trauma centers contraindicated due to the risk of subglottic stenosis, and
FAST is not 100% sensitive so diagnostic peritoneal tracheostomy should be performed.
aspiration is still advocated in hemodynamically
unstable patients without a defined source of blood loss 2. A patient presents with stable vital signs and respiratory
to rule out abdominal hemorrhage distress after a stab wound to the chest. Chest tubes are
placed and an air leak is noted. The patient is electively
Figure 22. FAST is used to identify free intraperitoneal fluid in intubated. The patient arrests after positive pressure
(1) subxiphoid/pericardium, (2) Morison's pouch/hepatorenal recess, ventilation is started. What is the most likely diagnosis?
(3) left upper quadrant/perisplenic, and (4) pelvis. Although this method a. Unrecognized hemorrhage in the abdomen
is sensitive for detecting intraperitoneal fluid of >250 ml, it does not b. Tension pneumothorax
reliably determine the source of bleeding nor grade solid organ injuries.
c. Pericardial tamponade
d. Air embolism

Answer: D
Air emboli can occur after blunt or penetrating trauma,
when air from an injured bronchus enters an adjacent
injured pulmonary vein and returns air to the left heart.
Air accumulation in the left ventricle impedes diastolic
filling, and during systole air is pumped into the
coronary arteries, disrupting coronary perfusion.
Patient should be placed in Trendelenburg’s position to
trap the air in the apex of the left ventricle. Emergency
thoracotomy is followed by cross clamping (left picture)
of the pulmonary hilum on the side of the injury to
prevent further introduction of air. Air is aspirated from
the apex of the left ventricle and the aortic root with an
18-g needle and 50-ml syringe (right picture). Vigorous
massage is used to force air bubble through the coronary
arteries. If unsuccessful, a tuberculin syringe may be
used to aspirate air from the right coronary artery. Once
Patients with fluid on FAST examination, considered a circulation is restored, patient should be kept in
"positive FAST," who do not have immediate indications Trendelenburg’s with the pulmonary hilum clamped
for laparotomy and are hemodynamically stable until pulmonary venous injury is controlled operatively.
undergo CT scanning to quantify their injuries
Management algorithm for blunt abdominal injury
patients is shown in Figure 23

Figure 23. Algorithm for the initial evaluation of a patient with suspected
blunt abdominal trauma. CT = computed tomography; DPA = diagnostic
peritoneal aspiration; FAST = focused abdominal sonography for
trauma/focused assessment with sonography for trauma; Hct=hematocrit

3. Which of the following is the expected blood loss in a


patient with 6 rib fractures?
a. 240 ml
QUICK REVIEW a b. 480 ml
c. 750 ml
Tachycardia is the earliest sign of ongoing blood loss d. 1500 ml
Adequate urine output is 0.5 ml/kg/hr in an adult, 1
ml/kg/hr in a child, and 2 ml/kg/hr in an infant <1 year Answer: C
of age For each rib fracture, there is ~100-200 ml of blood
Secondary survey consists of “AMPLE” (Allergies, loss; for tibial fractures, 300-500 ml; for femur fractures,
Medications, Past illnesses or Pregnancy, Last meal, 800-1000 ml; and for pelvic fractures, >1000 ml.
and Events related to the injury) Although no single injury may appear to cause a patient’s
hemodynamic instability, the sum of the injuries may
result in life-threatening blood loss

TOPNOTCH MEDICAL BOARD PREP SURGERY SUPPLEMENT HANDOUT Page 25 of 81


For inquiries visit www.topnotchboardprep.com.ph or email us at topnotchmedicalboardprep@gmail.com
TOPNOTCH MEDICAL BOARD PREP SURGERY SUPPLEMENT HANDOUT - Jules Lopez,MD-MBA,Teddy Carpio,MD-MBA
For inquiries visit www.topnotchboardprep.com.ph or email us at topnotchmedicalboardprep@gmail.com

BURNS ANATOMY a
LAYERS OF THE SKIN
A. Classification of Burns Epidermis is the outermost layer of the integument
B. Burn Depth composed of stratified squamous epithelial layer that is
C. Initial Evaluation of Burns devoid of blood vessels, consisting of 4-5 layers:
D. Management of Burns o Stratum Corneum is a superficial stratum later
E. Inhalational Injury consisting of flat, anucleated and keratinized
cells filled with keratin filaments embedded in
a dense matrix of proteins
A. CLASSIFICATION OF BURNS o Stratum Lucidum is only found in regions of
1. Thermal thick stratum corneum of palms and soles;
Flame: Most common cause for hospital admission; not found in thin skin
highest mortality (due to association with inhalational o Stratum Granulosum is polygonal cells with
injury and/or Carbon Monoxide (CO) poisoning) basophilic keratohyalin granules; 1 layer in
Contact thin skin while multiple layers in thin skin
Scald o Stratum Spinosum is a multilaminar layer of
cuboidal-like cells that are bound together by
2. Electrical means of numerous desmosomal junctions
(tonofibrils) and they produce keratin
Potential for cardiac arrhythmias; do baseline ECG i
o Stratum Basale/germinativum is a
Compartment syndromes with concurrent
mitotically active, single layer of columnar or
rhabdomyolysis is more common in high-voltage
cuboidal cells attached to the dermis via
injuries; check for neurologic or vascular compromise
hemidesmosome
Long-term neurologic and visual symptoms are also
o Mnemonics: “Californians Like Girls in String
common and thus, neurologic and ophthalmologic
Bikinis”
consultation should be done
Dermis is the connective tissue layer below the
epidermis and its basement membrane, consisting of 2
3. Chemical
layers:
Less common but usually severe
o Papillary layer appears loose that fills the
Offending agents can be systematically absorbed; may hollows at the deep surface of the epidermis
cause specific metabolic derangements
with frequent capillaries
Careful removal of toxic substance from patient and o Reticular layer appears denser and contains
irrigation of the affected area with water (~30 mins) fewer cells
EXCEPT in cases of concrete powder or powdered forms
Hypodermis is a layer of loose vascular connective
of lye, which should be swept from the patient instead
tissue infiltrated by adipocytes
to avoid activation of AlOH with water

B. BURN DEPTH
Burn wounds are commonly stratified according to
depth as superficial, partial thickness, full thickness, and
fourth degree burns, which affect underlying soft tissue
They are also described according to zone of tissue
injury (Refer to Table 34)

1. Superficial (First degree burn)


Painful but DO NOT blister
2. Partial thickness (Second degree burn)
Extremely painful with weeping and blisters
Classified as either superficial or deep depending on the
depth of dermal involvement
o Superficial: Heals with expectant management
o Deep: Requires excision and skin grafting

3. Full thickness (Third degree burn)


Painless, hard, and non-blanching

4. Fourth degree burn


Affects underlying soft tissue C. INITIAL EVALUATION OF BURNS
1. Airway management
Table 34. Jackson’s three zones of tissue injury following burn With direct thermal injury to the upper airway and/or
Jackson’s three zones of tissue injury following burn smoke inhalation (perioral burns, signed nasal hairs),
Most severely burned area (typically the rapid and severe airway edema is a potentially lethal
Zone of center of the wound)
Coagulation
threat
Affected tissue is coagulated and sometimes
necrotic, and will need excision and grafting Anticipating the need for intubation and establishing an
Between the first and third zones with local early airway is critical
response of vasoconstriction and ischemia Signs of impending respiratory compromise: hoarse
It has marginal perfusion and questionable voice, wheezing, or stridor
viability
Zone of Stasis
Resuscitation and wound care may help 2. Evaluation of other injuries
prevent conversion to a deeper burn Burn patients should be first considered
Burn wounds evolve over 48-72 hours after traumapatients (especially when details of the injury
injury
are unclear), as such, a primary survey should be
Outermost area, usually heals with minimal or
Zone of
no scarring
conducted
Hyperemia An early and comprehensive secondary survey must
There is increased blood flow in this area
also be performed in all burn patients
Urgent radiology studies (i.e. CXR) should be performed
in the ER, but non urgent skeletal evaluation (i.e.

TOPNOTCH MEDICAL BOARD PREP SURGERY SUPPLEMENT HANDOUT Page 26 of 81


For inquiries visit www.topnotchboardprep.com.ph or email us at topnotchmedicalboardprep@gmail.com
TOPNOTCH MEDICAL BOARD PREP SURGERY SUPPLEMENT HANDOUT - Jules Lopez,MD-MBA,Teddy Carpio,MD-MBA
For inquiries visit www.topnotchboardprep.com.ph or email us at topnotchmedicalboardprep@gmail.com
extremity X-rays) can be done later to avoid Therefore, if a patient receives large fluid bolus in a
hypothermia and delays in burn resuscitation prehospital setting or ER, that fluid has likely
leaked into the interstitium and the patient will still
3. Estimation of burn size require ongoing burn resuscitation
Most burn resuscitation formulas estimate fluid Several formulas are available to compute for the
requirements using the burn size as %Total Body total fluid requirement but among the most widely
Surface Area (TBSA) used one is the Parkland formula(Refer to Table
“Rule of nines” is a crude but quick and effective 36 )
method of estimating burn size (Refer to Figure 24 )
Thorough cleaning of soot and debris is mandatory to Table 36. Parkland formula
avoid confusing areas of soiling with burns Parkland formula
Superficial (first degree) burns SHOULD NOT be Total fluid requirement* = 4 mg/kg per %TBSA burn
included when calculating the %TBSA ½ volume during first 8 hours ½ during next 16
post-injury hours post-injury
*Use of lactated ringer’s solution
Figure 24. Rule of nines to estimate burn size

Continuation of fluid volumes should depend on


the time since injury, UO, and MAP
As the leak closes, patient will require less volume
to maintain the UO and BP
o Target MAP: 60 mmHg to ensure optimal
end-organ perfusion
o Target UO:30 cc/h in adults and 1 to 1.5
cc/kg/hr in pediatric patients
Maintenance IV fluid with glucose
supplementation in addition to the calculated
resuscitation fluid with LR is given in children
under 20 kg
o They do not have sufficient glycogen
stores to maintain an adequate glucose
level in response to the inflammation.
Blood transfusions be used only when there is an
apparent physiologic need

3. Treatment of burn wound


Patients with acute burn injuries should NEVER receive
4. Diagnosis of Carbon Monoxide and Cyanide poisoning prophylactic oral/IV antibiotics
Unexpected neurologic symptoms should raise the level This intervention has been clearly demonstrated to
of suspicion for CO poisoning promote development of fungal infections and resistant
o Affinity of CO for hemoglobin is 200-250x organisms
more than that of O2, which decreases the Silver sulfadiazine: most widely used
levels of normal oxygenated hemoglobin and o Wide range of anti-microbial activity,
can quickly lead to anoxia and death primarily as topical prophylaxis against burn
o Treatment ||Administration of 100% oxygen wound infections rather than treatment of
is the gold standard, and reduces the half-life existing infection
of CO from 250 mins in room air to 40-60 mins o Not significantly absorbed systemically
Cyanide poisoning is seen in smoke inhalation injury o Side effects ||Neutropenia as a result of
o May have lactic acidosis or ST elevation neutrophil margination due to the
o Cyanide inhibits cytochrome oxidase, which in inflammatory response to burn injury
turn inhibit cellular oxygenation o Destroy skin grafts and is contraindicated on
o Treatment ||Consists of sodium thiosulfate, burns in proximity to newly grafted areas
hydroxocobalamin, and 100% oxygen Others: Mafenide acetate, Silver nitrate, Bacitracin,
Neomycin, and Polymyxin B
D. MANAGEMENT OF BURNS Pain management
1. Referral to a burn center (Refer to Table 35) o Important to administer an anxiolytic such as
benzodiazepine with the initial narcotics
Table 35. Guidelines for referral to a burn center
Guidelines for referral to a burn center 4.
Complications of burn
1. Partial thickness burns greater than 10% TBSA Hypothermia is one of the common pre-hospital
2. Burns involving the face, hands, feet, genitalia, perineum, or complications that contributes to resuscitation failure
major joints o Patients should be kept wrapped with clean
3. Third degree burns in any age group blankets
4. Electric burns (including lightning injury)
Ventilator-associated pneumonia, like all critically ill
5. Chemical burns
patients, is a significant problem in burn patients
6. Inhalational injury
o Simple measures such as elevating the head of
7. Patients with complicated preexisting medical disorders
8. Patients with burns and concomitant trauma in which the burn the bed and maintaining excellent oral hygiene
is the greatest risk and pulmonary toilet are recommended to
9. Burned children in hospitals without qualified personnel for help decrease the risk of postinjury
the care of children pneumonia
10. Burn injury in patients who will require special social, 4. Complications of burn(continuation)
emotional, or rehabilitative intervention Massive resuscitation of burn patients may lead to an
abdominal compartment syndrome
2. Resuscitation o Characterized by increased airway pressures
Rationale: Burn (and/or inhalational injury) drives with hypoventilation, and decreased urine
inflammatory response that leads to capillary leak output and hemodynamic compromise
As the plasma leaks into the extravascular space, o Treatment ||Decompressive laparotomy is
crystalloid administration maintains the the standard of care for refractory abdominal
intravascular volume compartment syndrome but carries an
TOPNOTCH MEDICAL BOARD PREP SURGERY SUPPLEMENT HANDOUT Page 27 of 81
For inquiries visit www.topnotchboardprep.com.ph or email us at topnotchmedicalboardprep@gmail.com
TOPNOTCH MEDICAL BOARD PREP SURGERY SUPPLEMENT HANDOUT - Jules Lopez,MD-MBA,Teddy Carpio,MD-MBA
For inquiries visit www.topnotchboardprep.com.ph or email us at topnotchmedicalboardprep@gmail.com
especially lethal prognosis in burn patients Treatment ||Supportive care including aggressive
o Adjunctive measures such as minimizing fluid, pulmonary toilet, routine use of nebulized agents (e.g.
performing truncal escharotomies, decreasing Salbutamol) and ventilation for ARDS
tidal volumes, and chemical paralysis should
be initiated before resorting to decompressive REVIEW QUESTIONS a
laparotomy
Burn patients may be at higher risk for catheter- 1. Which of the following patients should be immediately
related bloodstream infections referred to a burn center?
Full thickness burns with a rigid eschar can form a a. 20 y/o with a 12% partial thickness burn
tourniquet effect as the edema progresses, leading to b. 30 y/o with a major liver injury and a 15%
compromised venous outflow and eventually arterial partial thickness burn
inflow, leading to compartment syndrome c. 2% TBSA partial thickness burn to the anterior
o Common in circumferential extremity burns leg, crossing the knee
o Warning signs include paresthesia, pain, d. 10 y/o with a 7% partial thickness burn
decreased capillary refill, and progression to
loss of distal pulses Answer: A
All patients with a partial thickness burn >10% TBSA
5. Nutrition of burn patients should be transferred to a burn center. A patient with a
Burn injury causes a hypermetabolic response raising burn and other major trauma can be treated in the
baseline metabolic rates by as much as 200%, leading to trauma center first. Burns that involve the entire joint
catabolism of muscle proteins and decreased lean should be transferred to a burn center, but a small burn
body mass that delay functional recovery to the anterior surface of the knee would not necessarily
Early enteral feeding for patients help prevent loss of mandate transfer. Children should be transferred if there
lean body mass, slow the hypermetabolic response, and are no personnel able to care for them, but for a child
result in a more efficient protein metabolism with a 7% TBSA burn, this would not be mandatory
If enteral feeds are started within the first few hours (Refer to Table 35)
after admission, gastric ileus can often be avoided
2. Which of the following is indicated in a 46 y/o patient
6. Surgery with a 22% TBSA partial thickness burn?
Escharotomies are rarely needed within the first 8 a. Prophylactic 1st generation cephalosporin
hours following injury and SHOULD NOT be performed b. Prophylactic clindamycin
unless indicated because of the aesthetic sequelae c. Tetanus booster
Burn excision and wound coverage should ideally d. Tetanus toxoid
start within the first several days, and in larger burns,
serial excisions can be performed as the patient’s Answer: C
condition allows Patients with acute burn injuries should never receive
Excision is performed with repeated tangential slices prophylactic antibiotics. This intervention promote
until only non burned tissue remains development of fungal infections and resistant
It is appropriate to leave healthy dermis, which will organisms and was abandoned in the mid-1980s. A
appear white with punctate areas of bleeding tetanus booster should be administered in the ER.

7. Wound coverage/ Grafts 3. Formic acid burns are associated with?


Split thickness sheet autografts make the most durable a. Hemoglobinuria
wound coverings b. Rhabdomyolosis
In larger burns, meshing of autografted skin provides a c. Hypocalcemia
larger area of wound coverage, allowing drainage of d. Hypokalemia
blood and serous fluid to prevent accumulation under
the skin graft with subsequent graft loss Answer: A
The offending agents in chemical burns can be
8. Rehabilitation systematically absorbed and may causes specific
Should be initiated on admission metabolic derangements. Formic acid has been known
Immediate and ongoing physical and occupational to cause hemolysis and hemoglobinuria.
therapy is mandatory to prevent loss of physical
function 4. The major improvement in burn survival in the 20th
century can be attributed to the introduction of which of
E. INHALATIONAL INJURY the following therapies?
Commonly seen in tandem with burn injuries a. Antibiotics
Drastically increase mortality in burn patients b. Central venous fluid resuscitation
Causes injury in 2 ways: c. Nutritional support
o Direct heat injury to the upper airways d. Early excision of the burn wound
Leads to maximal edema in the first
24 to 48 hours after injury Answer: D
Will require short course of
endotracheal intubation for airway
protection WOUND HEALING
o Inhalation of combustion products into the
lower airways A. Classification of Wound Healing
Irritants (combustion products) B. Normal Phases of Wound Healing
cause direct mucosal injury leading C. Classification of Wounds
to mucosal sloughing, edema,
reactive bronchoconstriction, and
eventually obstruction of the lower A. CLASSIFICATION OF WOUND HEALING
airways Surgical wounds can heal in several ways
Physiologic effects include decrease lung compliance, (Refer to Figure 25 )
increase airway resistance work of breathing, increase o Primary intention: an incised wound that is
overall metabolic demands, and an increase in fluid clean and closed by sutures
requirements during resuscitation of patients with burn o Secondary intention: Because of bacterial
injuries contamination or tissue loss, a wound will be
TOPNOTCH MEDICAL BOARD PREP SURGERY SUPPLEMENT HANDOUT Page 28 of 81
For inquiries visit www.topnotchboardprep.com.ph or email us at topnotchmedicalboardprep@gmail.com
TOPNOTCH MEDICAL BOARD PREP SURGERY SUPPLEMENT HANDOUT - Jules Lopez,MD-MBA,Teddy Carpio,MD-MBA
For inquiries visit www.topnotchboardprep.com.ph or email us at topnotchmedicalboardprep@gmail.com
left open to heal by granulation tissue o Exert a downregulating effect on fibroblast
formation and contraction collagen synthesis by cell-associated
o Tertiary intention or delayed primary interferon-gamma, TNF alpha, and IL1
closure: represents a combination of the first
two, consisting of the placement of sutures, Figure 26. Phases of wound healing
allowing the wound to stay open for a few
days, and the subsequent closure of the
sutures

Figure 25. Different clinical approaches to the closure and healing of


acute wounds

2. Proliferation
Roughly spans day 4 through 12
Phase where tissue continuity is re-established
Fibroblasts and endothelial cells are the last cell
populations to infiltrate the healing wound
Strongest chemotactic factor for fibroblasts is PDGF
Upon entering the wound environment, recruited
fibroblasts first need to proliferate, and then become
activated, to carry out their primary function of matrix
B. NORMAL PHASES OF WOUND HEALING
synthesis remodeling
Normal wound healing follows a predictable pattern
Fibroblasts from wounds synthesize more collagen,
that can be divided into three overlapping phases:
proliferate less, and actively carry out matrix
1. Hemostasis and inflammation
contraction
2. Proliferation
o Type I collagen is the major component of
3. Maturation and remodeling
extracellular matrix in skin
o Type III, which is also normally present in
1. Hemostasis and Inflammation
skin, becomes more prominent and important
Hemostasis precedes and initiates inflammation with
during the repair process
the ensuing release of chemotactic factors from wound
Endothelial cells also proliferate extensively during this
site
phase of healing, participating in angiogenesis, under
Cellular infiltration after injury follows a characteristic,
the influence of cytokines and growth factors such as
predetermined sequence
TNF-alpha, TGF-beta, and VEGF
o PMNs are the first infiltrating cells to enter
Macrophages represent a major source of VEGF
the wound site, peaking at 24 to 48 hours,
stimulated by increased vascular permeability,
3. Maturation and Remodeling
local prostaglandin release, and the presence
Begins during the fibroplastic phase
of chemotactic substances
Characterized by a reorganization of previously
o These cells DO NOT play a role in collagen
synthesized collagen
deposition and collagen synthesis
Collagen is broken down by matrix metalloproteases,
and the net wound collagen content is the result of a
Macrophages (Refer to Figure 26)
balance between collagenolysis and collagen synthesis
o Recognized to be essential in successful
There is a net shift toward collagen synthesis and
wound healing
eventually the re-establishment of extracellular matrix
o Achieve significant numbers by 48 to 96
composed of a relatively acellular collagen-rich scar
hours post injury and remain present until
Wound strength and mechanical integrity in the fresh
wound healing is complete
wound are determined by both the quantity and
o Participate in wound debridement via
quality of the newly deposited collagen
phagocytosis
The deposition of matrix at the wound site follows a
o Contribute to microbial stasis via oxygen
characteristic pattern: fibronectin and collagen type III
radical and nitric oxide synthesis
constitute the early matrix scaffolding,
o Activation and recruitment of other cells via
Glycosaminoglycans and proteoglycans represent the
mediators as well as directly by cell-cell
next significant matrix components, and collagen type I
interaction and intercellular adhesion
is the final matrix
molecules
By several weeks post injury, the amount of collagen in
the wound reaches a plateau, but the tensile strength
T lymphocytes
continues to increase for several more months
o Less numerous than macrophages
Scar remodeling continues for 6 to 12 months post
o Peak at about 1 week post injury and truly
injury, gradually resulting in a mature, avascular, and
bridge the transition from the inflammatory to
acellular scar
the proliferative stage of wound healing
Mechanical strength of the scar never achieves that
o Role is not fully defined
of the uninjured tissue
o Theory is that they play an active role in
modulation of the wound environment

TOPNOTCH MEDICAL BOARD PREP SURGERY SUPPLEMENT HANDOUT Page 29 of 81


For inquiries visit www.topnotchboardprep.com.ph or email us at topnotchmedicalboardprep@gmail.com
TOPNOTCH MEDICAL BOARD PREP SURGERY SUPPLEMENT HANDOUT - Jules Lopez,MD-MBA,Teddy Carpio,MD-MBA
For inquiries visit www.topnotchboardprep.com.ph or email us at topnotchmedicalboardprep@gmail.com
4. Epithelialization 2. Chronic
While tissue integrity and strength are being re- Defined as wounds that have failed to proceed through
established, the external barrier must also be restored the orderly process that produces satisfactory anatomic
Characterized primarily by proliferation and and functional integrity or that have proceeded through
migration of epithelial cells adjacent to the wound the repair process without producing an adequate
Process begin within day 1 of injury and is seen as anatomic and functional result
thickening of epidermis at the wound edge Wounds that have NOT healed in 3 months
Re-epithelialization is complete in less than 48 hours
in the case of approximated incised wounds, but may Table 37. Factors affecting wound healing
take longer in case of larger wounds, in which there is a Factors affecting wound healing
significant epidermal/dermal defect Systemic
Mediated by a combination of a loss of contact Age
inhibition, exposure to constituents of the extracellular Nutrition
matrix, particularly fibronectin, and cytokines produced Trauma
by immune mononuclear cells Metabolic diseases
Immunosuppression
Connective tissue disorders
5. Wound Contraction
Smoking
All wounds undergo some degree of contraction Local
Starts almost immediately after injury despite the Mechanical injury
absence of myofibroblasts Infection
For wounds that do not have surgically approximated Edema
edges, the area of the wound will be decreased by this Ischemic/necrotic tissue
action (healing by secondary intention), the shortening Topical agents
of the scar itself results in contracture Ionizing radiation
Myofibroblast has been postulated as being the major Low oxygen tension
cell responsible for contraction, and it differs from the Foreign bodies
normal fibroblast in that it possesses a cytoskeletal
structure
QUICK REVIEW a
C. CLASSIFICATION OF WOUNDS
1. Acute Normal wound healing follows a predictable pattern that
Heal in a predictable manner and time frame can be divided into three overlapping
Process occurs with few complications and the end phases:Hemostasis and inflammation, Proliferation,
result is a well-healed wound and Maturation and remodeling
Normal process of wound healing is characterized by a PMNs are the first infiltrating cells to enter the wound
constant and continual increase that reaches a plateau site, peaking at 24 to 48 hours
at some point post injury Myofibroblast has been postulated as being the major
Wounds with delayed healing are characterized by cell responsible for contraction
decreased wound breaking strength in comparison to
wounds that heal at a normal rate, however, they
eventually achieve the same integrity and strength as REVIEW QUESTIONS a
wounds that heal normally
Delayed healing is caused by conditions such as 1. The peak number of fibroblasts in a healing wound
nutritional deficiencies, infections, or severe trauma occurs?
which reverts to normal with correction of the a. 2 days post injury
underlying pathophysiology (Refer to Figure 27) b. 6 days post injury
Impaired healing is characterized by a failure to c. 15 days post injury
achieve mechanical strength equivalent to normally d. 60 days post injury
healed wounds
Patients with compromised immune system (diabetics, Answer: B
chronic steroid usage, tissues damaged by See Figure 26
radiotherapy) are prone to impaired healing
(Refer to Table 37) 2. The first cells to migrate into a wound are:
a. Macrophages
Figure 27. The acquisition of wound mechanical strength over time in b. T Lymphocytes
normal, delayed, and impaired healing c. PMNs
d. Fibroblasts

Answer: C
PMNs are the 1st infiltrating cells to enter the wound site,
peaking at 24-48 hours. Increased vascular permeability,
local prostaglandin release and the presence of
chemotactic substances, such as complement factors, IL-
1, TNF-alpha, TGF beta, platelet factor 4, or bacterial
products, all stimulate neutrophil migration.

3. The tensile strength of a completely healed wound


approaches the strength of uninjured tissue?
a. 2 weeks after injury
b. 3 months after injury
c. 12 months after injury
d. NEVER

Answer: D
By several weeks postinjury, the amount of collagen in
the wound reaches a plateau, but the tensile strength
continues to increase for several more months. Fibril
formation and fibril cross-linking result in decreased

TOPNOTCH MEDICAL BOARD PREP SURGERY SUPPLEMENT HANDOUT Page 30 of 81


For inquiries visit www.topnotchboardprep.com.ph or email us at topnotchmedicalboardprep@gmail.com
TOPNOTCH MEDICAL BOARD PREP SURGERY SUPPLEMENT HANDOUT - Jules Lopez,MD-MBA,Teddy Carpio,MD-MBA
For inquiries visit www.topnotchboardprep.com.ph or email us at topnotchmedicalboardprep@gmail.com
collagen solubility, increased strength and increased
resistance to enzymatic degradation of the collagen
matrix. Scar remodeling continues for many months (6- A. ANATOMY AND PHYSIOLOGY OF THE SKIN
12) post-injury, gradually resulting in a mature,
avascular and acellular scar. The mechanical strength of QUICK REVIEW a
the scar never achieves that of the uninjured tissue. Layers of the skin – p. 26
Phases of wound healing – p. 29
4. Which layer of the intestine has the greatest tensile
strength (ability to hold sutures)?
Epidermis
a. serosa o Keratinocyte transit time (basal layer to
b. muscularis shedding) is approximately 40 to 56 days.
c. submucosa o Melanocytes
d. mucosa Derived from precursor cells of the neural
crest/neuroectodermal in origin
Answer: C Produce melanin from tyrosine and cysteine
The submucosa is the layer that imparts the Despite differences in skin tone, the density
greatest tensile strength and gretest suture-holding of melanocytes is constant among
capacity, a characteristic that should be kept in individuals. It is the rate of melanin
mind during surgical repair of GI tract. Additionally, production, transfer to keratinocytes, and
serosal healing is essential for quickly achieving a melanosome degradation that determine
watertight seal from the luminal side of the bowel. the degree of skin pigmentation
The importance of the serosa is underscored by the Cutaneous melanocytes play a critical role in
significantly higher rates of anastomotic failure neutralizing the sun's harmful rays.
observed clinically in segments of bowel that are
UV-induced damage affects the function of
extraperitoneal and lack serosa (ex. Esophagus and tumor suppressor genes, directly causes cell
rectum) death, and facilitates neoplastic
transformation.
5. A 20 year old male presents to the ER with large
contaminated laceration received during a touch
football game. It has been irrigated with normal saline QUICK REVIEW a
and subsequently debrided. Which suture should be What factors increase melanin production?
used to close the subcutaneous layer? 1. UV radiation
2. Estrogen
a. biologic absorbable monofilament (plain gut) 3. Adrenocorticotropic hormone
b. synthetic absorbable monofilament 4. Melanocyte-stimulating hormone
c. absorbable braided
d. none of the above
o Keratinocytes
Answer: C Primarily found in the spindle layer
In general, the smallest suture required to hold the Contains intermediate filaments (keratin)
various layers of the wound in approximation provides flexible scaffolding resist external
should be selected in order to minimize suture- stress
related inflammation. Nonabsorbable or slowly Point mutations cause blistering diseases, such
absorbing monofilament sutures are most suitable as epidermolysis bullosa, associated with
for approximating deep fascial layers, particularly spontaneous release of dermal-epidermal
in the abdominal wall. Subcutaneous tissues should attachments.
be closed with braided absorbable sutures, with o Langerhans’ cells (not Langhan’s cells!)
care to avoid placement of sutures in fat. Although skin's macrophages; from the bone marrow
traditional teaching in wound closure emphasized expresses class II major histocompatibility
multiple-layer closures, additional layers of suture antigens antigen-presenting capabilities.
closure are associated with increased risk of wound Functions:
infection, especially when placed in fat. Drains may 1. rejection of foreign bodies
be placed in areas at risk of forming fluid 2. immunosurveillance against viral infections
collections. 3. immunosurveillance against neoplasms of
the skin
Dermis
o Collagen (main functional component of the dermis)
ORGAN SYSTEM PATHOLOGIES comprises 70% of its dry weight
o Skin is primarily comprised of type I collagen
o Fetal dermis is primarily comprised of type III
SKIN AND SOFT TISSUES collagen (reticulin fibers) provides tensile strength
(property of the skin that resists stretching) to both
A. Anatomy and Physiology of the Skin dermis and epidermis
B. Injuries to the Skin and subcutaneous Cutaneous Adnexal Structures
C. Infections of the skin and the subcutaneous 1. Eccrine glands: sweat-producing glands located over
D. Inflammatory diseases of the skin and the entire body but are concentrated on the palms,
subcutaneous soles, axillae, and forehead
E. Benign tumors of the skin and subcutaneous 2. Apocrine glands: Pheromone producing glands
F. Malignant tumors of the skin primarily found in the axillae and anogenital region.
G. Syndromic skin malignancies It is these structures that predispose both regions to
suppurative hidradenitis
3. hair follicles: contains a reservoir of pluripotential
stem cells critical in epidermal reproductivity

TOPNOTCH MEDICAL BOARD PREP SURGERY SUPPLEMENT HANDOUT Page 31 of 81


For inquiries visit www.topnotchboardprep.com.ph or email us at topnotchmedicalboardprep@gmail.com
TOPNOTCH MEDICAL BOARD PREP SURGERY SUPPLEMENT HANDOUT - Jules Lopez,MD-MBA,Teddy Carpio,MD-MBA
For inquiries visit www.topnotchboardprep.com.ph or email us at topnotchmedicalboardprep@gmail.com
- Solar or UV radiation: most common form of radiation
B. INJURIES TO THE SKIN AND SUBCUTANEOUS exposure
- Melanin: most important protective factor from UV
a. Exposure to Caustic substances related damage
- UV spectrum:
Table 38. Difference between acidic and alkali injury UVA (400 to 315 nm): majority of solar radiation
Acidic Alkali that reaches the Earth
Coagulative necrosis – can Liquefactive necrosis –
UVB (315 to 290 nm): less than 5% of all solar UV
damage nerves, blood vessels causes fat saponification that
and tendons but is less facilitates tissue penetration radiation; responsible for acute sunburn and
damaging compared to alkali and increases tissue damage chronic skin damage leading to malignant
injury producing a longer more degeneration (known risk factor in the
sustained injury compared to development of melanoma.)
acidic burns UVC (290 to 200 nm): absorbed by the ozone layer
Tx: copious irrigation with Tx: continuous irrigation with C. INFECTIONS OF THE SKIN AND THE SUBCUTANEOUS
either saline or water for 30 water for 2 hours or until
a. Cellulitis, Folliculitis, furuncles & carbuncles
minutes symptomatic relief is
Table 39: Comparison of skin infections
achieved
Cellulitis Folliculitis Furuncles Carbuncles
- Superficial, -infection of -begins as folliculitis - deep seated
- Intravenous fluid (IVF) extravasation: leakage of spreading the hair but progresses as a infections
injectable fluids into the interstitial space infection of the follicle fluctuant nodule that result in
o Is considered a chemical burn skin and subQ -usual cause: (boil/furuncle) multiple
o Produces chemical toxicity, osmotic toxicity and -usual cause: Staphylococc -tx: warm water draining
pressure effects in a closed environment. Grp. A strep & us, followed hastens liquefaction sinuses
o Culprits: S. aureus by G(-) & spontaneous -tx: incision
Cationic substances: K, Ca and bicarbonate -tx for organisms rupture; incision and drainage
uncomplicated -tx: adequate and drainage if
Osmotically active agents: TPN, hypertonic
cellulitis with hygiene necessary
dextrose solution no morbidities:
Antibiotics outpatient oral
Cytotoxic drugs / chemotherapeutic drugs – antibiotics
most common cause of extravasation in adults
o Most common site of extravasation in adults: b. Necrotizing soft tissue infections
dorsum of the hand - Basis of classification:
o Most common cause of extravasation in infants the tissue plane affected and extent of invasion
causing necrosis: high concentration dextrose, Ca, necrotizing fasciitis: rapid, extensive
bicarbonate and TPN infection of the fascia deep to the adipose
tissue
b. Thermal injuries – hypothermic vs hyperthermic necrotizing myositis: primarily involves the
injuries muscles but typically spreads to adjacent soft
tissues
QUICK REVIEW a the anatomic site
Jackson’s 3 zones of tissue injury for hyperthermic Most common sites: the external genitalia,
injuries – p. 26 perineum, or abdominal wall (Fournier
gangrene)
the causative pathogen
Hypothermic injuries polymicrobial more common than single
o Severe hypothermia primarily exerts its damaging organism infections
effect by causing direct cellular injury to bv walls most common causative organisms: group A
and microvascular thrombosis. streptococci, enterococci, coagulase-negative
o skin's tensile strength decreases by 20% in a cold staphylococci, S. aureus, S. epidermidis, and
environment [12°C, (53.6°F)]. Clostridium species
o Trench foot: reactive hyperthermia with blistering others (Gram negatives): Escherichia coli,
as a result of prolonged exposure to ice-cold water Enterobacter, Pseudomonas species, Proteus
after rapidly bringing it back to normal species, Serratia species, and bacteroides
temperature - risk factors: diabetes mellitus, malnutrition, obesity,
chronic alcoholism, peripheral vascular disease, CLL,
c. Pressure injury steroid use, renal failure, cirrhosis, and autoimmune
- 1 hour of 60 mmHg pressure can lead to deficiency syndrome
histologically identifiable venous thrombosis, muscle - tx: prompt recognition, broad-spectrum IV antibiotics,
degeneration, and tissue necrosis aggressive surgical debridement (should be extensive-
- Pressures: including all skin, subcutaneous tissue, and muscle, until
Normal arteriole: 32 mmHg there is no further evidence of infected tissue followed
Normal capillary: 20 mmHg by as needed debridement), and aggressive fluid
Normal venule: 12 mmHg replacement (needed to offset acute renal failure from
Sitting: 300 mmHg ongoing sepsis)
Sacral pressure at hospital mattress bed: 150
mmHg c. Hidradenitis suppuritiva
- Muscle tissue is more sensitive to ischemia than - is a defect of the terminal follicular epithelium
skin. Implication: necrosis usually extends to a deeper leading to apocrine gland blockage gives rise to
area than that apparent on superficial inspection abscess formation in the axillary, inguinal, and
- Treatment: relief of pressure, wound care, systemic perianal regions
enhancement (nutritional optimization) and surgical - Tx: warm compresses, antibiotics, and open drainage if
management (debridement of all necrotic tissue acute; wide excision with closure using skin graft or
followed by irrigation; if shallow ulcer close by local flap placement if chronic
secondary intention; if deeper ulcer require surgical
debridement and coverage) d. Actinomycosis
- is a granulomatous suppurative bacterial disease &
d. Radiation exposure deep cutaneous infections that present as nodules and
spread to form draining tracts caused by Actinomyces
TOPNOTCH MEDICAL BOARD PREP SURGERY SUPPLEMENT HANDOUT Page 32 of 81
For inquiries visit www.topnotchboardprep.com.ph or email us at topnotchmedicalboardprep@gmail.com
TOPNOTCH MEDICAL BOARD PREP SURGERY SUPPLEMENT HANDOUT - Jules Lopez,MD-MBA,Teddy Carpio,MD-MBA
For inquiries visit www.topnotchboardprep.com.ph or email us at topnotchmedicalboardprep@gmail.com
(pathognomonic: (+) sulfur granules within
purulent specimen). E. BENIGN TUMORS OF THE SKIN AND SUBCUTANEOUS
- Usual site: face or head (60%)
- Risk factors: tooth extraction, odontogenic infection, or a. Cutaneous cysts: Epidermal, dermoid or
facial trauma. trichelemmal
- Tx: Penicillin and sulfonamides; surgery for deep seated Table 42: Comparison between epidermal, dermoid & trichilemmal cyst:
infections.
Epidermal cyst Dermoid cyst Trichilemmal (pilar)
e. Viral infections – HPV cyst
Difference:
- Warts are epidermal growths resulting from human
- most common - congenital lesions that -2nd most common
papillomavirus (HPV) infection. - single, firm result when epithelium - when ruptured:
nodule is trapped during fetal produce an intense
Table 40: Comparison of HPV infections midline closure characteristic odor
Common Plantar Flat Venereal warts -location: - most common
wart warts warts (condylomata anywhere in the location: eyebrow -location: scalp (of
(verruca (verruca (verruca acuminata) body females)
vulgaris) plantaris) plana)
-fingers -soles and - the face, - the vulva, anus, and -histopath: -histopath:
and toes palms legs, and scrotum (relatively mature demonstrates -histopath: no
-described -resemble hands moist areas) epidermis squamous epithelium, granular layer;
as rough a common - slightly - STD complete with eccrine glands, and
and callus raised - HPV 6 & 11 granular layer pilosebaceous units. In
bulbous and flat. -buschke addition, these
Lowenstein tumor: particular cysts may
Extensive growths, develop bone, tooth, or
facilitated by nerve tissue on
concomitant HIV occasion
infection

Similiarity:
- histopathology: hyperkeratosis (hypertrophy of the -contain keratin (not sebum)
horny layer), acanthosis (hypertrophy of the -appear the same clinically (subcutaneous, thin-walled nodule
containing a white, creamy material)
spinous layer), and papillomatosis -treatment: excision; incision and drainage if infected; make sure to
- Tx: formalin, podophyllum, and phenol-nitric acid; remove the cyst wall to prevent recurrence
Curettage with electrodesiccation also can be used for
scattered lesions
- HPV types 5, 8, and 10: (+) association with b. Keratosis – seborrheic vs solar
squamous cell carcinoma:
lesions that grow rapidly, atypically, or ulcerate Table 43. Comparison between seborrheic keratosis and
should be biopsied actinic keratosis:
Seborrheic (or solar) Actinic keratosis
keratosis
D. INFLAMMATORY DISEASES OF THE SKIN AND
- considered as a -considered as a
SUBCUTANEOUS
premalignant lesion of SCC premalignant lesion of SCC
- appearance: light brown or (although at least 25%
a. Pyoderma gangrenosum yellow with a velvety, greasy spontaneously regress)
- Main characteristic: rapidly enlarging, destructive, texture
cutaneous necrotic lesion with undermined border and -arise in sun exposed areas
surrounding erythema (face, forearms, back of hands)
- (+) associated with a systemic disease 50% of the time -common in old age groups
(inflammatory bowel disease, rheumatoid arthritis, -sudden eruptions are
associated with internal
hematologic malignancy, and monoclonal
malignancies
immunoglobulin A gammapathy) -treatment: topical 5-
- Tx: Recognition of the underlying disease, systemic fluorouracil, surgical excision,
steroids or cyclosporine & chemotherapy with electrodesiccation, and
aggressive wound care and skin graft coverage dermabrasion

b. SSS vs TEN c. Nevi – acquired vs congenital: both are histologically


Table 41: comparison between SSSS & TEN similar.
SSSS TEN
Difference: Acquired melanocytic nevi
-caused by an exotoxin (TSS -caused by an immune response to - Classification is based on different stages of maturation
toxin-1)produced during staph certain drugs (sulfonamides, Junctional: epidermis
infection of the nasopharynx or phenytoin, barbiturates,
Compound: extend partially into dermis
middle ear cytokine release tetracycline)
throughout the body causing -more than 30% TBSA involved Dermal: dermis
diffuse injury and systemic (if less than 10% TBSA SJS)
symptoms Congenital nevi
- Rare (less than 1% of neonates)
-histopath: cleavage plane in the - histopath: structural defect at - Giant congenital lesions (giant hairy nevi): appear in a
granular layer of epidermis dermoepidermal jxn; similar to a swim trunk distribution, chest, or back
2nd degree burn - may develop into malignant melanoma in 1 to 5% of
Similarity: cases
- appearance: skin erythema, bullae formation, wide area of tissue - tx: total excision of nevus
loss
-diagnosis: skin biopsy
treatment: fluid and electrolyte replacement, as well as wound care
similar to burn therapy
- appearance: skin erythema, bullae formation, wide area of tissue
loss
-diagnosis: skin biopsy
treatment: fluid and electrolyte replacement, as well as wound care
similar to burn therapy

TOPNOTCH MEDICAL BOARD PREP SURGERY SUPPLEMENT HANDOUT Page 33 of 81


For inquiries visit www.topnotchboardprep.com.ph or email us at topnotchmedicalboardprep@gmail.com
TOPNOTCH MEDICAL BOARD PREP SURGERY SUPPLEMENT HANDOUT - Jules Lopez,MD-MBA,Teddy Carpio,MD-MBA
For inquiries visit www.topnotchboardprep.com.ph or email us at topnotchmedicalboardprep@gmail.com
d. Vascular tumors of the skin and subcutaneous
F. Malignant tumors of the skin
Table 44: Comparison between hemangioma, vascular malformation,
port wine stain and glomus tumors.
Hemangioma Vascular Capillary Glomus Basal cell carcinoma
malformat malformat tumors - most common type of skin cancer.
ion ion (port - Arises from the pluripotential basal epithelial cells of
wine epidermis and NOT DERMIS!
stain)
- Slow growing and metastasis is rare but are capable
-most common - vascular -flat, dull- -benign
cutaneous lesion of malformati red lesion -located at of extensive local tissue destruction
infancy ons are a often the - Subtypes:
-benign lesion that result of located on extremities Nodulocystic/noduloulcerative
present soon after structural the - arise from 70% of BCC tumors (most frequent form)
birth (not at birth!) abnormalit trigeminal dermal Waxy and frequently cream
-histopath: mitotically ies formed (CN V) neuromyoa colored/translucent;over time, can present as
active endothelial cells during distributio rterial a rolled, pearly borders surrounding a central
surrounding several, fetal n on the apparatus
ulcer (rodent ulcer)
confluent blood-filled developm face, trunk, (glomus
spaces ent or bodies). Morpheaphorm
-enlarge at 1st year of -histopath: extremities - usually flat, plaque-like lesion
life 90% eventually enlarged ; presents most aggressive clinically (due to presence of
involute vascular associated with severe type IV collagenase that facilitates local
-tx: if it interferes with spaces with pain, point spread) early excision
airway, vision, and lined by sturge- tenderness basosquamous type
feeding or results to nonprolifer weber and cold combination of both BCC + SCC
systemic problems ating symdrome sensitivity
aggressive treated right away!
(thrombocytopenia or endotheliu s -tx: tumor
high-output cardiac m. (leptomeni excision - Tx:
failure) resection - grow in ngeal Less than 2mm nodular lesions: curettage,
and proportion angiomato electrodesiccation, or laser vaporization.
prednisone/interferon to the body sis, If located at cheek, nose, or lip: Mohs' surgery
alpha 2 (for rapidly and never epilepsy, Large tumors, those that invade surrounding
enlarging lesions) involute and structures, & aggressive histologic types
glaucoma) (morpheaform, infiltrative, and
basosquamous): surgical excision with 0.5-cm
to 1-cm margins.
- Syndromic skin malignancies associated with BCC:
basal cell nevus (Gorlin's) syndrome:
e. Soft tissue tumors autosomal dominant disorder characterized
Table 45. Comparison of lipoma, dermatofibroma & by the growth of hundreds of BCCs during
achrochordon young adulthood.
Lipoma Dermatofibroma Achrochordo Palmar and plantar pits: common physical
n (skin tags)
finding
- most common - solitary, soft-tissue - fleshy,
subcutaneous nodules measuring 1 to pedunculated Tx: excision of aggressive and symptomatic
neoplasm 2 cm masses lesions
- soft and fleshy -usual location: legs and -usual nevus sebaceus of Jadassohn:
on palpation flanks location: lesion containing several cutaneous tissue
--usual location: -histopath: preauricular elements that develops during childhood.
back unencapsulated areas, axillae,
-histopath: connective tissue whorls trunk, and
Squamous cell carcinoma
lobulated tumor containing fibroblasts eyelids
composed of - do biopsy for atypical -tx: “tying-off” - Arise from epidermal keratinocytes
normal fat cells presentation or resection - Less common than BCC
-tx: excision -tx: excision - Highly invasive and tends to metastasize
- Tend to occur in persons with blond hair, light, thin, dry
f. Neural tumors and irritated skin.
- Benign - In situ lesions: Bowen’s disease; if in the penis
- Arise from the nerve sheath erythroplasia of Queyrat
- Risk factors:
Table 46. Comparison of neurofibroma, neurilemoma & Skin lesions: actinic keratosis, atrophic
granular cell tumor dermatitis
Neurofibroma Neurilemoma Granular cell Occupational exposure: arsenics, nitrates and
tumor hydrocarbons
-sporadic, solitary - solitary tumors - solitary lesions
Syndromic malignancies associated with SCC:
-can be syndromic arising from cells of the skin or,
(von Recklingh - of the peripheral more commonly, epidermolysis bullosus
ausen's disease: nerve sheath the tongue lupus erythematosus
café au lait spots, Epidermodysplasia verruciformis
Lisch nodules, rare autosomal recessive disease
and an autosomal associated with infection with HPV
dominant Xeroderma pigmentosum
inheritance)
autosomal recessive disease associated
- with direct
nerve attachment
with a defect in cellular repair of DNA
damage.
- histopath: , - histopath: tumor -histopath: - tumor thickness correlates well with malignant
proliferation of contains Schwann granular cells behavior.
perineurial and cells with nuclei derived from more than 4 mm: Tumor recurrence is more
endoneurial packed in Schwann cells prevalent
fibroblasts with palisading row that often if 10 mm or more: these lesions usually have
Schwann cells - tx: resection infiltrate the
associated metastasis
embedded in surrounding
collagen striated muscle.
-tx:resection
TOPNOTCH MEDICAL BOARD PREP SURGERY SUPPLEMENT HANDOUT Page 34 of 81
For inquiries visit www.topnotchboardprep.com.ph or email us at topnotchmedicalboardprep@gmail.com
TOPNOTCH MEDICAL BOARD PREP SURGERY SUPPLEMENT HANDOUT - Jules Lopez,MD-MBA,Teddy Carpio,MD-MBA
For inquiries visit www.topnotchboardprep.com.ph or email us at topnotchmedicalboardprep@gmail.com
- Burn scars (Marjolin's ulcer), areas of chronic Location: lesions of the extremities have a better
osteomyelitis, and areas of previous injury tend to prognosis than patients with melanomas of the
metastasize early. head, neck, or trunk
- Tx: (+) ulceration (due to increased angiogenesis):
Excision with 1 cm margin + histologic worse prognosis
confirmation of tumor free borders Gender: females have higher survival rates than
If located at cheek, nose, or lip: Mohs' surgery men
Regional LN excision is indicated for clinically Tumor types:
palpable nodes Best: lentigo maligna
If SCC arises from chronic wounds, Worse: acral lentiginous
lymphadenectomy before development of palpable - Staging from AJCC: breslow and clark level
nodes (prophylactic LN dissection) is indicated Breslow thickness: the vertical thickness of the
because it is more aggressive and lymph node primary tumor (from the granular layer of the
metastases are observed more frequently epidermis or base of ulcer to the greatest depth of
- Metastatic disease is a poor prognostic sign (13% the tumor); most important prognostic variable
survival after 10 years). predicting survival in those with cutaneous
melanoma; considered to be more precise in
MUST KNOW a predicting biologic behavior
I: 0.75 mm or less
Keratoacanthoma, which is characterized by rapid growth, II: 0.76 to 1.5 mm
rolled edges and a crater filled with keratin, can be confused III: 1.51 to 4.0 mm
with SCC or BCC. It spontaneously involutes over time. IV: 4.0 mm or more
Biopsy lesion to rule out carcinoma. Clark level: anatomic depth of invasion
I: superficial to basement membrane (in situ)
II: papillary dermis
Moh’s surgery for BCC and SCC (nice to know!) III: papillary/reticular dermal junction
- This precise, specialized surgical technique uses IV: reticular dermis
minimal tissue resection and immediate microscopic V: subcutaneous fat
analysis to confirm appropriate resection yet limit - LN status & Metastasis:
removal of valuable anatomy. Evidence of tumor in regional LNs is a poor
- Done for aesthetic purposes prognostic sign (automatic stage III)
- uses serial excision in small increments coupled with Identification of distant metastasis is the worst
immediate microscopic analysis to ensure tumor prognostic sign ( automatic stage IV)
removal all specimen margins are evaluated. - Dx: excisional biopsy
Recurrence and metastases rates are comparable to - Tx:
those of wide local excision. Melanoma in situ/lentigo maligna melanoma in
face: 0.5 cm margins
1mm or less: excision with 1 cm margin
Malignant Melanoma 1 to 4 mm: excision with 2 cm margin
- Arise from melanocytes More than 4 mm or (+) satellosis: 3-5 cm margin
- Premalignant lesion: dysplastic nevi (vs freckles - High dose interferon has a role in high risk
benign melanocytic neoplasms found on the skin) melanoma
- Most common location: skin (>90%); other sites: anus, LN dissection:
eyes Sentinel LN biopsy: 1mm or thicker with
- 4%: discovered as metastases without any identifiable clinically negative nodes or 0.75 mm thick +
primary site. clark level IV or ulcerated
- Suspicious features: pigmented lesion with an Radical regional lymphadenectomy:
irregular border, darkening coloration, ulceration, clinically (+) nodes with no evidence of distant
raised surface and recent changes in nevus disease on metastatic work up.
appearance
- Risk factors: QUICK REVIEW a
increased sun exposure of fair skinned people to Remember:
solar radiation Moh’s surgery is not appropriate for any type of
Familial dysplastic nevus syndrome melanoma
autosomal dominant disorder If melanoma is 4mm or greater + clinically negative
- Subtypes: nodes perform metastatic work up first
Table 47. comparison of malignant melanoma subtypes:

Superficial Nodular Lentigo Acral lentiginous Merkel cell carcinoma


spreading maligna - Primary Neuroendocrine Carcinoma of the Skin
- most common -15 to 30% -4 to -2 to 8% of - associated with a synchronous or metasynchronous SCC
type (70% of of 15% of melanomas (least 25% of the time.
melanomas) melanoma melano common) - Tx: wide local resection with 3-cm margins +
- location: s mas -occurs at palms, Prophylactic regional LN dissection + adjuvant radiation
anywhere on the - darker - occur soles, and subungual therapy are recommended.
skin except the coloration on neck, regions
- Prognosis: worse than malignant melanoma
hands and feet and often face, - Hutchinson's sign:
- flat and raised and presence of
measure 1 to 2 -lack hands of pigmentation in the Kaposi’s sarcoma
cm in diameter radial elderly proximal or lateral - rubbery bluish nodules that occur primarily on the
at diagnosis growth -best nail folds; diagnostic extremities (also skin and viscera)
- Before vertical - progno of subungual - usually multifocal rather than metastatic.
extension, a aggressive sis melanoma - Histopath: capillaries lined by atypical endothelial cells.
prolonged but same -dark skinned + - seen in people of Eastern Europe or sub-Saharan Africa,
radial growth prognosis acral lentiginous AIDS or immunosuppression with chemotx
phase is with melanoma:
characteristic of superficial increased risk of
- locally aggressive but undergo periods of remission
these lesions spreading malignancy

- Prognostic indicators:
TOPNOTCH MEDICAL BOARD PREP SURGERY SUPPLEMENT HANDOUT Page 35 of 81
For inquiries visit www.topnotchboardprep.com.ph or email us at topnotchmedicalboardprep@gmail.com
TOPNOTCH MEDICAL BOARD PREP SURGERY SUPPLEMENT HANDOUT - Jules Lopez,MD-MBA,Teddy Carpio,MD-MBA
For inquiries visit www.topnotchboardprep.com.ph or email us at topnotchmedicalboardprep@gmail.com
Extramammary Paget’s disease melanomas 4 mm in thickness and for those with
- cutaneous lesion that appears as a pruritic red patch associated satellosis. Moh’s chemosurgery is not
that does not resolve appropriate for the treatment of any melanomas. The
- histologically similar to the mammary type. indications for elective lymph node dissection remain
controversial. Sentinel lymph node biopsy is indicated
for aptients with melanoma 1 mm or thicker with
REVIEW QUESTIONS clinically negative nodes. The indication is extended to
patients with 0.75 mm thick melanomas if they are
1. Match the item in the left hand column with the Clark’s level IV or ulcerated. Patients with clinically
appropriate item in the right hand column positive lymph nodes with no evidence of distant
disease on metastatic workup (CT of chest, abdomen
a. modulate cold sensation a. Ruffini’s and pelvis; MRI of brain; PET) should undergo radical
endings regional lymphadenectomy. Patients with primary
b. modulate sensitivity to warmth b. Krause’ end- tumors 4 mm or greater with clinically negative nodes
bulb should undergo metastatic workup before undergoing
c. modulate sensation of pressure c. Meissner’s sentinel node biopsy and wide local excision.
corpuscles
d. modulate tactile sensation d. Pacinian 3. With regards to keloids and hypertrophic scars, which of
corpuscles the following statements is/are true?
e. modulate thermoregulation e. autonomic
nerve endings A. There are no histologic differences between the two
B. The differences between hypertrophic scar and
Answer: A-b; B-a; C-d; D-c; E-e keloid are clinical, not pathologic
A variety of highly specialized structures are responsible C. Hypertrophic scars outgrow their original borders
for modulating the skin’s various sensory functions. The D. Hypertrophic scars and keloids have been treated
numbers of these structures vary with the region of the successfully with intralesional injection of steroids
body. Pacinian corpuscles are found in the E. Keloids are seen in dark-skinned individuals,
subcutaneous tissue, in the nerves of the palm of the whereas hypertrophic scars are seen in fair-skinned
hand and the sole of the foot, and in other areas. Each of individuals
these corpuscles is attached to and encloses the
termination of a single nerve fiber. They are involved in Answer: A,B,D
the sensation of pressure. Ruffini’s endings are a variety Histologically, keloids and hypertrophic scars appear the
of nerve endings in the subcutaneous tissue of the same. Hypertrophic scars are thick, red, raised scars
fingers and modulate sensitivity to warmth. Krause’s that do not outgrow their original borders, whereas
end-bulbs are formed by the expansion of the keloids do. Keloids are dense accumulations of fibrous
connective tissue sheath of medullated fibers and are tissue that form at the surface of the skin. The defect
involved in the sensation of cold. Meissner’s corpuscles appears to result from a failure in collagen breakdown
occur in the papillae of the corium of the hands, the feet, rather than an increase in its production. Keloids and
the skin of the lips and other areas concerned with hypertrophic scars have been successfully treated with
tactile sensation. Autonomic fibers that synapse to intralesional steroid injectin, radiation, pressure and the
sweat glands and receptors in the vasculature govern use of silicone gel sheets.
thermoregulation.

2. Select the treatment options(s) in the 1st set of choices BREAST


(UPPER CASE) that is/are most appropriate for the
melanoma case summaries outlined in the 2nd set of A. Embryology of the breast
choices (lower case) B. functional anatomy of the breast
C. Physiology of breast
A. Level III superficial spreading melanoma (0.4 mm D. infectious and inflammatory disorders of the breast
thick with clinically negative regional lymph nodes E. common benign disorders and diseases of the
B. Level IV nodular melanoma (2mm thick) with breast
satellosis and clinically negative regional lymph F. breast cancer
nodes G. special clinical situations
C. Level IV superficial spreading melanoma (1.5 mm
thick) with palpable regional lymph nodes
D. Level IV acral lentiginous melanoma (2 mm thick)
with clinically negative regional lymph nodes A. EMBRYOLOGY OF THE BREAST
E. Level II lentigo maligna melanoma (0.3 mm) - 5th or 6th week of fetal development mammary ridges
(thickened ectoderm): precursors of breast
Extends from the base of the forelimb (future
f. Moh’s micrographic surgery axilla) to the hind limb (future inguinal region)
g. Wide local excision with 0.5 cm margins - Witch’s milk: (+) breast secretions in an infant
h. Wise local excision with 1.0 cm margins secondary to maternal hormones that crosses the
i. Wide local excision with 2.0 cm margins placenta
j. Wide local excision with 4.0 cm margins - Anomalies in embryology:
k. Sentinel lymph node biopsy Polymastia: accessory breast; can be seen in
l. Regional lymph node biopsy Turner’s syndrome (ovarian agenesis and
m. Radical regional lymphadenectomy dysgenesis)and Fleischer’s syndrome
(displacement of the nipples and bilateral renal
Answer: A – c; B – e,f; C – d,h; D – d,f; E – b hypoplasia); can enlarge during pregnancy &
Virtually all melanomas are best treated by wide lactation
excision. The excision margin that minimizes the risk of Polyethelia: accessory nipples; maybe associated
local recurrence depends on the thickness of the tumor. with CVS and urinary tract anomalies
Melanoma in situ and thin lentigo maligna melanomas Amastia: congenital absence of breast due to
of the face are treated adequately by margins of 0.5 cm. arrest in mammary ridge development during the
For melanomas less than 1.0 mm thick, 1 cm excision 6th week
margins are appropriate. For intermediate thickness Poland’s syndrome: hypoplasia or complete
melanoma (1-4 mm), a 2 cm margin is sufficient. absence of the breast, costal cartilage and rib
Margins of 3-5 cm are generally employed for
TOPNOTCH MEDICAL BOARD PREP SURGERY SUPPLEMENT HANDOUT Page 36 of 81
For inquiries visit www.topnotchboardprep.com.ph or email us at topnotchmedicalboardprep@gmail.com
TOPNOTCH MEDICAL BOARD PREP SURGERY SUPPLEMENT HANDOUT - Jules Lopez,MD-MBA,Teddy Carpio,MD-MBA
For inquiries visit www.topnotchboardprep.com.ph or email us at topnotchmedicalboardprep@gmail.com
defects, hypoplasia of the subcutaneous – level III the pectoralis
tissues of the chest wall, and brachysyndactyly minor muscle
Symmastia: webbing between the breasts across Interpector interposed receive lymph drainage
the midline (no cleavage) al group between the directly from the breast. The
(Rotter’s pectoralis major lymph fluid that passes
Inverted nipple: occurs in 4% of infants
nodes)- and pectoralis through the interpectoral
level II minor muscles group of lymph nodes
B. FUNCTIONAL ANATOMY OF THE BREAST passes directly into the
- It extends from the level of the 2nd or 3rd rib to the central and subclavicular
inframammary fold at the 6th or 7th rib. groups.
- It extends transversely from the lateral border of the
sternum to the anterior axillary line. Figure 28. Axillary lymph node groups
- The deep or posterior surface of the breast rests on the
fascia of the pectoralis major, serratus anterior, external Reference point:
pectoralis minor
oblique abdominal muscles, & the upper extent of the
(not major!)
rectus sheath. Level I: lateral to
- retromammary bursa: located at posterior aspect of the pectoralis
the breast between the investing fascia of the breast and minor muscle (PM)
the fascia of the pectoralis major muscles. Level II: deep to
- The axillary tail of Spence extends laterally across the PM
anterior axillary fold. Level III: medial to
- upper outer quadrant: greatest volume; most the PM.
common site of breast cancer
Arrows indicate the
- Blood supply: direction of lymph
flow.
Table 48. Blood supply of the breast Also seen: axillary
Arterial blood supply Venous blood supply vein & its major
- perforating branches of the -perforating branches of the tributaries,
internal mammary artery internal thoracic vein supraclavicular LN
- lateral branches of the -perforating branches of the
posterior intercostal arteries posterior intercostal veins
C. PHYSIOLOGY OF BREAST
-branches from axillary artery -tributaries of the axillary vein.
(highest thoracic, lateral - Batson's vertebral venous - Breast development and function
thoracic, and pectoral branches plexus : possible route for Hormonal stimuli:
of the thoracoacromial artery breast cancer metastases to the Estrogen: ductal development
vertebrae, skull, pelvic bones, Progesterone: differentiation of epithelium &
and central nervous system. lobular development
Prolactin: 1o hormonal stimulus for
- Innervation: lactogenesis in late pregnancy & the
Sensory innervation to breast & anterolateral chest postpartum period.
wall: Lateral cutaneous branches of the 3rd – 6th - Gynecomastia: enlarged breast in males measuring at
intercostal nerves (slips out in between serratus least 2 cm in diameter
anterior muscles) - Gynecomastia generally does not predispose the male
Intercostobrachial nerve: lateral cutaneous breast to cancer unless syndromic
branch of the second intercostal nerve; injury - Physiologic gynecomastia occurs due to excess in
to this nerve results to loss of sensation over circulating estrogens (in relation to circulating
the medial aspect of the upper arm. testosterone):
Cutaneous branches from cervical plexus (anterior neonatal period: action of placental estrogens on
branches of the supraclavicular nerve): supply a neonatal breast tissues
limited area of skin over the upper portion of the adolescence: excess of estradiol relative to
breast. testosterone; can be unilateral
- Lymphatics: senescence: circulating testosterone level falls;
6 axillary lymph node groups: usually bilateral

Table 49. Location and drainage pattern of breast D. INFECTIOUS AND INFLAMMATORY DISORDERS OF
Name location drainage THE BREAST
Lateral medial or upper extremity; receives
(axillary posterior to the 75% drainage of the
a. Breast abscess
vein vein breast; most common site
group) – of axillary LN metastasis - Staphylococcus aureus (more localized & deep) and
level I Streptococcus (diffuse superficial involvement) species:
Anterior or lower border of lateral aspect of the breast causative organisms
pectoral the pectoralis - SSx: point tenderness, erythema, and hyperthermia.
(external minor muscle - Risk factor: lactation (because a lactating breast is an
mammary contiguous with excellent culture medium)
group) – the lateral - Tx: preoperative UTZ + incision & drainage (if already
level I thoracic vessels with suppuration) + local wound care (warm
Posterior posterior wall of lower posterior neck, the
compresses &IV antibiotics - penicillins or
of the axilla at the posterior trunk, and the
subscapula lateral border of posterior shoulder cephalosporins).
r (scapular the scapula Remember: Biopsy of the abscess cavity wall is
group) – contiguous with recommended at the time of I&D rule out breast
level I the subscapular cancer with necrotic tumor.
vessels - Chronic breast abscesses: consider acid-fast bacilli,
Central embedded in receive lymph drainage both anaerobic and aerobic bacteria, and fungi.
group – the fat of the from the axillary vein, If fungal. Consider blastomycosis or sporotrichosis
level II axilla lying external mammary, and
(rare)
immediately scapular groups of lymph
posterior to the nodes, and directly from the
pectoralis minor breast b. Epidemic puerperal mastitis
muscle - MRSA: causative organism
Apical posterior and from all of the other groups - Transmission via suckling neonate
(subclavicu superior to the of axillary lymph nodes - Tx: stop breastfeeding , antibiotics & I&D
lar group) upper border of
TOPNOTCH MEDICAL BOARD PREP SURGERY SUPPLEMENT HANDOUT Page 37 of 81
For inquiries visit www.topnotchboardprep.com.ph or email us at topnotchmedicalboardprep@gmail.com
TOPNOTCH MEDICAL BOARD PREP SURGERY SUPPLEMENT HANDOUT - Jules Lopez,MD-MBA,Teddy Carpio,MD-MBA
For inquiries visit www.topnotchboardprep.com.ph or email us at topnotchmedicalboardprep@gmail.com
c. Nonepidemic (sporadic) puerperal mastitis
- involvement of the interlobular CT of the breast e. Sclerosing adenosis
- tx: Emptying of the breast using breast suction pumps + - Common in childbearing and perimenopausal years
antibiotics - no malignant potential.
- characterized by distorted breast lobules + multiple
d. Zuska's disease (recurrent periductal mastitis) microcysts + benign calcifications
- recurrent retroareolar infections and abscesses.
- Risk factor: smoking f. Radial scars (1 cm or less) or Complex central
- Tx: antibiotics + I&D as necessary. sclerosis (more than 1 cm)
- characterized by central sclerosis, epithelial
e. Hidradenitis suppurativa proliferation, apocrine metaplasia, and papilloma
- can also occur in the nipple-areola complex formation
- originates within the Montgomery glands or axillary - can mimic cancer hence an excisional biopsy is done to
sebaceous glands. to exclude diagnosis of cancer
- Risk factor: chronic acne
- may mimic Paget's disease of the nipple or invasive g. Ductal hyperplasia
breast cancer. - Severity:
- Tx: Antibiotic + I&D Mild: 3-4 cell layers above the basement
membrane.
f. Mondor’s disease Moderate:5 or more cell layers above the basement
- a benign self limited condition which is a variant of membrane.
thrombophlebitis that involves the superficial veins of Florid ductal epithelial hyperplasia: occupies at
the anterior chest wall and breast. least 70% of a minor duct lumen.
- Involved veins: lateral thoracic vein, the - associated with an increased cancer risk
thoracoepigastric vein, and, less commonly, the
superficial epigastric vein. h. Intraductal papillomas
- SSx: acute pain in the lateral aspect of the breast or the - Seen in premenopausal women.
anterior chest wall with palpation of a tender, firm cord - common symptom: serous or bloody nipple
along the distribution of the major superficial veins. discharge
- Tx: anti-inflammatory medications + warm compresses - Gross appearance: pinkish tan, friable,
along the symptomatic vein + Restriction of motion of - rarely undergo malignant transformation & no
the ipsilateral extremity and shoulder + brassiere increased risk of breast cancer, unless multiple
support of the breast are important (4 to 6 weeks) or
excision of vein (if not improving) i. Atypical proliferative disease
- has some of the features of Ca in situ but lack a major
E. COMMON BENIGN DISORDERS AND DISEASES OF defining feature of Ca in situ or have the features in less
THE BREAST than fully developed form
a. Fibroadenoma - Atypical ductal hyperplasia & lobar hyperplasia
- seen predominantly in younger women aged 15 to 25 Increases risk of breast cancer 4x; if with (+)
years family hx, 10x
- can be self limiting
- if greater than 3cm consider giant fibroadenoma F. BREAST CANCER
- if multiple (more than 5 lesions in 1 breast) - risk factors
considered as abnormal increased exposure to estrogen: early menarche,
- tx: cryoablation, surgical removal or observation nulliparity, late menopause, older age at first live
birth (after the age of 30 yo), HRT, obesity, (major
b. Cyclical mastalgia and nodularity source of estrogen in postmenopausal women is
- associated with premenstrual enlargement of the breast the conversion of androstenedione to estrone by
- physiologic. adipose tissue)
- If Painful nodularity persists for >1 week of the radiation exposure: patients with multiple
menstrual cycle consider a disorder. fluoroscopies, mantle radiation for treatment of
- bilateral bloody nipple discharge can be seen in hodgkin’s lymphoma
epithelial hyperplasia of pregnancy increased alcohol intake (leads to increased
estradiol levels)
c. Breast cysts high fat diet (increased serum estrogen levels)
- occurs when the stroma involutes too quickly, and prolonged use of OCPs (particularly estrogen-
alveoli remain forming microcysts & macrocysts plus-progesterone) and HRT
- characteristics of benign lesions: sharp, smooth (+) family history of breast cancer: the greater
margins, a homogenous interior and posterior the number of relatives affected, the closer the
enhancement (vs malignancy which will show irregular genetic relationship, the younger the age at
and jagged margins, heterogenous interior and posterior diagnosis, and the presence of bilateral versus
shawoding) unilateral disease all increased the likelihood of
- management: needle biopsy ( 1st line investigation for development of breast cancer in an individual.
palpable breast masses)
if (+) fluid on aspiration aspirate to dryness, no MUST KNOW
need to do cytologic examination Remember:
If after aspiration, (+) residual mass do UTZ
guided needle biopsy Smoking is not considered a risk factor for breast
If blood stained fluid aspirate 2 mL for cytologic cancer!!!!!!!!!!! Please don’t make the mistake of answering this
examination, utz imaging and biopsy solid areas as part of the risk factors in breast cancer.
- If complex cyst rule out malignancy.

d. Calcium deposits - risk management


- benign Postmenopausal hormone replacement tx
- cause: cellular secretions , debris or by trauma and Widely prescribed because it is effective in
inflammation. controlling symptoms of estrogen deficiency
- (+) cancer if <0.5 mm in size, fine, linear calcifications, (vasomotor symptoms such as hot flashes,
may branch (microcalcifications). night sweats and their associated sleep
TOPNOTCH MEDICAL BOARD PREP SURGERY SUPPLEMENT HANDOUT Page 38 of 81
For inquiries visit www.topnotchboardprep.com.ph or email us at topnotchmedicalboardprep@gmail.com
TOPNOTCH MEDICAL BOARD PREP SURGERY SUPPLEMENT HANDOUT - Jules Lopez,MD-MBA,Teddy Carpio,MD-MBA
For inquiries visit www.topnotchboardprep.com.ph or email us at topnotchmedicalboardprep@gmail.com
deprivation, osteoporosis, and cognitive (because the risk of breast cancer in BRCA
changes) mutation carriers increases after age 30 years)
Results of Women’s health initiative study
(2002): breast Ca risk is increased to 3- - HER-2 gene
4fold after > 4 years of use + no reduction Encodes transmembrane tyrosine kinase, a protein
in CAD or CVD with potent growth stimulating activity
In breast cancer, this gene is amplified (indicating
screening mammogram more rapid growth & aggressive behavior)
routine screening mammography starting 50 Treatment if there is a mutation: Herceptin
years old age reduces mortality from breast
cancer by 33% - Breast cancer signs and symptoms:
baseline mammography at age 35 mass (most common)
annual mammographic screening if size is 1 cm mass has been present for 5
beginning at age 40. years
If (+) family history for breast cancer breast enlargement or asymmetry
Baseline mammogram 10 years before nipple changes - retraction, or discharge ( due to
the youngest age of diagnosis of breast ca shortening of Cooper's suspensory ligament)
among 1st degree relatives. (this rule is skin dimpling
modified if age of diagnosis is less than ulceration / erythema of the skin
35) axillary mass or mets
firm or hard with continued growth of the
Chemoprevention metastatic cancer.
Tamoxifen: selective estrogen receptor involved sequentially from the low (level I) to
modulator the central (level II) to the apical (level III)
recommended only for women who have lymph node groups.
a Gail relative risk of 1.70 or ↑er. axillary lymph node status: most important
SE: deep vein thrombosis, pulmonary prognostic correlate of disease-free and
emboli, endometrial cancer overall survival
reduce the incidence of LCIS and ductal peau d'orange (Localized edema): blocked
carcinoma in situ (DCIS) drainage of lymph fluid
Raloxifene: estrogen receptor modulator musculoskeletal discomfort.
Equivalent to tamoxifen Distant metastases: most common cause of death
associated with a more favorable in breast cancer patients
adverse event profile Due to neovascularization (hematogenous
no effect on LCIS or DCIS spread) cancer cells shed directly to
axillary and intercostals veins or vertebral
prophylactic mastectomy column via batson’s plexus of veins.
greatly reduces risk for breast cancer Metastatic foci occurs after the 1o ca exceeds
only for high risk populations 0.5 cm in diameter
+ 3 years if with 40% risk of having breast Common sites of involvement (in order of
Ca frequency): bone, lung, pleura, soft tissues, and
+ 5 years if with 85% risk of having breast liver
cancer
MUST KNOW a
- BRCA mutation Remember:
Constitutes 5-10% of breast cancers Breast pain is usually associated with benign disease.
Autosomal dominant inheritance
tumor-suppressor genes
prevalent in Ashkenazi Jews - In situ breast cancer
Multicentricity: occurrence of a second
Table 50. Comparison of BRCA 1 & 2 breast cancer outside the breast quadrant of
BRCA 1 BRCA 2 the primary cancer (or at least 4 cm away)
- location: ch arm 17q -location: ch arm 13q Multifocality: the occurrence of a second
-predisposing genetic factor: - lifetime risk for carrier cancer within the same breast quadrant as the
45% of breast Ca & 85% of Ca: 85% for breast ca & 20% primary cancer (or within 4 cm of it)
ovarian Ca for ovarian ca ; if male
Difficult to differentiate from atypical
-lifetime risk for carrier carrier: 6%
Ca: 90% for breast ca & 40% -usually develops invasive hyperplasia or cancers with early invasion
for ovarian ca ductal carcinomas: well Subtypes:
- usually develops invasive differentiated, hormone
ductal carcinomas: poorly receptors (+) Table 51. Comparison of LCIS vs DCIS
differentiated and hormone -early age of onset, bilateral LCIS DCIS
receptor (-) breast cancer, & other Age 44-47 54-58
-early age of onset, bilateral associated ca: ovarian, colon, Incidence 2-5 5-10
breast cancer and other prostate, pancreatic, Clinical sx None Mass, pain &
associated ca: ovarian, colon gallbladder, bile duct & nipple discharge
and prostate stomach cancers, melanoma. Mammographic None / Microcalcification
sx mammographically s (usually in areas
Risk mgt strategies for BRCA carriers: featureless; may have of necrosis)
Prophylactic mastectomy and reconstruction calcifications in
adjacent tissues
Prophylactic oophorectomy (because of ↑ risk
occasionally
of ovarian ca) at the completion of
Premenopausal 2/3 (more common) 1/3
childbreaing or manopause + HRT
Intensive surveillance for breast and ovarian Incidence of 5% 2-46%
cancer synchronous
Chemoprevention invasive ca
BRCA mutation carriers who do not undergo Multicentricity 60-90 % 40-80% (more
prophylactic mastectomy must do clinical breast common if
examination every 6 months and mammography comedo type)
every 12 months beginning at age 25 years Bilaterality 50-70% (more 10-20%
common)
TOPNOTCH MEDICAL BOARD PREP SURGERY SUPPLEMENT HANDOUT Page 39 of 81
For inquiries visit www.topnotchboardprep.com.ph or email us at topnotchmedicalboardprep@gmail.com
TOPNOTCH MEDICAL BOARD PREP SURGERY SUPPLEMENT HANDOUT - Jules Lopez,MD-MBA,Teddy Carpio,MD-MBA
For inquiries visit www.topnotchboardprep.com.ph or email us at topnotchmedicalboardprep@gmail.com
LCIS DCIS Gross appearance: well circumscribed, soft &
Axillary 1% 1-2% hemorrhagic (when accompanied with a rapid
metastasis increase in size)
Subsequent carcinoma PE: bulky and mass is positioned deep within
Laterality Bilateral Ipsilateral the breast.
Interval to 15-20 y 5-10 y
Can mimic a benign condition on diagnostic
diagnosis
imaging (looks like a fibroadenoma on
Histo type Ductal; cytoplasmic ductal
mucoid globules are UTZ)
characteristic Occurs bilaterally in 20% of cases.
Specifics Only in ♀ breast; more Other name: Microscopically: dense lymphoreticular
common in whites; not intraductal infiltrate
considered an carcinoma (true
anatomic precursor anatomic Mucinous (colloid) carcinoma
of breast ca; only a precursor); 5% Occurs in the elderly
risk marker of male cancers Characteristic lesion: extracellular pools of
(increases risk of
mucin
breast ca 9x)
Gross appearance: glistening & gelatinous with
a firm consistency
Tx:
LCIS: observation, chemoprevention Papillary carcinoma
with tamoxifen, and bilateral total
Usually occurs in the 7th decade of life
mastectomy or may opt to do close follow More common in nonwhite ♀.
up + periodic PE + bilateral mammograms
defined by papillae with fibrovascular
for a more conservative approach
stalks and multilayered epithelium.
DCIS:
o > 4 cm or disease in >1 quadrant: Tubular carcinoma
mastectomy Usually occurs during perimenopausal or early
o Low-grade DCIS of the solid, menopausal periods.
cribriform, or papillary subtype that Microscopically: haphazard array of small,
is <0.5 cm: lumpectomy (If margins randomly arranged tubular elements is seen.
are free of disease) Well-differentiated type of infiltrating ductal
o Adjuvant tamoxifen therapy has a cancer
role for DCIS pt. Favorable diagnosis
MUST KNOW a Invasive lobular carcinoma
Remember: Histopath: small cells with rounded nuclei,
Mastectomy vs lumpectomy + adjuvant RT: same inconspicuous nucleoli, and scant cytoplasm;
mortality rate (<2%) but lumpectomy + adjuvant RT has (+) intracytoplasmic mucin, which may
a higher local recurrence rate (up to 9%, compared to 2% displace the nucleus (signet-ring cell
for mastectomy) carcinoma).
Role of RT: markedly decreases the risk of in-breast frequently multifocal, multicentric, and
recurrence and significantly reduces the risk that any bilateral.
recurrence will be invasive disease Hard to detect mammographically
High recurrence rate for DCIS comedo type
- Diagnosis of breast cancer
Mammography:
- Invasive Breast Cancer Views:
Paget's disease of the nipple (unrelated to Paget’s Craniocaudal (CC) view: medial aspect of
disease of the bone) the breast; permits greater breast
chronic, eryhthematous, eczemamatoid rash compression
or ulcer mediolateral oblique (MLO) view: images
associated with DCIS & invasive cancer. the greatest volume of breast tissue &
Pathognomonic sign: large, pale, upper outer quadrant and the axillary tail
vacuolated cells (Paget cells) in the rete of Spence
pegs of the epithelium. Features suggestive of breast cancer:
Rule out superficial spreading melanoma solid mass +/- stellate features
(+) s-100 antigen in immunostaining (vs asymmetric thickening of breast tissues
paget’s disease which is (+) in clustered microcalcifications
carcinoembryonic antigen presence of fine, stippled calcium in &
immunostaining) around a suspicious lesion is suggestive of
Tx: lumpectomy, mastectomy, or MRM breast cancer; occurs in 50% of
(depending on the extent of involvement and nonpalpable cancers.
the presence of invasive cancer) Mimickers of breast ca mammographically:
Invasive ductal carcinoma radial scars, fibromatosis, granular cell
Occurs in perimenopausal or postmenopausal tumor and fat necrosis (surgical excision is
♀ (5th-6th decade) indicated for these lesions, owing to their
Most common carcinoma presenting as a resemblance to ca)
breast mass % reduction in mortality for women after
poorly defined margin, central stellate screening mammography.
configuration with chalky white or yellow Recommendations:
streaks extending into surrounding breast normal-risk women at 20 yo breast
tissues examination every 3 years
macroscopic/microscopic axillary LN at age 40 yo annual breast examination
metastases in 60% of cases / mammography
false (-)/(+) rate: 10%
Medullary carcinoma
Associated with BRCA1phenotype & DCIS

TOPNOTCH MEDICAL BOARD PREP SURGERY SUPPLEMENT HANDOUT Page 40 of 81


For inquiries visit www.topnotchboardprep.com.ph or email us at topnotchmedicalboardprep@gmail.com
TOPNOTCH MEDICAL BOARD PREP SURGERY SUPPLEMENT HANDOUT - Jules Lopez,MD-MBA,Teddy Carpio,MD-MBA
For inquiries visit www.topnotchboardprep.com.ph or email us at topnotchmedicalboardprep@gmail.com
Ductography HER-2/neu expression is determined for node
Indication: is nipple discharge, (particularly (-) breast ca
when bloody) Trastuzumab: medication for HER-2/neu
Intraductal papillomas are seen as small filling (–)
defects surrounded by contrast media doxorubicin, cyclophosphamide, &
Ca: may appear as irregular masses or as paclitaxel: medication for HER-2/neu (+)
multiple intraluminal filling defects & node (+) breast cancer.

Ultrasonography Advanced local regional breast cancer (stage


Ideal for younger patients (because of IIIa or IIIb)
tendency to have denser breasts – can affect Surgery (MRM) + adjuvant RT + CT
results if mammography is used) (neoadjuvant)
Useful for resolving equivocal mammographic Role of CT: maximize distant disease-free
findings, defining cystic masses, and survival
demonstrating the echogenic qualities of Role of RT: maximize local-regional
specific solid abnormalities. disease-free survival.
breast cysts: well circumscribed, with smooth If stage IIIA ca: neoadjuvant
margins and an echo-free center (preoperative) CT reduce the size of
features of benign breast masses: smooth the primary ca & permit breast-
contours, round or oval shapes, weak internal conserving surgery.
echoes, and well-defined margins.
Features of breast ca: irregular walls but may Figure 29. Treatment pathyway for stage IIIa & IIIb cancer
have smooth margins with acoustic
enhancement.
does not reliably detect lesions that are 1cm.

- Breast cancer staging


Clinically based
tumor size correlates with the presence of axillary
lymph node metastases
The single most important predictor of 10- and
20-year survival rates in breast cancer is the
number of axillary lymph nodes involved with
metastatic disease.

Table 52. TNM breast cancer staging Distant metastases (stage IV)
T N M Not anymore curative but may prolong
T1: <2cm N1: suspicious M1: (+) lung, survival
T2: 2-5 cm mobile liver or bone
T3: >5cm axillary nodes involvement - Breast cancer prognosis
T4: (+) chest N2: matted or
wall & direct fixed axillary
5 year survival rate
skin nodes Stage I: 94%
involvement N3: ipsilateral stage IIA: 85%
internal stage IIB: 70%
mammary stage IIIA: 52%
nodes stage IIIB: 48%
stage IV: 18%
(+) supraclavicular nodes: stage III disease (not
stage IV as formerly classified) - surgical techniques in breast cancer tx
sentinel LN dissection: used to assess the regional
- Treatment for breast cancer LN in women with early breast ca who are
Treatment is dependent on the stage at diagnosis clinically node negative by PE & imaging studies
Early invasive breast cancer (stage I, IIa, IIb) breast conservation therapy (BCT)
Lumpectomy +/- RT (breast conservation sx) if stage 0, I & II, BCT is preferable to total
is an acceptable tx option since survival rates mastectomy ( with equivalent survival rates)
are comparable to total mastectomy. mastectomy and axillary dissection
However, recurrence ↑er in the simple mastectomy: removes all breast
lumpectomy with no RT stage I and II tissue, the nipple-areola complex, skin & level I
breast cancer. LN
CI to breast conservation sx: Modified radical mastectomy: removes all
prior RT to the breast or chest wall breast tissue, the nipple-areola complex, skin,
involved surgical margins or unknown & level I and level II LN.
margin status after re-excision preserves pectoralis major, pectoralis
multicentric disease minor, level III LN & medial (anterior
scleroderma or lupus erythematosus. thoracic) pectoral nerve
If clinically negative nodes but with T1-T2 complications
primary ca perform sentinel LN o most frequent: Seromas beneath the
dissection skin flaps or in the axilla
If (+): perform axillary lymph node o injury to the long thoracic nerve
dissection should be performed. (affects serratus anterior)
Adjuvant chemotherapy is indicated for node- winging of scapula
positive cancers, >1 cm, and node-negative o lymphatic fibrosis painless, slow
cancers of >0.5 cm when adverse prognostic progressive swelling of the involved
features (blood vessel or lymph vessel arm
invasion, high nuclear grade, high histologic o injury to the axillary vein
grade, HER-2/neu overexpression, and sudden painful early postoperative
negative hormone receptor status). swelling of the involved arm (due to
Tamoxifen therapy: women with hormone acute thrombosis as the collateral
receptor (+) cancer that are >1 cm.
TOPNOTCH MEDICAL BOARD PREP SURGERY SUPPLEMENT HANDOUT Page 41 of 81
For inquiries visit www.topnotchboardprep.com.ph or email us at topnotchmedicalboardprep@gmail.com
TOPNOTCH MEDICAL BOARD PREP SURGERY SUPPLEMENT HANDOUT - Jules Lopez,MD-MBA,Teddy Carpio,MD-MBA
For inquiries visit www.topnotchboardprep.com.ph or email us at topnotchmedicalboardprep@gmail.com
channels do not have the chance to - tx:
develop acute and painful) if benign total excision with 2-3 cm margin
o injury to the thoracodorsal if malignant total mastectomy w/o axillary LN
vascular pedicle ischemic loss of dissection; if small wide excision with 2cm
the entire latissimus dorsi flap margin is acceptable
utilized for reconstruction if large mastectomy.
o injury to the medial pectoral Follow up is important due to high local recurrence
pedicle progressive atrophy of rate
the pectoralis muscle
o injury to the 2nd intercostals c. Inflammatory breast cancer
brachiocutaneous nerve - variant of infiltrating ductal ca
hypesthesia of the upper inner - characterized by the skin changes of brawny
aspect of the ipsilateral arm induration, erythema with a raised edge, and
Halsted radical mastectomy: removes all edema or peau d'orange (hence the name
breast tissue and skin, the nipple-areola inflammatory) + breast mass
complex, the pectoralis major and pectoralis - appearance is due to a dermal lymphatic
minor muscles & the level I, II, and III LN. invasion

- Non surgical breast cancer tx


RT REVIEW QUESTIONS a
Adjuvant RT after mastectomy decrease
local recurrence rates but will not prolong 1. a 58 yo woman presents with chronic, erythematous,
survival oozing, eczematoid rash involving the left nipple and
o Indicated for those with high risk for areola. There are no breast masses palpable, and her
local recurrence: large tumors, skin mammogram is normal. Which of the following
involvement, > 4 axillary LN involved recommendations is appropriate?
Chemotx
Adjuvant chemotheraphy a. Referral to a dermatologist
Indicated if node (-) tumor >1cm that are b. Oral vitamin E and topical aloe and lanolin
ER (-) c. Biopsy
Neoadjuvant chemotherapy d. Non allergenic brassiere
Neoadjuvant endocrine therapy e. Standard treatment that includes breast
Tamoxifen: indicated if node (-) tumor conservation
>1cm that are ER (+)
Herceptin: (+) her2/neu Answer: C
This is a case of Paget’s disease of the breast. It is a
- breast cancer in pregnancy case of primary ductal carcinoma that secondarily
occurs in 1 of every 3000 pregnant ♀ invades the epithelium of the nipple and areola. Biopsy
TX: of any chronic nipple rash is mandatory and will show
MRM: 1st & 2nd trimesters of pregnancy the distinctive pagetoid cells. Because of the possible
lumpectomy with axillary node dissection: 3rd invasion of the tumor on the underlying rich lymphatics
trimester of the nipple areolar complex, mastectomy is usually
adjuvant RT: after delivery. indicated. In selected cases, breast conservation
therapies can also be employed.
- male breast cancer
<1% of all breast cancers occur in men 2. If patient with metastatic breast ca is ER (+), which of
preceded by gynecomastia in 20% of men. the following statements are appropriate?
associated with radiation, estrogen tx, testicular
feminizing syndromes, and Klinefelter's syndrome a. Bilateral oophorectomy
(XXY) b. Antiestrogen drugs (tamoxifen)
usual types of cancer: DCIS, infiltrating ductal ca c. Hypophysectomy
Overall, men do worse because of the advanced d. Adrenalectomy
stage of their ca (stage III or IV) at the time of e. Aromatase inhibitor
diagnosis and poorer prognosis. But stage for stage,
the results of treatment are similar to those in Answer: A,B,E
women.
Patients with high ER & PR levels (based on
G. special clinical situations immunohistomchemical stains) have better prognosis
a. nipple discharge compared to those with zero or low levels. The most
suggestive of cancerous lesion: spontaneous, common hormonal manipulation is estrogen withdrawal,
unilateral, localized to a single duct, present in usually with a receptor-blocking agent (tamoxifen).
women 40 years of age, bloody, clear, serous, or However, bilateral oophorectomy in premenopausal
associated with a mass. women is still considered a reasonable option. Surgical
Suggestive of a benign condition: bilateral, hypophysectomy & adrenalectomy were at one point
multiductal in origin, occurs in women 39 years of considered forms of hormonal manipulation, but are
age, or is milky or blue-green. now being replaced by “medical adrenalectomy” in the
Consider prolactin-secreting pituitary form of anastrazole, which inhibit the production of
adenomas ( ↑ serum prolactin levels, Optical adrenal steroids and conversion of androgens to
nerve compression, visual field loss, & estrogens in the adrenal gland and peripherally. The
infertility) aromatase inhibitors are beneficial only in
postmenopausal women.
b. Cystosarcoma phylloides tumor
- Resembles a giant fibroadenoma 3.
a 39 year old woman presents with an ill-defined 2 cm
- Can occur in benign and malignant forms mass in the outer quadrant of her breast. Mammography
- gross appearance: classical leaf-like (phyllodes) shows very dense tissue but no discrete lesion.
appearance; greater cellular activity than fibroadenoma Ultrasound examination shows a solid lesion. An
- metastasis is usually vascular and no axillary LN ultrasound-guided fine needle aspiration (FNA) is
involvement is expected. performed, and the aspirate is plated, fixed, and sent to
the laboratory for cytologic study. A highly cellular
TOPNOTCH MEDICAL BOARD PREP SURGERY SUPPLEMENT HANDOUT Page 42 of 81
For inquiries visit www.topnotchboardprep.com.ph or email us at topnotchmedicalboardprep@gmail.com
TOPNOTCH MEDICAL BOARD PREP SURGERY SUPPLEMENT HANDOUT - Jules Lopez,MD-MBA,Teddy Carpio,MD-MBA
For inquiries visit www.topnotchboardprep.com.ph or email us at topnotchmedicalboardprep@gmail.com
monomorphic pattern is seen, with poorly cohesive - Synchronous neoplasm: a 2nd 1o tumor detected within
intact cells, nuclear “crowding” with a variation in 6 months of the diagnosis of the initial primary lesion
nuclear size, radial dispersion and clumping of the - Metachronous tumor: detection of a 2nd 1o lesion more
chromatin, and prominent nucleoli. Which of the than 6 months after the initial.
following management choices is/are appropriate? - Initial evaluation of patients with primary CA of H&N:
"panendoscopy."
a. MRM
b. Reassuring the patient that the process is B. ANATOMY OF ORAL CAVITY
benign - Borders:
c. Lumpectomy, sentinel lymph node biopsy and Anterior: vermilion border of the lip
irradiation Superior: hard-palate/soft-palate junction
d. Excision of a fibroadenoma with narrow Inferior: circumvallate papillae
margins Lateral: anterior tonsillar pillars
e. Lumpectomy and sentinel lymph node biopsy - The oral cavity includes lips, alveolar ridges, oral
without irradiation tongue, retromolar trigone, floor of mouth, buccal
mucosa, and hard palate.
Answer: A,C - Regional metastatic spread of lesions of the oral cavity
Aspiration biopsy with a 22 gauge needle is an effective is to the lymphatics of the submandibular and the
and safe way of assessing palpable breast lesions. upper jugular region (levels I, II, and III)
Performing the aspiration under ultrasound guidance - Majority of tumors in the oral cavity are squamous
ensures that the lesion has been sampled thoroughly cell carcinoma (>90%)
while under direct vision. Although a smaller volume of
tissue is obtained than the core needle biopsy, FNA C. CANCER OF THE LIP
frequently yields results that may be equal to core - most commonly seen old people (50-70 years old) with
biopsy if read by an experienced cytopathologist. A fair complexion
fibroadenoma would show broad sheets of cohesive - Risk factors: prolonged exposure to sunlight, fair
cells with nuclei that are unfirm in size and shape. The complexion, immunosuppression, and tobacco use.
chromatin pattern would be finely granular and large - Most common location: lower lip (88 to 98%), upper
numbers of bare nuclei would be present. The cytologic lip (2 to 7%) & oral commissure (1%).
findings described in this question is diagnostic of - Predominantly squamous cell CA
carcinoma. Appropriate management, therefore, - Basal cell carcinoma presents more frequently on the
includes either a modified radical mastectomy or upper lip than lower.
lumpectomy, axillary evaluation by either a sentinel - Clinical findings:
lymph node biopsy or an axillary nodal dissection, and ulcerated lesion on the vermilion or cutaneous
whole-breast irradiation. surface.
(+) paresthesia in the area of lesion: mental
nerve involvement.
- unfavorable prognosticating factors: perineural
HEAD and NECK: BENIGN CONDITIONS & TUMORS invasion, involvement of maxilla/mandible, upper lip or
commissure involvement, regional lymphatic
A. Risk factors for tumors of head and neck metastasis, and age younger than 40 years at onset.
B. Anatomy of Oral cavity - primary echelon of nodes at risk is in the
C. Cancer of the Lip submandibular and submental regions
D. Cancer of the Tongue - Tx:
E. Tumors of Alveolus/gingiva T1 & T2 (≤4cm): Surgery = RT
F. Anatomy of pharynx T3 & T4: surgical excision with histologic
G. Tumors of Nasopharynx confirmation of tumor-free margins + postop RT
H. Tumors of Oropharynx Prophylactic supraomohyoid neck dissection
I. Tumors of Hypopharynx/cervical esophagus should be considered for patients with tumors
J. Anatomy Larynx greater than 4 cm, desmoplastic tumor & (+)
K. Benign conditions of the Larynx perineural invasion
L. Laryngeal Carcinoma Realignment of the vermilion border during the
M. Neck and associated conditions reconstruction and preservation of the oral
N. Salivary gland tumors commissure (when possible) are important
O. Thyroid and associated conditions principles in attempting to attain an acceptable
cosmetic result.
- Prognosis is most favorable for all H&N CA

A. RISK FACTORS FOR TUMORS OF HEAD AND NECK D. CANCER OF THE TONGUE
- muscular structure with overlying nonkeratinizing
- tobacco & alcohol: most common preventable risk squamous epithelium.
factors associated with head and neck CA. - Posterior border: circumvallate papillae
- betel nut chewing - Tongue cancer
- reverse smoking Same risk factors with other H&N CA
- HPV 16 and 18. Associated with plummer-vinson syndrome
- UV light exposure (for lip CA) (cervical dysphagia, IDA, atrophic oral mucosa,
- Patients with H&N CA are predisposed to the brittle spoon finger nails)
development of a 2nd tumor within the aerodigestive Clinical findings: ulcerations or as exophytic
tract. masses
presentation of a new-onset dysphagia, The regional lymphatics of the oral cavity are to the
unexplained weight loss, or chronic submandibular space and the upper cervical
cough/hemoptysis must be assessed lymph nodes
thoroughly in patients with a history of prior Involvement of lingual nerve ipsilateral
treatment for a head and neck cancer paresthesias
ex. If (+) primary malignancy of oral cavity Involvement of hypoglossal nerve deviation of
orpharynx secondary malignancy at cervical tongue on protusion + fasciculations atrophy
esophagus; (+) primary malignancy at larynx most common location: lateral and ventral surfaces
secondary malignancy at lungs
TOPNOTCH MEDICAL BOARD PREP SURGERY SUPPLEMENT HANDOUT Page 43 of 81
For inquiries visit www.topnotchboardprep.com.ph or email us at topnotchmedicalboardprep@gmail.com
TOPNOTCH MEDICAL BOARD PREP SURGERY SUPPLEMENT HANDOUT - Jules Lopez,MD-MBA,Teddy Carpio,MD-MBA
For inquiries visit www.topnotchboardprep.com.ph or email us at topnotchmedicalboardprep@gmail.com
if base of the tongue advanced stage and poorer MRI: assess for intracranial and soft-tissue
prognosis extension.
tx: - Tx: chemoradiation
Surgical treatment of small (T1–T2) primary
tumors is wide local excision with either H. TUMORS OF THE OROPHARYNX
primary closure or healing by secondary - Direct extension of tumors from the oropharynx into
intention. these lateral tissues may involve spread into the
If base of tongue Partial glossectomy with parapharyngeal space
supraomohyoid dissection if N0 or MRND if - histology of the majority of tumors in this region is
N(+) squamous cell carcinoma
- (+) asymmetrical enlargement of the tonsils and tongue
E. TUMORS OF ALVEOLUS/GINGIVAL base think lymphoma
- Because of the tight attachment of the alveolar mucosa - Clinical findings: ulcerative lesion, exophytic mass,
to the mandibular and maxillary periosteum, treatment tumor fetor, muffled or "hot potato" voice (large tongue
of lesions of the alveolar mucosa frequently requires base tumors), Dysphagia, weight loss, Referred otalgia,
resection of the underlying bone. (tympanic branches of CN IX & CN X), Trismus
- Diagnosis for alveolar or gingival cancer (involvement of the pterygoid musculature), ipsilateral
Panorex: demonstrate gross cortical invasion or bilateral nontender cervical lymphadenopathy
CT: imaging subtle cortical invasion - LN metastasis from oropharyngeal cancer most
MRI: demonstrates invasion of the medullary commonly occurs in the subdigastric area of level II.
cavity Others - levels III, IV, & V, retropharyngeal &
- Tx for alveolar or gingival cancer parapharyngeal LN.
If minimal bone invasion: mandibular resection Bilateral metastases: seen in tumors originating
If (+) medullary cavity invasion: segmental from the tongue base and soft palate; if found in
mandibulectomy these areas associated with poor survival
- Tx:
F. ANATOMY OF PHARYNX Options: surgery, primary radiation alone, surgery
- three regions: with postoperative radiation, & combined
nasopharynx chemotherapy with radiation therapy.
extends from the posterior nasal septum and If tongue base crossing middling: do total
choana to the skull base glossectomy with possible total laryngectomy
includes fossa of rossenmuller, Eustachian Tumors of the oropharynx tend to be
tube orifices (torus tuberous) and adenoid pad radiosensitive.
bilateral regional metastatic spread in this
area is common I. TUMORS OF THE HYPOPHARYNX/CERVICAL
Lymphadenopathy of the posterior triangle ESOPHAGUS
(level V) of the neck should provoke - Squamous cancers of the hypopharynx frequently
consideration for a nasopharyngeal primary present at an advanced stage, hence are associated with
Oropharynx: poorer survival rates
Includes tonsillar region, base of tongue, soft - Clinical findings: neck mass, muffled or hoarse voice,
palate, and posterolateral pharyngeal walls referred otalgia, progressive dysphagia to solids
Regional lymphatic drainage for liquids, weight loss.
oropharyngeal lesions frequently occurs to the - Invasion of the larynx by direct extension vocal
upper and lower cervical lymphatics cord paralysis (if unilaterally affected) airway
(levels II, III, IV) +Retropharyngeal compromise (if bilaterally affected)
metastatic spread - Diagnosis:
flexible fiber-optic laryngoscopy
hypopharynx.
CT and/or MRI imaging: check for regional
extends from the vallecula to the lower border
metastases (paratracheal and upper mediastinal
of the cricoid posterior and lateral to the
larynx. lymph nodes)
includes pyriform fossa, the postcricoid space, - Tx:
and posterior pharyngeal wall. T1: RT
Regional lymphatic spread is frequently T2 & T3: chemoradiation
bilateral and to the mid- and lower cervical Larynx-preserving surgical procedures: only if the
lymph nodes (levels III, IV) tumor must not involve the apex of the pyriform
sinus, vocal cord mobility must be unimpaired, and
G. TUMORS OF THE NASOPHARYNX the patient must have adequate pulmonary
- Tumors arising in the nasopharynx are usually of reserve.
squamous cell origin Bilateral neck dissection is frequently indicated
given the elevated risk of nodal metastases found
- Most common nasopharyngeal malignancy in the
with these lesions
pediatric age group: lymphoma
- Risk factors for nasopharyngeal carcinoma: area of
habitation & ethnicity (southern China, Africa, J. ANATOMY OF LARYNX:
Alaska, and in Greenland Eskimos.), EBV infection, & - divided into 3 regions:
tobacco use. supraglottis: epiglottis (lined by stratified,
- Symptoms: nonkeratinizing squamous epithelium), false vocal
nasal obstruction, posterior (level V) neck cords (lined by pseudostratified, ciliated
mass, epistaxis, headache, serous otitis media respiratory epithelium), medial surface of the
with hearing loss, and otalgia. aryepiglottic folds, and the roof of the laryngeal
Cranial nerve involvement is indicative of skull ventricles
base extension and advanced disease. has a rich lymphatic network, which
- Lymphatic spread occurs to the posterior cervical, accounts for the high rate of bilateral spread
upper jugular, and retropharyngeal nodes. of metastatic disease
- Bilateral regional metastatic spread is common. glottis: the true vocal cords, anterior and posterior
- Diagnosis for nasopharyngeal CA: commissure, and the floor of the laryngeal
flexible or rigid fiber-optic endoscope ventricle.
CT with contrast: determining bone destruction

TOPNOTCH MEDICAL BOARD PREP SURGERY SUPPLEMENT HANDOUT Page 44 of 81


For inquiries visit www.topnotchboardprep.com.ph or email us at topnotchmedicalboardprep@gmail.com
TOPNOTCH MEDICAL BOARD PREP SURGERY SUPPLEMENT HANDOUT - Jules Lopez,MD-MBA,Teddy Carpio,MD-MBA
For inquiries visit www.topnotchboardprep.com.ph or email us at topnotchmedicalboardprep@gmail.com
Subglottis: extends from below the true vocal Glottic larynx: hoarseness (early; because only a
cords to the cephalic border of the cricoid within small degree of change is required to produce
the airway hoarseness), Airway obstruction (late), Decreased
pseudostratified, ciliated respiratory vocal cord mobility may be caused by direct muscle
epithelium invasion or involvement of the RLN.
Glottic and subglottic lesions: spread to the Subglottic larynx: vocal cord paralysis (usually
cervical chain, paralaryngeal and paratracheal unilateral) and/or airway compromise (are
LN relatively uncommon).
- Lymphatic drainage:
K. BENIGN CONDITIONS OF THE LARYNX Supraglottic larynx: subdigastric and superior
jugular nodes
Recurrent respiratory papillomatosis (RRP) glottic and subglottic larynx: prelaryngeal node
- (+)HPV 6 & 11 (the Delphian node), the paratracheal nodes, and
- larynx is the most frequently involved site the deep cervical nodes
- presents in early childhood, secondary to viral glottic cancers have limited lymphatic access
acquisition during vaginal delivery. regional nodal metastases is low
- Sx: hoarseness, airway compromise - treatment
- Diagnosis: endoscopy early stage glottis & supraglottic cancer: RT
- Tx: operative microlaryngoscopy with excision or laser small glottic cancers: Partial laryngectomy
ablation supraglottic cancers w/o arytenoid or vocal cord
- High tendency to recur extension: supraglottic laryngectomy
advanced tumors with extension : total
Laryngeal granulomas laryngectomy + postop RT
- typically occur in the posterior larynx on the arytenoid Subglottic cancers: total laryngectomy.
mucosa
- risk factors: reflux, voice abuse, chronic throat clearing,
M. NECK AND ASSOCIATED CONDITIONS
endotracheal intubation, and vocal fold paralysis - differential diagnosis of neck masses is dependent on its
- Sx: pain often with swallowing (less commonly: vocal
location and patient’s age
changes)
pediatric age: think congenital or inflammatory
- Dx: fiber-optic laryngoscopy, voice analysis, laryngeal
conditions
electromyography (EMG), and pH probe testing.
adult + risk factors: rule out malignancy
- Tx: voice rest, voice retraining therapy, and antireflux
in terms of location, think about patterns of
therapy.
drainage
Reinke's edema
Lymphatic drainage of the neck is divided into 7 levels.
- located at the superficial lamina propria due to injury to
the capillaries that exist in this layer, with subsequent Figure 30. levels of the neck bearing LN bearing regions
extravasation of fluid.
- Sx: rough, low-pitched voice.
- Risk factors: smoking, laryngopharyngeal reflux,
hypothyroidism, and vocal hyperfunction.

Vocal cord cyst


- may occur under the laryngeal mucosa (in regions
containing mucous-secreting glands)
- Cysts of the vocal cord may be difficult to distinguish
from vocal polyps
- Diagnosis: video stroboscopic laryngoscopy
- Tx: Large cysts of the supraglottic larynx are treated by
marsupialization with cold steel or a CO2 laser.

Vocal cord paralysis 1. Level I: submental & submandibular nodes


- most commonly is iatrogenic (s/p thyroid, - Level Ia: the submental nodes; medial to the anterior
parathyroid, carotid, or cardiothoracic surgeries) belly of the digastric muscle bilaterally, symphysis of
- can be secondary to malignant processes in the lungs, mandible superiorly, and hyoid inferiorly
thyroid, esophagus, thoracic cavity, skull base, or neck. - Level Ib: the submandibular nodes and gland; posterior
- Sx: presents with hoarseness and “breathy” voice to the anterior belly of digastric, anterior to the
If superior laryngeal nerve is affected posterior belly of digastric, and inferior to the body of
demonstrate aspiration secondary to diminished the mandible
supraglottic sensation
left vocal cord is more commonly involved 2. Level II: upper jugular chain nodes
secondary to its longer course of the recurrent - Level IIa: jugulodigastric nodes; deep to
laryngeal nerve (RLN) on that side sternocleidomastoid (SCM) muscle, anterior to the
if anterior surgical approaches to the cervical spine posterior border of the muscle, posterior to the
are performed right RLN is at an increased risk posterior aspect of the posterior belly of digastric,
(courses more laterally to the tracheoesophageal superior to the level of the hyoid, inferior to spinal
complex) accessory nerve (CN XI)
- Level IIb: submuscular recess; superior to spinal
L. LARYNGEAL CARCINOMA accessory nerve to the level of the skull base
- Suspect if with (+) Hx of smoking & complaint of a
change in vocal quality 3. Level III: middle jugular chain nodes
- are primarily squamous cell carcinoma - inferior to the hyoid, superior to the level of the cricoid,
- sx: deep to SCM muscle from posterior border of the
supraglottic larynx: chronic sore throat, muscle to the strap muscles medially
dysphonia ("hot potato" voice), dysphagia, or a
neck mass secondary to regional metastasis, 4.
Level IV: lower jugular chain nodes
Referred otalgia or odynophagia is encountered - inferior to the level of the cricoid, superior to the
with advanced supraglottic cancers. clavicle, deep to SCM muscle from posterior border of
the muscle to the strap muscles medially
TOPNOTCH MEDICAL BOARD PREP SURGERY SUPPLEMENT HANDOUT Page 45 of 81
For inquiries visit www.topnotchboardprep.com.ph or email us at topnotchmedicalboardprep@gmail.com
TOPNOTCH MEDICAL BOARD PREP SURGERY SUPPLEMENT HANDOUT - Jules Lopez,MD-MBA,Teddy Carpio,MD-MBA
For inquiries visit www.topnotchboardprep.com.ph or email us at topnotchmedicalboardprep@gmail.com
jaime is the best!! - Contents of poststyloid compartment: CNs IX to XII, the
carotid space contents, cervical sympathetic chain, fat,
5. Level V: posterior triangle nodes and lymph nodes.
- Level Va: lateral to the posterior aspect of the SCM - Tumors in this space can produce displacement of the
muscle, inferior and medial to splenius capitis and lateral pharyngeal wall medially into the oropharynx
trapezius, superior to the spinal accessory nerve , dysphagia, cranial nerve dysfunction, Horner's
- Level Vb: lateral to the posterior aspect of SCM muscle, syndrome, or vascular compression.
medial to trapezius, inferior to the spinal accessory - Tumors found in the parapharyngeal space:
nerve, superior to the clavicle 40 to 50% of the tumors are of salivary gland
origin
6. Level VI: anterior compartment nodes usually arising anterior to the styloid
- inferior to the hyoid, superior to suprasternal notch, process
medial to the lateral extent of the strap muscles 20 to 25% of tumors are of neurogenic origin
bilaterally such as paragangliomas (glomus vagale, carotid
body tumor), schwannomas, and neurofibroma
7. Level VII: paratracheal nodes usually arising posterior to the styloid
- inferior to the suprasternal notch in the upper process
mediastinum angiography has a role if the tumor in question
is located posterior to the styloid process
Patterns of spread from primary tumor sites: if a paraganglioma is suspected request for
- oral cavity and lip: levels I, II, and III a 24-hour urinary catecholamine
Skip metastases may occur with oral tongue 15% represent LN metastases & 1o lymphoma
cancers such that involvement of nodes in level III
or IV may occur without involvement of higher Benign neck masses
echelon nodes (levels I & II).
- oropharynx, hypopharynx, and larynx: levels II, III, and Thyroglossal duct cyst
IV. - most commonly encountered congenital cervical
- nasopharynx and thyroid: level V nodes in addition to anomalies
the jugular chain nodes. - represents the vestigial remainder of the tract of the
- nasopharynx, soft palate, and lateral and posterior walls descending thyroid gland from the foramen cecum, at
of the oropharynx and hypopharynx: Retropharyngeal the tongue base, into the lower anterior neck during
lymph nodes fetal development.
- hypopharynx, cervical esophagus, and thyroid: - An embryological anomaly wherein there is failure of
paratracheal nodal compartment + upper mediastinum obliteration of the midline pharyngeal diverticulum
nodes (level VII). during thyroid descent
- advanced tumors of the glottis with subglottic spread: - present as a midline or paramedian cystic mass
Delphian node adjacent to the hyoid bone.
- After an upper respiratory infection, the cyst may
Neck dissections: enlarge or become infected.
- Tx: removal of the cyst, the tract, and the central portion
- Radical neck dissection (RND or CRILE method): of the hyoid bone (Sistrunk procedure) + portion of
removes levels I to V of the cervical lymphatics + SCM + the tongue base up to the foramen cecum.
internal jugular vein + CN XI - Check 1st for normal thyroid tissue in the lower neck
area & if ensure that patient is euthyroid
- Modified radical neck dissection (MRND) or - 1% of thyroglossal duct cysts contain cancer (85% is
functional neck dissection: Any modification of the usually papillary)
RND that preserves nonlymphatic structures (i.e., CN XI,
SCM muscle, or internal jugular vein) Congenital branchial cleft anomalies:
Comparable to RND in controlling regional - remnants are derived from the branchial cleft apparatus
metastasis with superior functional results that persists after fetal development.
- Selective neck dissection (SND): any modification of 1st branchial cleft: EAC & parotid gland.
the RND that preserves lymphatic compartments 2nd branchial cleft: courses between the internal
normally removed in RND and external carotid arteries and proceeds into the
Also comparable to RND in controlling regional tonsillar fossa
metastasis with superior functional results 3rd branchial cleft: courses posterior to the
Types: common carotid artery, ending in the pyriform
1. supraomohyoid neck dissection sinus region.
used with oral cavity malignancies
removes lymph nodes in levels I to III
Dermoid cysts
2. lateral neck dissection
- midline masses and represent trapped epithelium
used for laryngeal malignancies
originating from the embryonic closure of the midline.
removes lymph nodes in levels II through IV
3. posterolateral neck dissection
N. SALIVARY GLAND TUMORS
Used for thyroid cancer
- Majority of neoplasms are benign
removes lymph nodes in levels II to V
- Most common gland involved: parotid gland (85% of
- if clinically N(+) necks: do MRND or RND or SND ( only
all salivary gland neoplasms)
if limited N1 disease)
- Most common benign tumor of the salivary gland:
- if (+) extracapsular spread, perineural invasion,
pleomorphic adenoma
vascular invasion, and the presence of multiple involved
- Most common malignant epithelial neoplasm of
lymph nodes are noted neck dissection of choice +
salivary gland: mucoepidermoid carcinoma
Adjuvant RT +/- chemoRT
- 2nd most common malignant epithelial neoplasm of
salivary gland:Adenoid cystic carcinoma, which has a
Parapharyngeal space masses
propensity for neural invasion,
- Is a potential space, shaped like an inverted pyramid
- Risk of malignancy depending on location: minor
spanning the skull base to the hyoid.
salivary gland > submandibular, sublingual >
- Contents of the prestyloid space: parotid, fat, and lymph
parotid gland
nodes.
- Symptoms suggestive of malignancy: pain, paresthesias,
facial nerve weakness, skin invasion, fixation to the
TOPNOTCH MEDICAL BOARD PREP SURGERY SUPPLEMENT HANDOUT Page 46 of 81
For inquiries visit www.topnotchboardprep.com.ph or email us at topnotchmedicalboardprep@gmail.com
TOPNOTCH MEDICAL BOARD PREP SURGERY SUPPLEMENT HANDOUT - Jules Lopez,MD-MBA,Teddy Carpio,MD-MBA
For inquiries visit www.topnotchboardprep.com.ph or email us at topnotchmedicalboardprep@gmail.com
mastoid tip and trismus (invasion of the masseter or - Loop of galen: where the pharyngeal branches of the
pterygoid muscles recurrent laryngeal nerve communicate with the
- Tx: branches of the superior laryngeal nerve. Maybe
If benign neoplasm: do surgical excision injured when dissecting or ligating the superior
If parotid: minimal surgical procedure for thyroid artery
neoplasms of the parotid is superficial - Regional lymph nodes include pretracheal,
parotidectomy with preservation of the facial paratracheal, perithyroidal, RLN, superior mediastinal,
nerve. retropharyngeal, esophageal, and upper, middle, and
Most frequently injured nerve in lower jugular chain nodes.
parotid surgery: greater auricular - Histology:
nerve (not facial nerve!); if transected, the thyroid is divided into lobules that contain 20
will produce numbness of the lower to 40 follicles
portion of the auricle & periauricular skin Each follicle is lined by cuboidal epithelial cells
If the auriculotemporal nerve is C cells or parafollicular cells: secrete the
injured Frey’s syndrome hormone calcitonin.
(postoperative gustatory sweating)
Evaluation of patients with thyroid disease:
If malignant: do en bloc removal of the involved
gland with preservation of all nerves unless Tests of thyroid function:
directly invaded by tumor.
if parotid tumor arising from the lateral 1. Serum TSH
lobe: superficial parotidectomy with - normal: 0.5 – 5μU/mL
ecpreservation of CN VII is indicated. - only test necessary in most patients with thyroid
If the tumor extends into the deep lobe of the nodules that clinically appear euthyroid
parotid: a total parotidectomy with nerve - serum TSH levels reflect the ability of the anterior
preservation is performed pituitary to detect free T4 levels
If submandibular involvement: en bloc - ultrasensitive TSH assay: most sensitive and most
resection of the gland and submental and specific test for the diagnosis of hyperthyroidism and
submandibular lymph nodes. hypothyroidism
Nerves at risk for a submandibular
gland removal: lingual and hypoglossal 2. Total T4
nerve - normal: T4: 55 – 150 nmol/L
Postoperative radiation treatment plays an - Total T4 levels reflect the output from the thyroid gland
important role in the treatment of salivary - Not suitable as a general screening test
malignancies. The presence of - Increased levels seen in hyperthyroid patients, elevated
extraglandular disease, perineural Tg levels secondary to pregnancy,
invasion, direct invasion of regional estrogen/progesterone use or congenital diseases
structures, regional metastasis, and high- - Decreased levels seen in hypothyroid patients,
grade histology are all indications for decreased Tg levels secondary to anabolic steroid use
radiation treatment and protein losing disorders (i.e. nephrotic syndrome)
These individuals maybe euthyroid if their free
O. THYROID AND ASSOCIATED CONDITIONS T4levels are normal

Important facts about Thyroid anatomy: 3. Total T3


- Normal: 1.5 – 3.5 nmol/L
- Total T3 levels reflect peripheral thyroid hormone
- Weight of a normal thyroid gland: 20 g
metabolism
- pyramidal lobe is present in about 50% of patients
- Not suitable as a general screening test
in disorders resulting in thyroid hypertrophy (e.g.,
- Measurement of total T3 levels is important for
Graves' disease, diffuse nodular goiter, or
clinically hyperthyroid patients with normal T4 levels
lymphocytic thyroiditis), the pyramidal lobe
think T3 thyrotoxicosis
usually is enlarged and palpable
- enveloped by a loosely connecting fascia
4. Free T4
- thyroidea ima artery: arises directly from the aorta or
- Normal: 12 – 28 pmol/L
innominate in 1-4% of individuals
- Measures the biologically active hormone
- ligament of berry: posteromedial suspensory
- Not performed as a routine screening test in thyroid
ligament; has a close relationship with the recurrent
disease
laryngeal nerve
- Its utility is in detecting early hyperthyroidism in which
- inferior thyroid artery crosses recurrent laryngeal
total T4 levels maybe normal but free T4 levels are
nerve (RLN) , necessitating identification of the RLN raised
before ligation - Refetoff syndrome: end-organ resistance to T4 wherein
- RLNs innervate all the intrinsic muscles of the free T4 are increased and TSH levels are normal
larynx, except the cricothyroid muscles, which are
innervated by the external laryngeal nerves 5. Free T3
Injury to one RLN: paralysis of the ipsilateral - normal: 3 – 9 pmol/L
vocal cord (lie in the paramedian or the abducted - most useful in the diagnosis of early hyperthyroidism in
position) which levels of free T3 and T4 rise before total T3 and T4
Injury to Bilateral RLN: airway obstruction,
necessitating emergency tracheostomy, or loss of 6. Serum TRH
voice. - used for the evaluation of pituitary TASH secretory
Most common position of right RLN: posterior function
to the inferior thyroid artery
- Injury to the internal branch of the superior 7. Thyroid antibodies
laryngeal nerve aspiration. - include anti-Tg, antimicrosomal, or anti-TPO and TSI
- Injury to the external branch of the superior - anti-Tg & anti-TPO antibody levels: elevated if with
autoimmune thyroiditis
laryngeal nerve inability to tense the ipsilateral
- can be elevated in Hashimoto’s, Graves’, multinodular
vocal cord and hence difficulty "hitting high notes”
goiter & thyroid neoplasms

TOPNOTCH MEDICAL BOARD PREP SURGERY SUPPLEMENT HANDOUT Page 47 of 81


For inquiries visit www.topnotchboardprep.com.ph or email us at topnotchmedicalboardprep@gmail.com
TOPNOTCH MEDICAL BOARD PREP SURGERY SUPPLEMENT HANDOUT - Jules Lopez,MD-MBA,Teddy Carpio,MD-MBA
For inquiries visit www.topnotchboardprep.com.ph or email us at topnotchmedicalboardprep@gmail.com
8. Serum Thyroglobulin (TSH) and radioactive iodine (RAI) ablation followed
- amount is increased in destructive processes of the by hormone replacement.
thyroid gland (thyroiditis) or overactive states (graves’
or toxic multinodular goiter) Ectopic thyroid
- most important use is for the monitoring of - Normal thyroid tissue in aberrant locations (esophagus,
differentiated thyroid cancer recurrence, after total trachea and anterior mediastinum)
thyroidectomy and RAI ablation
elevated anti-Tg antibodies can interfere with the Pyramidal lobe
accuracy of Tg levels and should always be - The distal end of the atrophied thyroglossal duct that
measured when interpreting Tg levels. connects to the thyroid, projecting up to the isthmus,
lying just to the left or right of the midline.
9. Serum Calcitonin
- normal: 0-4 pg/mL basal Benign thyroid disorders
- secreted by C cells
- function: lower serum calcium Hyperthyroidism
- sensitive marker for medullary thyroid cancer
Grave’s disease
Thyroid Imaging: - autoimmune disease with a strong familial
predisposition, female preponderance (5:1), and
1. Radionuclide imaging peak incidence between the ages of 40 to 60 years.
- Most common cause of hyperthyroidism in North
Types: America
1. iodine 123 (123I) - characterized by thyrotoxicosis, diffuse goiter, and
- emits low dose radiation extrathyroidal conditions including ophthalmopathy,
- t 1/2 : 12 – 14 hours dermopathy (pretibial myxedema), thyroid acropachy
- used to image lingual thyroids or goiter - hallmark: thyroid-stimulating antibodies stimulate
the thyrocytes to grow and synthesize excess
2. iodine 131 (131I) thyroid hormone
- higher dose radiation exposure because of longer t ½ - associated with other autoimmune conditions (ex. type I
- t ½ : 8 to 10 days DM, Addison's disease, pernicious anemia, and
- used to screen and treat patients with differentiated myasthenia gravis)
thyroid cancers for metastatic disease - Macroscopic appearance: diffusely and smoothly
** Cold: trap less radioactivity compared to the enlarged, increase in vascularity
surrounding gland, risk of malignancy is higher in cold - Microscopic appearance: hyperplastic gland, minimal
lesions (20%) compared to hot lesions (<5%) colloid present
** Hot: trap more radioactivity, therefore, with - Clinical features:
increased acitivity Hyperthyroid Sx: heat intolerance, ↑sweating,
↑thirst, ↑ weight loss despite adequate caloric
3. technetium Tc 99m pertechnetate (99mTc) intake
- this isotope is taken up by the mitochondria adrenergic excess: palpitations, nervousness,
- shorter t ½, therefore, less radiation exposure fatigue, emotional lability, hyperkinesis, and
- sensitive for nodal metastases tremors
most common GI symptom: diarrhea
4. F-fluorodeoxyglucose PET scan can also develop amenorrhea, decreased fertility,
- used to screen for metastases in patients with and an increased incidence of miscarriages
thyroid cancer in whom other imaging studies are - PE: facial flushing, warm & moist skin, Tachycardia,
negative. atrial fibrillation, fine tremor, muscle wasting, and
- May show clinically occult lesions proximal muscle group weakness with hyperactive
tendon reflexes
2. Ultrasound 50% of patients ophthalmopathy
- excellent noninvasive imaging study of thyroid gland lid lag (von Graefe's sign)
- no radiation exposure spasm of the upper eyelid
- useful for the evaluation of thyroid nodules, revealing the sclera above the corneoscleral
distinguinshing cystic from solid ones, size, limbus (Dalrymple's sign)
multicentricity and cervical lymphadenopathy prominent stare
1 to 2% of patients dermopathy (deposition of
3. CT/MRI glycosaminoglycans leading to thickened skin in
- useful for the evaluation of extent of large, fixed or the pretibial region and dorsum of the foot)
substernal goiters and their relationship to the - Diagnostic: suppressed TSH with or without an
airway and vascular structures elevated free T4 or T3 level.
If eye signs are present, other tests are generally
Developmental abnormalities: not needed.
123I uptake and scan: elevated uptake, with a
Thyroglossal duct cyst (see Neck) diffusely enlarged gland, confirms the diagnosis
- Treatment:
Lingual thyroid Antihyroid drugs:
- failure of the median thyroid anlage to descend propylthiouracil (PTU, 100 to 300 mg three
normally times daily) or methimazole (10 to 30 mg
- may appear as reddish brown mass at the base of the three times daily, then once daily – because it
tongue has a longer half t ½ )
- may be the only thyroid tissue present (hence, if
MOA: inhibits the organic binding of
surgical tx is warranted, evaluation of normal
iodine and the coupling of iodotyrosines
thyroid tissue in the neck must be carried out 1st)
(mediated by TPO).
- Intervention becomes necessary for obstructive
PTU also inhibits the peripheral
symptoms such as choking, dysphagia, airway
conversion of T4 to T3
obstruction, or hemorrhage or if suspicious for
Most patients have improved
malignancy
symptoms in 2 weeks and become
- Tx: administration of exogenous ORAL thyroid
euthyroid in about 6 weeks.
hormone to suppress thyroid-stimulating hormone
TOPNOTCH MEDICAL BOARD PREP SURGERY SUPPLEMENT HANDOUT Page 48 of 81
For inquiries visit www.topnotchboardprep.com.ph or email us at topnotchmedicalboardprep@gmail.com
TOPNOTCH MEDICAL BOARD PREP SURGERY SUPPLEMENT HANDOUT - Jules Lopez,MD-MBA,Teddy Carpio,MD-MBA
For inquiries visit www.topnotchboardprep.com.ph or email us at topnotchmedicalboardprep@gmail.com
IMPORTANT SIDE EFFECT OF PTU: Acute (suppurative) thyroiditis
AGRANULOCYTOSIS - more common in children and often is preceded by an
Propranolol is the most commonly upper respiratory tract infection or otitis media.
prescribed medication in doses of about 20 to - It is characterized by severe neck pain radiating to the
40 mg four times daily for control of jaws or ear, fever, chills, odynophagia, and dysphonia.
adrenergic symptoms - Complications: systemic sepsis, tracheal or esophageal
RAI: most often used in older patients with small rupture, jugular vein thrombosis, laryngeal chondritis,
or moderate-sized goiters, those who have and perichondritis or sympathetic trunk paralysis
relapsed after medical or surgical therapy, and - Streptococcus and anaerobes account for about 70%
those in whom antithyroid drugs or surgery are of cases
contraindicated. - Diagnosis: leukocytosis on blood tests and FNAB for
Absolute CI: women who are pregnant or Gram's stain, culture, and cytology.
breastfeeding - Tx: parenteral antibiotics & drainage of abscesses.
Relative contraindications:
young patients (i.e., especially children Subacute (de quervain’s) thyroiditis
and adolescents) - strong association with the HLA-B35 haplotype.
those with thyroid nodules - Self-limiting painful thyroiditis most commonly occurs
those with ophthalmopathy in 30- to 40-year-old women
Surgery: - characterized by the sudden or gradual onset of neck
Patients should be rendered euthyroid before pain, which may radiate toward the mandible or ear.
operation - History of a preceding upper respiratory tract
Lugol's iodide solution or saturated infection often can be elicited.
potassium iodide generally is administered - The gland is enlarged, exquisitely tender, and firm.
beginning 7 to 10 days preoperatively (three - Diagnosis: TSH is decreased, and Tg, T4 , and T3 levels
drops twice daily) to reduce vascularity of are elevated (during the early phase)
the gland and decrease the risk of - tx: symptomatic; NSAIDs are used for pain relief
precipitating thyroid storm. (steroids may be indicated in more severe cases)
Indications for Total or near-total
thyroidectomy: Patients with coexistent Hashimoto’s thyroiditis (chronic thyroiditis)
thyroid cancer, and those who refuse RAI - autoimmune process leads to destruction of thyrocytes
therapy or have severe ophthalmopathy or by autoantibodies, which lead to complement fixation
have life-threatening reactions to antithyroid and killing by natural killer cells
medications (vasculitis, agranulocytosis, or - Antibodies directed against three main antigens—Tg
liver failure) (60%), TPO (95%), the TSH-R (60%), and, less
commonly, to the sodium/iodine symporter (25%)
- more common in women (male:female ratio 1:10 to 20 )
between the ages of 30 and 50 years old.
REMEMBER: a
- The most common presentation is that of a
minimally or moderately enlarged firm granular
How would you know if there is an undiagnosed
gland discovered on routine PE or the awareness of
hyperthyroid problem intraoperatively?
a painless anterior neck mass
- Gross appearance: mildly enlarged, pale, gray-tan cut
Increased vascularity increased bleeding in a sedated
surface that is granular, nodular, and firm.
patient
- microscopic examination: the gland is diffusely
infiltrated by small lymphocytes and plasma cells and
occasionally shows well-developed germinal centers,
Toxic multinodular goiter
follicles are lined by Hürthle or Askanazy cells
- Symptoms and signs of hyperthyroidism are similar to
- Dx: elevated TSH and the presence of thyroid
Graves' disease, but extrathyroidal manifestations
autoantibodies usually confirm the diagnosis.
are absent
- Tx: Thyroid hormone replacement therapy or surgery
- Possible presence of cervical compressive symptoms
(if with compressive symptoms)
- Diagnosis:
suppressed TSH level and elevated free T4 or T3
Reidel’s thyroiditis
levels.
- characterized by the replacement of all or part of the
RAI uptake also is increased, showing multiple
thyroid parenchyma by fibrous tissue
nodules with increased uptake
- primary autoimmune etiology
Treatment: subtotal thyroidectomy
- occurs predominantly in women between the ages of 30
to 60 years old.
Toxic adenoma (Plummer’s disease)
- presents as a painless, hard anterior neck mass, which
- Hyperthyroidism from a single hyperfunctioning nodule
progresses over weeks to years to produce symptoms
- typically occurs in younger patients
of compression, including dysphagia, dyspnea,
- PE: solitary thyroid nodule without palpable thyroid
choking, and hoarseness.
tissue on the contralateral side
- Can result to hypothyroidism
- RAI: "hot" nodule
- Associated with retroperitoneal fibrosis and sclerosing
- rarely malignant.
mediastinitis
- Tx: Surgery (lobectomy and isthmusectomy on the
- PE: hard, "woody" thyroid gland with fixation to
affected side) is preferred to treat young patients and
surrounding tissues.
those with larger nodules.
- Tx: surgery
Goal of surgery: to decompress the trachea by
Thyroid storm
wedge excision of the thyroid isthmus and to make
- hyperthyroidism + fever, central nervous system
a tissue diagnosis
agitation or depression, cardiovascular dysfunction due
to infection, surgery, trauma or amiodarone
Solitary thyroid nodule
administration.
- History: time of onset ( usually slow and indolent),
- Tx: ICU, Beta blockers, Oxygen supplementation, Fever
change in size, and associated symptoms such as pain,
reduction, fluids, hemodynamic support, PTU,
dysphagia, dyspnea, choking, hoarseness (secondary to
Corticosteroids (to prevent adrenal exhaustion and
malignant involvement of the RLNs)
block hepatic thyroid hormone conversion)
- Risk factors for malignancy:
1. exposure to ionizing radiation
TOPNOTCH MEDICAL BOARD PREP SURGERY SUPPLEMENT HANDOUT Page 49 of 81
For inquiries visit www.topnotchboardprep.com.ph or email us at topnotchmedicalboardprep@gmail.com
TOPNOTCH MEDICAL BOARD PREP SURGERY SUPPLEMENT HANDOUT - Jules Lopez,MD-MBA,Teddy Carpio,MD-MBA
For inquiries visit www.topnotchboardprep.com.ph or email us at topnotchmedicalboardprep@gmail.com
2. (+) FH of thyroid and other malignancies Thyroid cancer
associated with thyroid cancer
3. Men > women Papillary CA
4. Children > adults - 80% of all thyroid malignancies in iodine-sufficient
- PE: nodules that are hard, gritty, or fixed to surrounding areas
structures (if malignant) - predominant thyroid cancer in children and
- Diagnosis: individuals exposed to external radiation.
- occurs more often in women, 2:1
Figure 31. Management of a solitary thyroid nodule - symptoms of locally advanced disease: Dysphagia,
dyspnea, and dysphonia
- Diagnosis:
FNAB of the thyroid mass or lymph node.
Complete neck UTZ: to evaluate the contralateral
lobe and for LN metastases in the central and
lateral neck compartments.
- The most common sites are lungs, followed by bone,
liver, and brain.
- Spread via lymphatic route
- Gross appearance: hard and whitish and remain flat on
sectioning with a blade, macroscopic calcification,
necrosis, or cystic change may be apparent.
- Microscopically:
papillary projections -a mixed pattern of papillary
and follicular structures
pure follicular pattern (follicular variant).
Cells are cuboidal with pale, abundant
cytoplasm, large nuclei that may demonstrate
"grooving," and intranuclear cytoplasmic
FNAB: single most important test in the inclusions (Orphan Annie nuclei)
evaluation of thyroid masses; 1st diagnostic test Psammoma bodies : microscopic, calcified
ordered in a patient with a solitary thyroid deposits representing clumps of sloughed cells,
nodule also may be present
Results: benign – cysts & colloid nodules Multifocality: associated with an increased risk of
(65%), suspicious – follicular or hurthle cell cervical nodal metastases
neoplasms (20%), malignant (5%), and Other variants: tall cell, insular, columnar, diffuse
nondiagnostic (10%) sclerosing, clear cell, trabecular, and poorly
false-positive results is about 1% differentiated types.
false-negative results occur in approximately are generally associated with a worse
3% prognosis.
a negative FNAB does not rule out CA - Tx:
if suspicious result, the diagnosis of If less than 1.5 cm: lobectomy + isthmusectomy
malignancy relies on demonstrating If multicentric: near total or total thyroidectomy
capsular or vascular invasion, features (+) cervical node mets: MDRD
that cannot be determined via FNAB. patients with papillary thyroid CA have an
RAI scan: excellent prognosis with a >95% 10-year
Single, cold, solid nodule malignant survival rate.
Multiply, Hot, cystic benign Age is the most important prognostic factor
Labs: in determining long term survival
TSH: expect euthyroid
Tg levels: useful for patients who have Follicular CA
undergone total thyroidectomy for thyroid - account for 10% of thyroid cancers
cancer & for serial evaluation of patients - occur more commonly in iodine-deficient areas.
undergoing nonoperative management of - Women have a higher incidence of follicular cancer,
thyroid nodules. with a female-to-male ratio of 3:1
Serum calcitonin: obtained in patients with - usually present as solitary thyroid nodules, occasionally
MTC or a family history of MTC or MEN2 with a history of rapid size increase, and long-standing
RET oncogene mutations: All patients with goiter.
MTC should be tested for RET oncogene - In <1% of cases, follicular cancers may be
mutations and have a 24-hour urine collection hyperfunctioning, leading patients to present with signs
with measurement of levels of and symptoms of thyrotoxicosis.
vanillylmandelic acid (VMA), metanephrine, - Spread via hematogenous route, hence their spread
and catecholamine to rule out a coexisting is more distant, than regional
pheochromocytoma. - Most common site of distant metastasis: lung &
Ultrasound: helpful for detecting nonpalpable bone
thyroid nodules, differentiating solid from cystic - Diagnosis:
nodules, and identifying adjacent FNAB shows follicular type must do lobectomy
lymphadenopathy to demonstrate capsular or vascular invasion
- Tx: (criteria for malignancy)
Malignant tumors are treated by thyroidectomy - Microscopically: follicles are present, but the lumen may
Simple thyroid cysts resolve with aspiration; if be devoid of colloid.
persists after 3 attempts at aspiration unilateral - Tx:
thyroid lobectomy is recommended. If follicular lesion thyroid lobectomy +
Lobectomy is recommended for cysts >4 cm in isthmusectomy because at least 80% of these
diameter or complex cysts with solid and cystic patients will have benign adenomas.
components older patients with follicular lesions >4 cm: total
Colloid nodule observe with serial ultrasound thyroidectomy
and Tg measurements. if (+) thyroid CA: do Total thyroidectomy
(+) cervical node mets: MDRD
TOPNOTCH MEDICAL BOARD PREP SURGERY SUPPLEMENT HANDOUT Page 50 of 81
For inquiries visit www.topnotchboardprep.com.ph or email us at topnotchmedicalboardprep@gmail.com
TOPNOTCH MEDICAL BOARD PREP SURGERY SUPPLEMENT HANDOUT - Jules Lopez,MD-MBA,Teddy Carpio,MD-MBA
For inquiries visit www.topnotchboardprep.com.ph or email us at topnotchmedicalboardprep@gmail.com
mortality from follicular thyroid cancer is history, physical examination, raised serum
approximately 15% at 10 years and 30% at 20 calcitonin, or CEA levels, and FNAB cytology of the
years. thyroid mass
Poor long-term prognosis: age over 50 years all new patients with MTC should be screened
old at presentation, tumor size >4 cm, higher for RET point mutations, pheochromocytoma,
tumor grade, marked vascular invasion, and HPT. If (+) carrier, perform total
extrathyroidal invasion, and distant thyroidectomy
metastases at the time of diagnosis. - Tx:
Total thyroidectomy + bilateral central node
Hurtle cell CA dissection (level 6) because of high incidence of
- account for approximately 3% of all thyroid multicentricity
malignancies If (+) pheochromocytoma manage this 1st
- considered to be a subtype of follicular thyroid If with palpable cervical nodes or involved central
cancer. neck nodes: ipsilateral or bilateral MDRD
- Cannot be diagnosed by FNAB. If tumors >1 cm, ipsilateral prophylactic modified
- Microscopically: hurthle cells (variable enlargement, radical neck dissection is recommended because
hyperchromatic nuclei and granular cytoplasm)
>60% of these patients have nodal metastases.
- Difference from follicular CA:
multifocal if ipsilateral nodes are positive do contralateral
bilateral (about 30%) node dissection
usually do not take up RAI (about 5%) - Postoperative Follow-Up: annual measurements of
more likely to metastasize to local nodes (25%) calcitonin and CEA levels
associated with a higher mortality rate (about 20% - Prognosis:
at 10 years) 10-year survival rate is approximately 80% but
higher recurrence rate decreases to 45% in patients with lymph node
- Same management with follicular neoplasms involvement.
- If (+) for hurthle malignancy: perform total best in patients with non-MEN familial MTC,
thyroidectomy + routine central neck (level 6) node followed by those with MEN2A
removal or MDRD when lateral neck nodes are palpable. Prognosis is the worst (survival 35% at 10
years) in patients with MEN2B.
Medullary thyroid CA (MTC)
- accounts for about 5% of thyroid malignancies Anaplastic CA
- arises from the parafollicular or C cells of the thyroid - approximately 1% of all thyroid malignancies
usually located superolaterally in the thyroid lobes - the most aggressive of thyroid malignancies
(usual site of MTC) - Women are more commonly affected
- female-to-male ratio is 1.5:1 - present in the 7th & 8th decade of life
- Most patients present between 50 and 60 years old - Clinical features:
- Most MTCs occur sporadically. Presents as a long-standing neck mass, which
Occur singly rapidly enlarges and may be painful. Associated
unilateral (80%) symptoms such as dysphonia, dysphagia, and
no familial predisposition dyspnea are common.
- approximately 25% occur within the spectrum familial Lymph nodes usually are palpable at presentation.
MTC - MEN2A (pheochromocytoma + parathyroid - Gross appearance: firm and whitish in appearance.
hyperplasia), and MEN2B (pheochromocytoma + - Microscopically: characteristic giant and
neuromas) multinucleated cells. with marked heterogeneity are
due to germline mutations in the RET proto- seen (spindle shaped, polygonal, or large,
oncogene multinucleated cells)
encodes for tyrosine-kinase receptor in the - Tx: if resectable mass surgery will only give small
cell membrane improvement in survival
RET protein is expressed in tissues derived - Prognosis: 6 months
from embryonic nervous and excretory
systems Thyroid Lymphoma
present at a younger age - <1% of thyroid malignancies
multicentric - Most common: non-Hodgkin's B-cell type.
(+) C cell hyperplasia: premalignant lesion - develop in patients with chronic lymphocytic
- clinical features: thyroiditis.
present with a neck mass that may be associated - present with a rapidly enlarging neck mass that is often
with palpable cervical lymphadenopathy (15 to painless.
20%). - may present with acute respiratory distress.
Pain or aching is common - Tx:
dysphagia, dyspnea, or dysphonia – already CT(CHOP—cyclophosphamide, doxorubicin,
invasive vincristine, and prednisone) + RT
diarrhea – indicates metastatic disease (due to Thyroidectomy and nodal resection: for alleviation
increased intestinal motility and impaired of airway obstructive symptoms who do not
intestinal water and electrolyte flux) respond quickly to the above regimens or who
2 to 4% of patients develop Cushing's syndrome as have completed the regimen before diagnosis.
a result of ectopic production of - The overall 5-year survival rate is about 50%; patients
adrenocorticotropic hormone (ACTH) with extrathyroidal disease have markedly lower
- tumor markers: calcitonin (diagnostic, most sensitive survival rates.
tumor marker), CEA (better predictor of prognosis),
calcitonin gene–related peptide, histaminadases, Thyroid surgeries:
prostaglandins E2 and F2 & serotonin.
- Microscopically: Total thyroidectomy: dissection and remonal of all visible
Heterogenous thyroid tissue bilaterally, which usually reveals the entrance of
Cells are polygonal or spindle shaped the recurrent laryngeal nerve as they enter the ligament of berry
infiltrating neoplastic cells separated by collagen
and amyloid Near total thyroidectomy: complete hemithyroidectomy and
presence of amyloid is a diagnostic finding isthmusectomy; most of the contralateral side is removed but a
- Diagnosis: remnant is left to prevent damage to parathyroid glands
TOPNOTCH MEDICAL BOARD PREP SURGERY SUPPLEMENT HANDOUT Page 51 of 81
For inquiries visit www.topnotchboardprep.com.ph or email us at topnotchmedicalboardprep@gmail.com
TOPNOTCH MEDICAL BOARD PREP SURGERY SUPPLEMENT HANDOUT - Jules Lopez,MD-MBA,Teddy Carpio,MD-MBA
For inquiries visit www.topnotchboardprep.com.ph or email us at topnotchmedicalboardprep@gmail.com
should management of this patient at this point
Subtotal thyroidectomy: removes all visible thyroid tissue entail?
except for a rim of thyroid tissue bilaterally to ensure
parathyroid viability and avoids damage to the recurrent a. Complete the operation and evaluate the vocal
laryngeal nerve cords postoperatively via flexible
bronchoscopy
REVIEW QUESTIONS b. Perform intraoperative flexible bronchoscopy
to evaluate vocal cords
1. Regarding salivary gland tumors, which one of the c. Repair the nerve using 8.0 monofilament
following statements is true? sutures
d. None of the above
a. The majority of malignant salivary gland
tumors arise in the parotid gland Answer: D
b. Most parotid neoplasms are malignant If the recurrent laryngeal nerve is injured or
c. Fine needle aspiration biopsy is recommended transected during an otherwise uncomplicated
for all suspected salivary gland malignancies operation, it should be repaired using loupes or an
d. Minor salivary gland tumors occur most operating microscope to visualize the field, and 8.0
commonly in the floor of the mouth or 9.0 monofilament sutures to anstamose the cut
ends of the nerves. There is no role for flexible
Answer: A bronchoscopy either intraoperatively or
The likelihood of a given tumor’s being malignant is postoperatively unless there is uncertainty about
lowest in the parotid gland (approximately 20%), the injury or the function of the contralateral nerve.
followed by the submandibular salivary gland
(approximately 50%) and sublingual glands (nearly
100%). However, because more than 75% of all salivary ESOPHAGUS
gland tumors occur in the parotid gland, the parotid
gland accounts for the majority of the malignant salivary A. Diagnostic tests for esophageal function
gland tumors. The diagnostic evaluation of a salivary B. GERD
gland mass depends on the location and clinical C. Diaphragmatic hernia
scenario. FNAB is not indicated for all parotid tumors, D. Schatzki’s ring
since a tissue diagnosis does not change the treatment E. Scleroderma of esophagus
plan for a patient with a small, mobile mass clearly F. Zenker’s diverticulum
within the gland. When the location is uncertain, the G. Achalasia
history suggests the possibility of metastatic disease, or H. Diffuse and segmental esophageal spasm
the tumor size or location indicates a difficult facial I. Nutcracker esophagus
nerve dissection, FNAB may be helpful. Biopsy, usually a J. Hypertensive LES
punch or excisional biopsy, should be performed for K. Esophageal diverticulum
suspected minor salivary gland tumors, the most L. Esophageal perforation
common site of which is the palate, usually at the M. Mallory weiss syndrome
junction of the hard and aoft palate. Like FNAB, imaging N. Caustic injury
studies (CT or MRI) should be used when they are likely O. Esophageal carcinoma
to augment the clinical assessment of staging and affect
treatment planning.
A. DIAGNOSTIC TESTS FOR ESOPHAGEAL FUNCTION
2. A 40 year-old woman comes to the clinician’s office
with a thyroid mass, which is confirmed on FNA
Tests to detect structural abnormalities
and UTZ to be unilateral, 3.2 cm follicular neoplasm.
She has been completely asymptomatic. What will
1. Barium swallow
the next intervention be?
- 1st diagnostic test in patients with suspected esophageal
disease (with full assessment of stomach and
a. Total thyroidectomy
duodenum)
b. Hemithyroidectomy or isthmusectomy
- can reveal anatomic problems
c. Excisional biopsy
- if patient complains of dysphagia and no obstructing
d. Core-needle biopsy
lesion seen in barium swallow use a barium-
e. Thyroid suppression via T3 or T4 analogues
impregnanted marshmallow, barium-soaked bread or
barium hamburger
Answer: A, B will bring out the functional disturbance in the
The presence of a follicular neoplasm as confirmed esophageal transport that can be missed when
by FNA manadtes further evaluation, since FNA liquid barium is used.
does not provide enough information about tissie
architecture to differentiate between a benign 2. endoscopic evaluation
follicular adenoma and a follicular carcinoma. - endoscopy is indicated in patients complaining of
Vascular or capsular invasion confirms the dysphagia even with a normal radiographic study
presence of carcinoma. The management of small,
Tests to detect functional abnormalities
unilateral follicular lesions is controversial (total
1. manometry
versus hemi-thyroidectomy with frozen section).
- indicated when a motor abnormality of the esophagus is
Hwoever, lesions larger than 4 cm should be
considered on the basis of complaints (dysphagia,
treated with total thyroidectomy, since
odonyphagia, or noncardiac chest pain) and barium
multicentricity becomes more common as tumor
swallow and endoscopy does not show a structural
size increases. Total thyroidectomy also facilitates
abnormality
the effectiveness of postoperative radioactive
- essential tool in preoperative evaluation of patients
iodine, since no residual thyroid tissue remains to
before antireflux surgery
serve as a sink for the radioisotope.
Tests to detect increased exposure to gastric juice
3. During a total thyroidectomy for papillary cancer,
1. 24 hour ambulatory pH monitoring
the clinician observes an intact recurrent laryngeal
- most direct method of measuring increased esophageal
nerve on the right side and a completely transected
exposure to gastric juice (not reflux)
nerve on the left, with both ends in view. What
TOPNOTCH MEDICAL BOARD PREP SURGERY SUPPLEMENT HANDOUT Page 52 of 81
For inquiries visit www.topnotchboardprep.com.ph or email us at topnotchmedicalboardprep@gmail.com
TOPNOTCH MEDICAL BOARD PREP SURGERY SUPPLEMENT HANDOUT - Jules Lopez,MD-MBA,Teddy Carpio,MD-MBA
For inquiries visit www.topnotchboardprep.com.ph or email us at topnotchmedicalboardprep@gmail.com
- sensitivity and specificity of 96% antireflux surgery is an excellent means of
- gold standard for the dioagnosis of GERD long-term control for most patients
one third of all patients with BE present with
2. radiographic exposure of gastroesophageal reflux malignancy
- radiographic demonstration of spontaneous o should undergo surveillance with
regurgitation of barium into the esophagus in the biopsy every 2 years
upright position is a reliable indicator that reflux is o if (+) low grade dysplasia, increase
present frequency to 6 months
- note: failure to see this does not indicate absence of 2. Esophageal Adenocarcinoma
disease Most important etiologic factor in its
development is barrett’s esophagus
B. GERD 3. Respiratory symptoms
- Clinical features: LERD
1. Heartburn: substernal burning-type discomfort, Adult-onset asthma
beginning in the epigastrium and radiating upward. Idiopathic pulmonary fibrosis
-It is often aggravated by meals, spicy or fatty - Treatment:
foods, chocolate, alcohol, and coffee Medical:
-worse in the supine position Uncomplicated GERD: 12 weeks of empiric
1. Regurgitation: effortless return of acid or bitter treatment of antacid
gastric contents into the chest, pharynx, or mouth; Persistent sx: PPIs or H2 antagonists
highly suggestive of foregut pathology A structurally defective LES is the most
-severe at night when supine or when bending over important factor predicting failure of
-secondary to either an incompetent GEJ medical therapy
-explains the associated pulmonary symptoms, They don’t respond to medical therapy
including cough, hoarseness, asthma, and recurrent well; candidates for anti-reflux surgery
pneumonia. Lifestyle changes: elevate the head of the bed
2. Dysphagia: most specific symptom of foregut during sleep; avoid tight-fitting clothing; eat small,
disease; sensation of difficulty in the passage of frequent meals; avoid eating the nighttime meal
food from the mouth to the stomach immediately prior to bedtime; and avoid alcohol,
3. Chest pain coffee, chocolate, and peppermint (which are
- primary cause of GERD: permanent attenuation of the known to reduce resting LES pressure)
collar sling musculature, with a resultant opening of Surgical
the gastric cardia and loss of the high-pressure zone Nissen fundiplication: a abdominal or
as measured with esophageal manometry thoracic approach using a 360 degree
characteristics of a defective sphincter circumferential wrap of the gastric fundus
1. LES with a mean resting pressure of less than 6 Belsey operation: difficult to learn, performed
mmHg through the chestm, involves placement of 2
2. overall sphincter length of <2 cm layers of placating structures between the
3. intra-abdominal sphincter length of <1 cm gastric fundus and lower esophagus with
(most important consideration affecting the subsequent creation of 280 degree anterior
competence of the GE jxn) gastric wrap and posterior approximation of
- diagnosis: the crura
24 hour pH monitoring (gold standard): most Hill operation: approach is through the
sensitive for the detection of reflux abdomen, posterior approximation of the
Endoscopic examination: assessing anatomic crura followed by anchoring of the posterior
damage produced by reflux (esophagitis, ulceration and anterior aspects of the GEJ to the median
and strictures) & for ruling out CA arcuate ligament adjacent to the aorta,
Grading of esophagitis creating a 180 degree gastric wrap
Grade I: small circular nonconfluent erosions Collis gastroplasty: esophageal lengthening
Grade II: presence of linear erosions lined with procedure
granulation tissue that bleeds easily when Angelchik prosthesis: horshe shoe shape
touched silastic device placed around the distal
Grade II: linear erosions coaslesce into a esophagus, keeping this segment in the
circumferential loss of the epithelium; abdomen
cobblestone mucosa
Grade IV: (+) stricture MUST KNOW a
Principles of surgical therapy in reflux disease
**absence of esophagitis above a stricture
suggest chemical induced injury or neoplasm 1. the operation should restore the pressure of the distal
as a cause. esophageal sphincter to a level twice the resting gastric
i. pressure
Manometric studies: rule out motility DO 2. the operation should place an adequate length of the
- Complications: distal esophageal sphincter in the positive-pressure
1. Metaplastic (Barrett's Esophagus) environment of the abdomen by a method that ensures
condition whereby the tubular esophagus is its response to changes in intra-abdominal pressure
lined with columnar epithelium rather than 3. the operation should allow the reconstructed cardia to
squamous epithelium relax on deglutition
occurs in 10 to 15% of patients with GERD 4. the fundoplication should not increase the resistance of
end stage of natural Hx of GERD the relaxed sphincter to a level that exceeds the
hallmark: presence of intestinal goblet peristaltic power of the body of the esophagus
cells in esophageal epithelium (intestinal 5. the operation should ensure that the fundoplication can
metaplasia) be placed in the abdomen without undue tension, and
endoscopically: difficulty visualizing the maintained there by approximating the crura of the
squamocolumnar junction at its normal diaphragm above the repair
location & appearance of redder mucosa than
normally seen in lower esophagus
earliest sign for malignant degeneration: C. DIAPHRAGMATIC HERNIA
severe dysplasia or intramucosal - Types:
adenocarcinoma 1. type I (sliding hernia)
TOPNOTCH MEDICAL BOARD PREP SURGERY SUPPLEMENT HANDOUT Page 53 of 81
For inquiries visit www.topnotchboardprep.com.ph or email us at topnotchmedicalboardprep@gmail.com
TOPNOTCH MEDICAL BOARD PREP SURGERY SUPPLEMENT HANDOUT - Jules Lopez,MD-MBA,Teddy Carpio,MD-MBA
For inquiries visit www.topnotchboardprep.com.ph or email us at topnotchmedicalboardprep@gmail.com
upward dislocation of the cardia in the Manometrically: observation of normal peristalsis
posterior mediastinum in the proximal striated esophagus, with absent
the phrenoesophageal ligament is peristalsis in the distal smooth muscle portion
stretched but intact Barium swallow: dilated, barium-filled esophagus,
most common stomach, and duodenum, or a hiatal hernia with
can evolve into a type III hernia distal esophageal stricture and proximal dilatation
2. type II (rolling or paraesophageal or giant
hiatal hernia) F. ZENKER’S DIVERTICULUM
upward dislocation of the gastric fundus - most common esophageal diverticulum
alongside a normally positioned cardia - classified as false diverticulum
defect in the phrenoesophageal membrane - clinical features: dysphagia associated with the
rare spontaneous regurgitation of undigested food,
more likely to occur in women (4:1) halitosis, weight loss, chronic aspiration and repetitive
3. type III (the combined sliding-rolling or mixed respiratory infection
hernia)
- due to weakness of the cricopharyngeal muscle
upward dislocation of both the cardia and the
gastric fundus; therefore weakness at the Killian’s area
the esophagogastric junction is in the - Diagnosis: Barium swallow (to exclude neoplasia or
mediastinum ulceration)
4. type IV: colon, herniates as well (in some - Treatment:
classifications) Pharyngomyotomy: 2 cm or less
5. intrathoracic abdomen Diverticulectomy/diverticuopexy: >2cm
the end stage of type I and type II hernias
occurs when the whole stomach migrates up G. ACHALASIA
into the chest by rotating 180° around its - Characterized by complete absence of peristalsis in the
longitudinal axis, with the cardia and pylorus esophageal body and failure of LES relaxation
as fixed points - Classic triad of symptoms: dysphagia, regurgitation
- most common complications: and weight loss; also associated with nocturnal asthma
occult GI bleeding from gastritis and foul smelling esophageal contents
ulceration in the herniated portion of the stomach - pathogenesis of achalasia is presumed to be a
gastric volvulus (surgical emergency): or neurogenic degeneration, which is either idiopathic or
Borchardt’s triad of pain, nausea with inability to due to infection.
vomit and inability to pass NGT - Can be caused by T. cruzi which demonstrates
- Diagnosis: destruction of smooth muscle myenteric auerbach’s
Barium esophagogram: for diagnosis of plexus
paraesophageal hiatal hernia - Diagnosis:
Fiber-optic esophagoscopy Barium Esophagogram: dilated esophagus with a
Detection of pouch lined with gastric rugal tapering or other wise known as "bird's beak"
folds lying 2 cm or more above the margins of Manometric studies: failure of the LES to relax,
the diaphragmatic crura (identified by having progressive peristalsis in proximal esophagus (if
the patient sniff) late disease)
- Treatment: surgical Has a 10% chance of developing carcinoma due to
Important principles prolonged mucosal irritation
Reduce the hernia contents - Tx:
After reduction, excise the sac heller’s myotomy (surgical myotomy of the LES)
The use of mesh can reduce recurrence rates goal of surgery: relieve functional obstruction at
of hernia is > than 8 cm the LES

D. SCHATZKI’S RING H. DIFFUSE AND SEGMENTAL ESOPHAGEAL SPASM


- thin submucosal circumferential ring in the lower - characterized by substernal chest pain and/or
esophagus at the squamocolumnar junction, often dysphagia.
associated with a hiatal hernia. - The LES in patients with DES usually shows a normal
- probably an acquired lesion that can lead to stenosis resting pressure and relaxation on swallowing
from chemical-induced injury by pill lodgment in the - Diagnosis:
distal esophagus, or from reflux-induced injury to the Manometric studies: frequent occurrence of
lower esophageal mucosa simultaneous waveforms and multipeaked
- Symptoms: brief episodes of dysphagia during hurried esophageal contractions, which may be of
ingestion of solid foods. abnormally high amplitude or long duration.
- Treatment options: dilation alone, dilation with Esophagogram: corkscrew esophagus or
antireflux measures, antireflux procedure alone, pseudodiverticulosis
incision, and excision of the ring
I. NUTCRACKER ESOPHAGUS
E. SCLERODERMA OF ESOPHAGUS - Other name: supersqueezer esophagus
- Scleroderma is a systemic disease accompanied by - most common of the primary esophageal motility
esophageal abnormalities in approximately 80% of disorders
patients. - characterized by peristaltic esophageal contractions
- onset of the disease is usually in the third or fourth with peak amplitudes greater than two SDs above the
decade of life, occurring twice as frequently in normal values (up to 400 mmHg)
women as in men. - Treatment in these patients should be aimed at the
- Small vessel inflammation appears to be an treatment of GERD
initiating event, with subsequent perivascular
deposition of normal collagen, which may lead to J. HYPERTENSIVE LES
vascular compromise. - This disorder is characterized by an elevated basal
- Muscle ischemia due to perivascular compression has pressure of the LES with normal relaxation and normal
been suggested as a possible mechanism for the motility propulsion in the esophageal body.
abnormality in scleroderma. - Treatment: Myotomy of the LES may be indicated in
- predominant feature at GI tract: smooth muscle patients not responding to medical therapy or dilation.
atrophy.
- Diagnosis: K. ESOPHAGEAL DIVERTICULUM
TOPNOTCH MEDICAL BOARD PREP SURGERY SUPPLEMENT HANDOUT Page 54 of 81
For inquiries visit www.topnotchboardprep.com.ph or email us at topnotchmedicalboardprep@gmail.com
TOPNOTCH MEDICAL BOARD PREP SURGERY SUPPLEMENT HANDOUT - Jules Lopez,MD-MBA,Teddy Carpio,MD-MBA
For inquiries visit www.topnotchboardprep.com.ph or email us at topnotchmedicalboardprep@gmail.com
- Classification: Upper endoscopy: longitudinal fissures in the
Location: proximal, mid, distal mucosa of the herniated stomach as the source of
Pathology bleeding.
Pulsion: motor DO - Treatment:
Traction: inflammatory DO bleeding will stop 90% of the time
- Epiphrenic diverticula: spontaneously with nonoperative management.
terminal 3rd of the thoracic esophagus & are usually Decompression
found adjacent to the diaphragm antiemetics
associated with distal esophageal muscular
hypertrophy, esophageal motility abnormalities, N. CAUSTIC INJURY
and increased luminal pressure - Alkalies vs acids
considered as "pulsion" diverticula Alkalies dissolve tissue, and therefore
classified as false diverticulum (pouch of mucosa penetrate more deeply (more serious)
that is protruding in the wall of the esophagus) acids cause a coagulative necrosis that limits
- Midesophageal or traction diverticula their penetration
Classified as true diverticulum (composed of all - The strength of esophageal contractions varies
layers of the esophageal wall) according to the level of the esophagus
noted in patients who had mediastinal LN - weakest at the striated muscle–smooth muscle interface
involvement with tuberculosis, mediastinal slower clearance allowing caustic substances to
lymphadenopathy, such as pulmonary fungal
remain in contact with the mucosa longer explains
infections (e.g., aspergillosis), lymphoma, or
why the esophagus is preferentially and more severely
sarcoid
affected at this level than in the lower portions.
- Phases of injury:
L. ESOPHAGEAL PERFORATION
acute necrotic phase:
- true emergency.
lasting 1 to 4 days after injury
- It most commonly occurs following diagnostic or
coagulation of intracellular proteins results in
therapeutic procedures (endoscopy).
cell necrosis
- Boerhaave's syndrome: spontaneous perforation,
living tissue surrounding the area of necrosis
- Clinical features: chest pain (very striking and
develops an intense inflammatory reaction.
consistent symptom), fever, tachycardia, subcutaneous
ulceration and granulation phase:
emphysema, dysphagia, dysnea
3 to 5 days after injury
- Diagnosis:
Considered a quiescent period because
contrast esophagogram with water soluble (like
symptoms seem to disappear
gastrografin) medium: (+) extravasation
the superficial necrotic tissue sloughs, leaving
(diagnostic)
an ulcerated, acutely inflamed base, and
chest xray: air or effusion in pleural space,
granulation tissue fills the defect left by the
mediastinal or cervical emphysema
sloughed mucosa.
- treatment:
This phase lasts 10 to 12 days
the incidence of mortality is related to the time
period that the esophagus is the weakest
interval between perforation and treatment; hence
cicatrization and scarring
the key to optimum management is early
begins third week following injury.
diagnosis.
previously formed connective tissue begins to
The most favorable outcome is obtained following
contract, resulting in narrowing of the
primary closure of the perforation within 24
esophagus
hours, resulting in 80 to 90% survival.
characterized by dysphagia
The most common location for the injury is the left
It is during this period that efforts must be
lateral wall of the esophagus, just above the GEJ
made to reduce stricture formation.
- non-operative management
- Clinical features: pain in the mouth and substernal
usually follows an injury occruing during dilation
region, hypersalivation, pain on swallowing, and
of esophageal strictures or pneumatic dilations of
dysphagia, fever (strongly correlated with the
achalasia
presence of an esophageal lesion)
indications
- Diagnosis: early esophagoscopy is advocated to
1. barium swallow must show the perforation to
establish the presence of an esophageal injury
be contained within the mediastinum and
To lessen the chance of perforation, the scope
drain well back into the esophagus
should not be introduced beyond the proximal
2. mild symptoms
esophageal lesion.
3. minimal evidence of clinical sepsis
- Treatment:
- approach:
hyperalimentation Figure 32. algorithm for acute caustic injury
antibiotics
cimetidine: to decreased acid secretion, diminish
pepsin activity

M. MALLORY WEISS SYNDROME


- Longitudinal tear in the mucosa of the GE junction
- Characterized by acute upper GI bleeding caused by
forceful vomiting and/or retching
- Commonly seen in alcoholics
- arterial massive
- mechanism: an acute increase in intra-abdominal
pressure against a closed glottis in a patient with a
hiatal hernia.
- Diagnosis:
requires a high index of suspicion (the pattern of
sudden upper GI bleeding following prolonged
vomiting or retching is indicative).

TOPNOTCH MEDICAL BOARD PREP SURGERY SUPPLEMENT HANDOUT Page 55 of 81


For inquiries visit www.topnotchboardprep.com.ph or email us at topnotchmedicalboardprep@gmail.com
TOPNOTCH MEDICAL BOARD PREP SURGERY SUPPLEMENT HANDOUT - Jules Lopez,MD-MBA,Teddy Carpio,MD-MBA
For inquiries visit www.topnotchboardprep.com.ph or email us at topnotchmedicalboardprep@gmail.com
REVIEW QUESTIONS a
O. ESOPHAGEAL CARCINOMA
- Squamous carcinoma accounts for the majority of 1. a 4 year old child is brought to the ER 15 minutes
esophageal carcinomas worldwide. after accidentally ingesting a drain cleaner. The
- Risk factors: child exhibits a hoarse voice and is stridorous.
nitroso compounds in pickled vegetables and Which of the following apply?
smoked meats
zinc & molybdenum deficiency a. Laryngeal ulceration
smoking (more squamous CA) b. Instillation of vinegar into the stomach
alcohol consumption (more squamous CA) c. Immediate fiberoptic endoscopy
achalasia d. tracheostomy
lye strictures
tylosis (an autosomal dominant disorder Answer: A & D
characterized by hyperkeratosis of the palms and This is a case of caustic ingestion. Since the child
soles) already exhibits laryngeal and epiglottic edema,
human papillomavirus. preservation of the airway must be the priority.
Barrett’s esophagus (more adenocarcinoma) Therefore, endoscopy is deferred.
- Most common presenting symptom: dysphagia
(already a late symptom) 2. A 50 year old healthy man is brought to the ER with
- Diagnosis: retching followed by hematemesis.
barium esophagogram (if with lesion) upper
endoscopy a. Treatment is by balloon tamponade
CT scan of chest and abdomen: delineate the tumor b. Bleeding often stops spontaneously
and detect distant pulmonary or hepatic metastasis c. It is not caused by forceful vomiting
- Characteristics based on tumor location and treatment: d. There is air in the mediastinum
Cervical esophagus (proximal 1/3) e. Diagnosis is not made by endoscopic
Almost always squamous carcinoma examination
Frequently unresectable because of early
invasion of larynx, great vessels or trachea Answer: B
Tx: stereotactic radiation with concomitant This is a case of Mallory-weis tear. The mechanism
chemotherapy is similar to boerhave syndrome (postemetic
thoracic esophagus (middle 1/3) esophageal rupture) in which there is associated
almost always squamous carcinoma with LN perforation and vomiting against a closed cardia. It
metastasis is diagnosed by endoscopic examination, and the
tx: video assisted thoracic surgery (VATS) ± bleeding usually stops spontaneously. Because the
thoracotomy bleeding is arterial, a pressure tamponade (i.e.
distal 1/3 or near/at cardia Sengstaken-blakemore tube) does not help and may
almost always adenocarcinoma lead to esophageal disruption. If bleeding does not
tx: curative resection requires cervical division stop, gastrotomy and oversewing of the bleeding
of esophagus + >50% proximal gastrectomy point is the proper therapy, although nonsurgical
- Staging: alternatives, such as endoscopic injection of
Stage 0: in situ, high-grade dysplasia, no LN mets epinephrine and cautery have been attempted.
Stage I: invaded lamina propria
Stage IIA: invaded esophageal wall but not surrounding 3. After diagnostic esophagoscopy, a patient
structures complains of odynophagia and chest pain, but
Stage IIB: LN (+); primary tumor has only invaded results of water-soluble contrast swallow are
submucosa or muscularis propria negative. Which of the following apply?
Stage III: invaded the adventitia and surrounding
structures (pericardium, pleura and aorta) a. Discharge the patient if ECG is normal.
Stage IV: (+) metastasis b. Use of barium in the chest is devastating
- Clinical factors that indicate advanced stage (and c. Esophageal manometry should be performed
therefore exclude surgery for curative intent): immediately
Horner’s syndrome d. Repeat swallow with barium
Persistent spinal pain Answer: D
Paralysis of diaphragm Chest pain, fever, tachycardia, subcutaneous
Fistula formation emphysema, dysphagia and dyspnea are typical of
Malignant pleural effusion esophageal perforation. Perforation may result
- Treatment: from iatrogenic operations, external trauma,
Surgery: primary esophageal disease or postemetic
Ivor lewis procedure: (“spontaneous”) esophageal hypertension. The
primarily for middle esophageal lesion; all incidence of mortality from esophageal perforation
LNs are removed en bloc with the lesser is clearly related to the time interval between
curvature of the stomach perforation and definitive treatment. Whenever
most radical highest number of perforation is suspected, a contrast study should be
complication rate performed with water-soluble contrast material.
Left thoracoabdominal approach However, if this study does not demonstrate the
excellent exposure of distal esophagus perforation, it should be repeated with barium.
Transhiatal blunt resection: resection of the Although barium is contraindicated in the presence
thoracic esophagus from abdomen with of colonic injuries because of the harmful effects of
subsequent pull-up of stomach and feces and barium, it does not cause a problem in the
esophagogastric anastomosis in the neck chest. Barium is more accurate than water in
Goes against the principle of en-bloc detecting esophageal leakage. Contrast studies are
resection of cancer surgery important not just for verifying esophageal rupture
but also for documenting the level of injury, which
Minimized morbidity and mortality
has important implications for treatment.
compared to the other procedures

TOPNOTCH MEDICAL BOARD PREP SURGERY SUPPLEMENT HANDOUT Page 56 of 81


For inquiries visit www.topnotchboardprep.com.ph or email us at topnotchmedicalboardprep@gmail.com
TOPNOTCH MEDICAL BOARD PREP SURGERY SUPPLEMENT HANDOUT - Jules Lopez,MD-MBA,Teddy Carpio,MD-MBA
For inquiries visit www.topnotchboardprep.com.ph or email us at topnotchmedicalboardprep@gmail.com

STOMACH Double contrast upper GI series


- better then EGD in detecting the ff: diverticula, fistula,
A. Anatomy tortuisity or stricture location, and size of hiatal hernia
B. Diagnostic tests for stomach
C. Peptic ulcer disease 3. CT and MRI
D. Zollinger-Ellison syndrome - is part of routine staging work-up for most patients
E. Gastritis with a malignant gastric tumor
F. stress ulcer
G. Malignant neoplasms of the stomach 4. Gastric secretory analysis
H. Benign gastric neoplasms: polyps - maybe useful in the evaluation of patients with
I. Gastric volvulus hypergastrinemia, including Zollinger-Ellison
J. Postgastrectomy problems syndrome, patients with refractory ulcer or GERD or
recurrent ulcer after operation

5. Tests for Helicobacter pylori


A. ANATOMY
Serologic test for H. pylori
- Stomach is composed of 3 smooth muscle layers:
- a positive test is a presumptive evidence of active
1. Outler longitudinal layer –greater and lesser
infection if the patient has never been treated for H.
curvatures of the stomach
pylori infection
2. Middle circular – pylorus
3. Inner oblique
Histologic examination of antral mucosal biopsy (with
- Majority of parietal cells are in the Body of the
special stains)
stomach
- gold standard for H. pylori
- Largest artery to the stomach is the left gastric artery
(from the celiac trunk) Urease breath test
- Gastric contraction is via the vagus nerve (primarily - standard test for to confirm eradication of H. pylori
due to parasympathetic fibers) post-treatment
1. The vagus nerves forms LARP (left:anterior & - basis: the patient ingests urea labeled with
right:posterior) at the esophageal hiatus as it nonradioactive 13C labeled urea is acted upon by the
descends from the mediastinum urease present in H. pylori converts urea into
2. Anterior branch of vagus: nerves of Laterjet ammonia and carbon dioxide radiolabeled carbon
they send segmental branches to the body of dioxide is excreted from lungs and is detected in
the stomach before they terminate near the expired air.
angularis incisura as the "crow's foot”
3. Posterior branch: Criminal nerve of Grassi
(posterior fundus) MUST KNOW a
easily missed during truncal or highly
selective vagotomy (HSV). H.pylori has the enzyme urease, which converts urea into
- Gastric relaxation is due to CCK, distention of ammonia and bicarbonate, thus creating an environment
duodenum and presence of glucose in the duodenum around the bacteria that buffers the acid secreted by the stomach.

MUST KNOW a

Atonic gastritis and abnormal distention and failure to H. pylori fecal antigen test
empty of the stomach can occur in the postoperative patient - sensitive and specific for active H. pylori infection
due to electrolyte disturbances, hyperglycemia and uremia. - can also be used to confirm cure

Gastric ulcers located in the PYLORUS are associated with C.


PEPTIC ULCER DISEASE
increased gastric production (see below – Type II & III -focal defects in the gastric or duodenal mucosa that
ulcers) extend into the submucosa or deeper
- caused by an imbalance between mucosal defenses and
acid/peptic injury.
- Etiology
B. DIAGNOSTIC TEST FOR STOMACH H. Pylori: associated with both gastric and
duodenal ulcer but is a higher predictor of
1. EGD duonal ulcer formation
- patients with one or more of the alarm symptoms must NSAID – patients taking NSAID and/or aspirin need
undergo immediate upper endoscopy acid suppressing medication if any of the ff are
present: age over 60 yo, hx of PUD, concomitant
Table 53. Alarm symptoms steroid/anticogualant/high dose NSAIDs intake
Alarm symptoms that indicate the need for upper endoscopy Smoking – largest positive predictor of risk (also
Weight loss with alcoholic drinking)
Recurrent vomiting Stress – both physiologic and psychologic stress
Dysphagia Others
Bleeding
More common in Type A personality
Anemia
Sex: duodenal ulcer is twice more common in
males; same incidence between sexes for
- requires an 8 hour fasting
gastric ulcer
- more sensitive than double contrast upper GI series
Blood type:
- most serious complication: esophageal perforation
Type O: duodenal ulcer
2. Radiologic tests Type A: gastric ulcer
- Types of ulcer based on location and
Plain abdominal xray pathophysiology
- helpful in the diagnosis of gastric perforation Duodenal ulcers patients have ↑ daytime and
(pneumoperitoneum) or delayed gastric emptying nocturnal acid secretion, ↑ BAO and MAO, ↑ gastric
(large air-fluid level) emptying compared to gastric ulcer patients
Gastric ulcers patients have variable patterns of
secretion
TOPNOTCH MEDICAL BOARD PREP SURGERY SUPPLEMENT HANDOUT Page 57 of 81
For inquiries visit www.topnotchboardprep.com.ph or email us at topnotchmedicalboardprep@gmail.com
TOPNOTCH MEDICAL BOARD PREP SURGERY SUPPLEMENT HANDOUT - Jules Lopez,MD-MBA,Teddy Carpio,MD-MBA
For inquiries visit www.topnotchboardprep.com.ph or email us at topnotchmedicalboardprep@gmail.com
Figure 33. Modified Johnson classification of gastric ulcer Salvage regimens for patients who fail one of the above initial
regimens
Bismuth subsalicylate 525 mg qid + metronidazole 250 mg 10-14 d
qid + tetracycline 500 mg qid + PPI
PPI + amoxicillin 1000 mg bid + levofloxacin 500 mg daily 10 d

- Indications for surgical treatment for PUD


bleeding
perforation
obstruction
intractability or nonhealing ulcers (with discretion)
- For nonhealing PUD
Rare indication for surgery
Consider possible differentials for nonhealing PUD
first
Surgical treatment is considered in patients with
nonhealing or intractable PUD who have multiple
recurrences, large ulcers (>2 cm), complications
(obstruction, perforation, or hemorrhage), or
suspected malignancy

- Complications of PUD:

Type I: located near the angularis incisura on the lesser Table 55: Comparison of complications of PUD
curvature; usually have normal or decreased acid secretion; most Bleeding PUD Perforation Gastric outlet
common Obstruction
Type II: same with type I but with an associated active or -most common -2nd most common -rare (5% of all PUD
cause of ulcer complication of PUD complications)
quiescent duodenal ulcer; associated with normal or increased
related death -classic symptom: -usually due to
gastric acid secretion -most common patient can duodenal or
Type III: prepyloric ulcer disease; associated with normal or cause of UGIB in remember the exact prepyloric disease
increased gastric acid secretion admitted patients time of onset of -presents with bilous
Type IV: occur near the GE junction, and acid secretion is normal -presents with abdominal pain vomiting, profound
or below normal melena, hematemesis, -presents acute hypochloremic,
Type V: NSAID induced, can occur anywhere in the stomch shock abdomen with metabolic alkalosis
-abdominal pain is peritoneal signs -tx: nasogastric
uncommon (+)pneumoperitoneum suction, IV hydration
MUST KNOW a -tx: acid suppression on upright chest xray and electrolyte
and NPO, transfusion (80% of patients) repletion, and
Curling ulcers: peptic ulcers formed after severe burn injury and endocopic tx -Tx: analgesia, antisecretory
Cushing’s ulcers: peptic ulcers formed after severe brain (electrocautery + epi) antibiotics, isotonic medication, OR
damage for high risk group fluid resuscitation, - rule out
immediate OR pancreatic, gastric &
duodenal CA as a
cause of obstruction
- Pathophysiology, Clinical manifestations, diagnosis
and treatment
MUST KNOW a
Table 54: Comparison between gastric vs duodenal ulcer
Gastric ulcer Duodenal ulcer High risk lesions for massive bleeding (based on location):
Pathophysiology H.pylori, overuse of ↑ acid production & -posterior duodenal ulcer with erosion of gastroduodenal
NSAIDS & steroids H.pylori artery
Clinical Sharp burning pain in Severe epigastric pain -lesser curvature gastric ulcer with erosion of left gastric
manifestation epigastrium shortly 2-3 hours after eating; artery or branch
after eating; nausea, epigastric pain can
vomiting and anorexia also awaken them
from sleep
Diagnosis Endoscopy and biopsy Endoscopy, history, - Surgical options for PUD
(must for all gastric PE, test for H pylori 1.HSV or parietal cell vagotomy or proximal gastric
ulcers to rule out vagotomy
cancer; test for - safe (mortality risk <0.5%) with minimal side effects
H.pylori) - done by severing the vagal nerve supply to the proximal
Best test to confirm eradication of H. pylori: 2/3 of the stomach (where essentially all parietal cells
negative urea breath test are located) & preserves the vagal innervation to the
treatment Triple therapy; PPI, Triple therapy; stop antrum and pylorus and remaining abdominal viscera.
antacids and H2 smoking, alcohol
2. Taylor procedure
blockers consumption
- posterior truncal vagotomy and anterior seromyotomy
- attractive to HSV with similar results
- More than 90% of patients with PUD complain of
3. Vagotomy + drainage (V+D) procedures
abdominal pain (non-radiating, burning in quality &
- Truncal vagotomy dennervates the antrapyloric
epigastriac in location)
mechanism, therefore, some sort of procedure is needed
- Indication for endoscopy in PUD:
to bypass or ablate the pylorus
Any symptomatic patient 45 yo and up
- Types:
Any symptomatic patient regardless of age with
Truncal vagotomy and pyroplasty
alarm symptoms (see table 54) Pyroplasty – useful in patients who require
- Medical treatment for PUD: PPIs are the mainstay of pyloroduodenotomy to deal with the ulcer
therapy for PUD. complication (i.e. posterior bleeding duodenal
ulcer), limited focal a scarring in the pyloric
table 50. treatment regimens for H. pylori
region
PPI + clarithromycin 500 mg BID + amoxicillin 1000 mg BID 10-14 d
Truncal vagotomy and gastrojejunostomy
PPI + clarithromycin 500 mg BID + metronidazole 500 mg 10-14 d
BID
PPI + + amoxicillin 1000 mg BID, then 5d - disadvantage: 10% of significant dumping / diarrhea
PPI + clarithromycin 500 mg BID + tinidazole mg BID
TOPNOTCH MEDICAL BOARD PREP SURGERY SUPPLEMENT HANDOUT Page 58 of 81
For inquiries visit www.topnotchboardprep.com.ph or email us at topnotchmedicalboardprep@gmail.com
TOPNOTCH MEDICAL BOARD PREP SURGERY SUPPLEMENT HANDOUT - Jules Lopez,MD-MBA,Teddy Carpio,MD-MBA
For inquiries visit www.topnotchboardprep.com.ph or email us at topnotchmedicalboardprep@gmail.com
4. Vagotomy and distal gastrectomy Other causes: alcohol, NSAIDs, Crohn's disease,
tuberculosis, and bile reflux
D. ZOLLINGER-ELLISON SYNDROME - Pathophysiology:
- uncontrolled secretion of abnormal amounts of infectious and inflammatory causes: result in
gastrin by a duodenal or pancreatic neuroendocrine immune cell infiltration and cytokine production
tumor (i.e., gastrinoma) leading to excessive which damage mucosal cells.
production of HCl by the parietal cells, further chemical agents (alcohol, aspirin, and bile): disrupt
excacerbating PUD. the mucosal barrier, allowing mucosal damage by
- The inherited or familial form of gastrinoma is back diffusion of luminal hydrogen ions.
associated with multiple endocrine neoplasia type 1
or MEN1 (parathyroid, pituitary, and pancreatic or F. STRESS ULCER
duodenal tumors) - Pathophysiology: due to inadequate gastric mucosal
blood flow during periods of intense physiologic
PHYSIOLOGY a stress.
Gastrin Adequate mucosal blood flow is important to
- produced by antral G cells maintain the mucosal barrier, and to buffer any
- major hormonal stimulant of acid secretion during the back-diffused hydrogen ions. When blood flow is
gastric phase. inadequate, these processes fail and mucosal
- The biologically active pentapeptide sequence at the C- breakdown occurs
terminal end of gastrin is identical to that of CCK
- Luminal peptides and amino acids are the most G. MALIGNANT NEOPLASMS OF THE STOMACH
potent stimulants of gastrin release - The three most common primary malignant gastric
- luminal acid is the most potent inhibitor of gastrin neoplasms are adenocarcinoma (95%), lymphoma
secretion. (4%), and malignant GIST (1%)
- principal mediator of gastrin-stimulated acid
production is histamine from mucosal ECL cells GASTRIC ADENOCARCINOMA
- Epidemiology & etiology
Gastric adenoCA is a disease of the elderly
Risk factors:
- Gastrinoma triangle (or Pasaro’s triangle) : where
Black race: twice more common in blacks
90% of ZES tumors are found
compared to whites
boundaries: jxn of cystic & CBD, confluence of 2nd &
Pernicious anemia
3rd segments of the duodenum and jxn of body and
Blood group A
neck of pancreas
FH of gastric CA
- most common symptoms of ZES are epigastric pain,
Diet: starchy diet high in pickled, salted, or
GERD & diarrhea. Can also be associated with
smoked food, nitrates increases risk
steatorrhea and other symptoms of malabsorption.
H. pylori
- Diagnosis:
Smoking
Fasting gastrin of 1mg/L, BAO >15 mEq/h or >5
EBV infections
mEq/h (if with previous procedure for peptic
Remember: Alcohol has no role in gastric CA
ulcer) are suggestive of ZES
protective factors: aspirin (Yes! Schwartz says so.
Confirmatory test: secretin stimulation test
You don’t believe me? Check p. 927, 9th edition),
(+) secretin stimulation test: paradoxical rise
vitamin C and diet high in fruits and vegetables
in gastrin levels (200 pg/mL or greater) upon
premalignant conditions:
administration of IV bolus of secretin (an
polyps
inhibitor of gastrin)
hyperplastic and adenomas are the
Should also check for serum calcium and PTH
types associated with carcinoma
levels to rule out MEN1.
Preoperative imaging of choice for gastrinoma: inflammatory, hamartomatous and
somatostatin receptor scintigraphy (octreotide heterotropic polyps are considered
scan) benign lesions
Basis: Gastrinoma cells contain type 2 atrophic gastritis: most common
somatostatin receptors that bind the indium- precancerous lesion / precursor of gastric
labeled somatostatin analogue (octreotide) cancer
intestinal metaplasia: can be caused by H.
with high affinity, making imaging with a
pylori
gamma camera possible
- Pathology
Gastric Dysplasia: universal precursor to
PHYSIOLOGY a gastric adenocarcinoma
Somatostatin Early gastric cancer: adenocarcinoma limited to
- produced by D cells located throughout the gastric the mucosa and submucosa of the stomach,
mucosa. regardless of lymph node status.
- major stimulus for somatostatin release is antral 4 forms of gastric cancer (Gross morphology):
acidification 1. Polypoid: bulk of tumor is intraluminal, not ulcerated
- acetylcholine inhibits its release 2. Fungating: bulk of tumor is intraluminal, ulcerated
- Somatostatin effects: inhibits acid secretion from 3. Ulcerative: bulk of tumor is within the stomach wall
parietal cells, inhibits gastrin release from G cells & 4. Scirrhous (linitis plastic): bulk of tumor is within the
decreases histamine release from ECL cells. stomach wall; infiltrate the entire thickness of stomch
- Octreotide is a somatostatin analogue and cover a large surface area, poor prognosis
Location of primary tumor: 40% distal stomach,
30% middle stomach and 30% proximal stomach
- Treatment: Most important prognosticating factors: lymph
Surgical resection of gastrinoma node involvement and depth of tumor invasion
If (+) MEN1, perform parathyroidectomy 1st
before resection of gastrinoma - Clinical manifestations:
PPI for symptomatic relief Most patients diagnosed with gastric CA have
advanced stage III or IV disease
E. GASTRITIS S/Sx:
- Definition: Mucosal inflammation
- Most common cause: H. pylori
TOPNOTCH MEDICAL BOARD PREP SURGERY SUPPLEMENT HANDOUT Page 59 of 81
For inquiries visit www.topnotchboardprep.com.ph or email us at topnotchmedicalboardprep@gmail.com
TOPNOTCH MEDICAL BOARD PREP SURGERY SUPPLEMENT HANDOUT - Jules Lopez,MD-MBA,Teddy Carpio,MD-MBA
For inquiries visit www.topnotchboardprep.com.ph or email us at topnotchmedicalboardprep@gmail.com
weight loss and decreased food intake due
to anorexia and early satiety (most H. BENIGN GASTRIC NEOPLASMS: POLYP (see also
common) premalignant conditions of gastric adenoCA)
Abdominal pain (usually not severe and often - most common benign tumor of the stomach
ignored) - 5 types:
nausea, vomiting, & bloating. 1. Adenomatous: (+) malignant potential; 10-15% of all
Acute GI bleeding (unusual) gastric polyps
chronic occult blood loss (iron deficiency 2. hyperplastic (regenerative): most common gastric
anemia and heme+ stool) polyp (75% of all gastric polyps); occurs in the
Dysphagia: if the tumor involves the cardia of setting of gastritis and has a low malignant potential
the stomach. 3. hamartomatous: benign
Paraneoplastic syndromes - Trousseau's 4. inflammatory: benign
syndrome (thrombophlebitis), acanthosis 5. heterotopic (e.g., ectopic pancreas): benign
nigricans (hyperpigmentation of the axilla and ***Polyps that are symptomatic, >2 cm, large
groin), or peripheral neuropathy can be hyperplastic or adenomatous should be removed,
present. usually by endoscopic snare polypectomy.
Physical examination:
Enlarged Cervical, supraclavicular (on the left I. GASTRIC VOLVULUS
referred to as Virchow's node), and axillary - is a twist of the stomach that usually occurs in
lymph nodes association with a large hiatal hernia or unusually
Sister Joseph’s nodule: palpable umbilical mobile stomach without hiatal hernia.
nodue; pathognomonic for advanced - Gastric volvulus is a chronic condition that can be
disease surprisingly asymptomatic.
Blumer nodes: palpable nodularity in the - Clinical manifestations: abdominal pain and pressure
pouch of douglas; evidence of drop related to the intermittently distending and poorly
metastasis emptying twisted stomach, dyspnea (due to pressure on
the lung), palpitations (due to pressure on the
- Diagnosis pericardium) and dysphagia (pressure on the
Do endoscopy and biopsy esophagus)
Pre-operative staging: abdominal/pelvic CT - Management:
scanning with IV and oral contrast Vomiting and passage of a NGT may relieve
symptoms
- Treatment Gastric infarction is a surgical emergency
Surgery is the only curative treatment for gastric
cancer (radical subtotal gastrectomy) J. POSTGASTRECTOMY PROBLEMS
Goal in resecting gastric adenocarcinoma: grossly
negative margin of at least 5 cm to achieve R0 DUMPING SYNDROME
resection - occurs after bariatric surgery and PUD repair (after
pyloroplasty, pyloromyotomy or distal gastrectomy)
- mechanism: there is accumulation of digested food in
GASTRIC LYMPHOMA the small intestine (or abrupt delivery of hyperosmolar
- stomach is the most common site of primary GI load into the small bowel) leading to circumferential
lymphoma expansion, additional accumulation of fluids emptying
- over 95% are non-Hodgkin's type. from stomach to duodenum and sudden expulsion of
- Most are B-cell type, thought to arise in MALT food to GIT possibly due to ablation of the pylorus or
- MALT lymphomas is a form of NHL arising from the B decreased gastric compliance with accelerated
cells in the marginal zone of MALT emptying of liquids (after highly selective vagotomy)
- Is associated with chronic inflammation due to H. pylori - clinical manifestation: tachycardia, crampy abdominal
- Diligent search for extragstric disease should be done pain and diarrhea, dizziness, lightheadedness,
before giving a diagnosis of primary gastric lymphoma diaphoresis, nausea and vomiting after ingestion of a
- Treatment: chemotx is equivalent to surgery fatty or carbohydrate laden meal
due to sudden shift in electrolytes and fluids
GASTROINTESTINAL STROMAL TUMOR (GIST) combined with increased blood flow to small
- Are submucosal solitary slow growing tumors arising intestine
from interstitial cells of Cajal (ICC) - treatment:
- 2/3 of all GISTs occur in the stomach, occurring decreasing fluid and food intake to small frequent
commonly in the body portions
- defining feature of GISTS is their gain of function avoid fatty and simple sugars
mutation of protooncogene KIT, a receptor tyrosine
kinase (majority of GISTS have activated mutation in
the c-kit protooncogene, which causes KIT to be AFFERENT LIMB OBSTRUCTION (BLIND LOOP
constitutively activated, presumably leading to SYNDROME)
persistence of cellular growth or survival signals) - occurs usually after a Billroth II procedure (distal
- Epithelial cell stromal GIST: most common cell type gastric resection followed by gastrojejunal
arising in the stomach; cellular spindle type is the next anastomosis)
most common; glomus tumor type is seen only in the - location of obstruction: at the limb associated with the
stomach. gastric remnant going to the duodenum
- Markers: (+) c-KIT, a protooncogene; a characteristic - clinical manifestations: severe epigastric pain following
shared with ICC eating, bilous emesis without food
- Diagnosis: endoscopy and biopsy, - treatment: convert Billroth II to roux en-Y gastric
- Mode of metastasis: hematogenous route; most bypass (possible problem: can delay gastric emptying)
common sites: liver and lung
- Treatment:
Wedge resection with clear margins is adequate GASTRIC OUTLET OBSTRUCTION (see complications
surgical treatment of PUD as well)
Imatinib (Gleevec): a chemotherapeutic agent - presents with hypochloremic, hypokalemic
that blocks the activity of the tyrosine kinase
metabolic alkalosis dehydration
product of c-kit, is reserved for metastatic or
- as a compensatory response due to worsening
unresectable GIST. benign gastric neoplasms
dehydration, Na conservation occurs in the kidney,
TOPNOTCH MEDICAL BOARD PREP SURGERY SUPPLEMENT HANDOUT Page 60 of 81
For inquiries visit www.topnotchboardprep.com.ph or email us at topnotchmedicalboardprep@gmail.com
TOPNOTCH MEDICAL BOARD PREP SURGERY SUPPLEMENT HANDOUT - Jules Lopez,MD-MBA,Teddy Carpio,MD-MBA
For inquiries visit www.topnotchboardprep.com.ph or email us at topnotchmedicalboardprep@gmail.com
leading to renal tubular acidosis with subsequent - Layers of the small intestine (from innermost to
aciduria outermost layers): mucosa, submucosa, muscularis
propria and serosa
REVIEW QUESTIONS Contraction of the inner circular layer causes
results in luminal narrowing
1. A patient with a vagotomy and pyloroplasty Contraction of the outer longitudinal layer results
returns with a recurrent ulcer. The best method in bowel shortening
for determining if there was an inadequate Contraction of the muscularis mucosa contribute to
vagotomy performed is mucosal or villus motility (but not peristalsis)
- Mucosal folds: plicae circulares / valvulae
a. Direct vagal stimulation conniventes
b. Stimulated gastric analysis - Peyer’s patches: most commonly located in the ileum
c. Stimulated PPI (pancreatic polypeptide) levels which are aggregates of lymphoid follicles and is a local
d. None of the above – there is no good test to source of IgA
determine inadequate vagotomy - Difference between jejunum and ileum: jejunum has
larger circumference, thicker wall, less fatty
Answer: C mesentery, and longer vasa recta
Historically, gastric analysis was performed most - Calcium is primarily absorbed in the duodenum
commonly to test for the adequacy of vagotomy in through both transcellular transport and paracellular
postoperative patients with recurrent or persistent diffusion.
ulcer. Now this can be done by assessing peripheral
pancreatic polypeptide levels in response to sham
feeding. A 50% increase in pancreatic polypeptide PHYSIOLOGY a
within 30 minutes of sham feeding suggests vagal
integrity. Representative Regulatory Peptides produced in the small
Intestine:
2. Which of the following procedures for PUD has the
highest incidence of postoperative diarrhea? Hormone Source Actions
Somatostatin D Cell Inhibits GI secretion, motility &
a. Graham patch splanchnic perfusion
b. Parietal cell vagotomy Secretin (1st S cell Stimulates exocrine pancreatic
c. Truncal vagotomy and pyloroplasty hormone discovered in secretion; stimulates intestinal
d. Distal gastrectomy without vagotomy the human body)
secretion
Cholecystokinin I cell Stimulates exocrine pancreatic
Answer: C secretion; Stimulates GB
Parietal Truncal Truncal
Cell vagotomy & vagotomy & emptying; Inhibits sphincter of
vagotomy pyloroplasty Antrectomy Oddi contraction
Operative 0 <1 1
Motilin M cell Stimulates intestinal motility
mortality Glucagon-like L cell Stimulates intestinal
rate (%)
Ulcer 5-15 5-15 <2
peptide 2 proliferation
recurrence Peptide YY L cell Inhibits intestinal motility &
(%)
Dumping(%)
secretion

Mild <5 10 10-15

Severe 0 1 1-2
B. SMALL BOWEL OBSTRUCTION
Diarrhea - Epidemiology:
most frequently encountered surgical disorder
Mild <5 25 20 of the small intestine.
Lesions can be described as:
Severe 0 2 1-2
Intraluminal: foreign bodies, gallstones,
meconium
Intramural: tumors, Crohn’s disease
associated inflammatory strictures
Extrinsic: adhesions, hernias, carcinomatosis
- Etiology:
Intra-abdominal adhesions related to prior
SMALL INTESTINE abdominal surgery: most common cause (75%
of cases)
A. Gross Anatomy and Histology Hernias
B. Small bowel obstruction Malignancy: due to extrinsic compression or
C. Ileus & other disorders of intestinal motility invasion by advanced malignancies arising in
D. Crohn’s disease organs other than the small bowel
E. Intestinal fistulas Crohn's disease.
F. Small bowel neoplasms Congenital abnormalities (i.e. midgut volvulus and
G. Radiation enteritis intestinal malrotation) diagnosed at adulthood.
H. Meckel’s diverticulum superior mesenteric artery syndrome: rare;
I. Acquired diverticulum compression of the 3rd portion of the duodenum by
J. Mesenteric Ischemia the superior mesenteric artery as it crosses over
K. Obscure GI bleeding this portion of the duodenum; seen in young
L. Intussuception asthenic individuals who have chronic symptoms
M. Short bowel syndrome suggestive of proximal small bowel obstruction.

- Pathophysiology
Gas (usually from swallowed air) and fluid (from
A. GROSS ANATOMY AND HISTOLOGY swallowed liquids and GI secretions) accumulate
- raison d'être of the GI tract because it is the principle within the intestinal lumen proximal to the site of
site of nutrient digestion and absorption. obstruction intestinal activity ↑ to overcome the
TOPNOTCH MEDICAL BOARD PREP SURGERY SUPPLEMENT HANDOUT Page 61 of 81
For inquiries visit www.topnotchboardprep.com.ph or email us at topnotchmedicalboardprep@gmail.com
TOPNOTCH MEDICAL BOARD PREP SURGERY SUPPLEMENT HANDOUT - Jules Lopez,MD-MBA,Teddy Carpio,MD-MBA
For inquiries visit www.topnotchboardprep.com.ph or email us at topnotchmedicalboardprep@gmail.com
obstruction (seen as colicky pain and diarrhea) If partial SBO, may be approached conservatively
bowel distention ↑ intraluminal and intramural given that there is no fever, tachycardia,
pressures rise intestinal motility is eventually tenderness, or an increase in white cell count
reduced with fewer contractions If intramural (indicates perforation)
pressure becomes high enough impaired most patients with partial small obstruction
intestinal microvascular perfusion intestinal whose symptoms do not improve within 48 hours
ischemia necrosis (strangulated bowel after initiation of nonoperative therapy should
obstruction) undergo surgery.
With obstruction, the luminal flora of the small Obstruction presenting in the early postoperative
bowel (which is usually sterile) changes period (particularly those undergoing pelvic
Translocation of these bacteria to regional lymph surgery, especially colorectal procedures) pose
nodes the greatest risk for developing early
postoperative small bowel obstruction.
Partial SBO: only a portion of the intestinal lumen is obstruction should be considered if Sx of
occluded, allowing passage of some gas and fluid. intestinal obstruction occur after the initial
return of bowel function or if bowel function
Complete SBO: complete occlusion fails to return within the expected 3 to 5
days after abdominal surgery.
Closed loop obstruction: dangerous form of SBO, in Regardless of etiology, the affected intestine should
which a segment of intestine is obstructed both be examined, and nonviable bowel resected.
proximally and distally (e.g., with volvulus). In such Criteria for viability: normal color
cases, the accumulating gas and fluid cannot escape (pinkish), (+)peristalsis, and marginal
either proximally or distally from the obstructed segment, arterial pulsations.
leading to a rapid rise in luminal pressure, and a rapid
progression to strangulation. Ogilvie syndrome
- Distention of the abdomen leading to obstruction
- Clinical presentation - Tends to occur following non-abdominal procedures
Symptoms: colicky abdominal pain, nausea, (i.e. cardiac surgery)
vomiting (a more prominent symptom with - Due to a neurologic dysfunction, electrolyte
proximal obstructions than distal; vomitus is abnormality and ↑age
usually feculent), and obstipation, continued - Treatment: NGT, IV neostigmine, IV atropine (to
passage of flatus and/or stool beyond 6 to 12 hours counter bradycardia as SE of neostigmine), exploratory
after onset of symptoms (more for partial SBO than laparotomy during worst case scenario)
complete SBO)
Signs: abdominal distention (pronounced if the site C. ILEUS & OTHER DISORDERS OF INTESTINAL
of obstruction is distal ileum & absent if the site of MOTILITY
obstruction is in the proximal small intestine), - Ileus is a temporary motility disorder
initially hyperactive bowel sounds (maybe minimal - Postoperative ileus: most frequently implicated
towards the late stages of bowel obstruction) cause of delayed discharge following abdominal
Lab findings: hemoconcentration and electrolyte operations
abnormalities (reflect intravascular volume - Pathophysiology:
depletion) & Mild leukocytosis Common etiologies: abdominal operations,
Features of strangulated SBO: abdominal pain infection and inflammation, electrolyte
often disproportionate to the degree of abnormalities (↓K, ↓&↑Mg, ↓ Na) & drugs
abdominal findings ( suggestive of intestinal (anticholinergics, opiates, phenothiazine, CCB,
ischemia), tachycardia, localized abdominal Tricyclic antidepressants)
tenderness, fever, marked leukocytosis, & acidosis. Proposed mechanisms: surgical stress-induced
sympathetic reflexes, inflammatory response
- Diagnosis mediator release, and anesthetic/analgesic effects
Confirmatory test: abdominal series (radiograph Normal temporal pattern of return of GI
of the abdomen with the patient in a supine position, motility : small intestinal motility (1st 24
upright position &radiograph of the chest with the hours), gastric motility (48 hours) and colonic
patient in an upright position) motility (3 to 5 days)
Sensitivity of abdominal radiographs for - Clinical presentation (usually resembles SBO):
detecting SBO is 70-80% Inability to tolerate liquids and solids by mouth, nausea,
Triad of dilated small bowel loops (>3 cm in and lack of flatus or bowel movements, vomiting,
diameter), air-fluid levels seen on upright abdominal distention & diminished or absent bowel
films, and a paucity of air in the colon is sounds
MOST SPECIFIC - diagnosis: If ileus persists beyond 3 to 5 days
CT scan postoperatively or occurs in the absence of
80 to 90% sensitive abdominal surgery, further investigation is warranted
70 to 90% specific to rule out possibility of mechanical obstruction
Apperance of closed-loop obstruction in
CT: presence of U-shaped or C-shaped dilated CLINICAL PEARLS a
bowel loop associated with a radial
distribution of mesenteric vessels converging Measures to REDUCE postoperative ileus:
toward a torsion point. Intraoperative measures:
Appearance of strangulation in CT: - minimize handling of bowel
thickening of the bowel wall, pneumatosis - laparascopic approach, if possible
intestinalis (air in bowel wall), portal venous - avoid excessive intraoperative fluid administration
gas, mesenteric haziness and poor uptake of IV
contrast into the wall of the affected bowel. Postoperative measures
- early enteral feeding
- Treatment - epidural anesthesia, if indicated
Fluid resuscitation: isotonic replacement - avoid excessive IV fluid administration
Broad spectrum antibiotics - correct electrolyte abnormalities
NGT placement for decompression - consider m-opiod antagonists (
If complete SBO, perform surgery

TOPNOTCH MEDICAL BOARD PREP SURGERY SUPPLEMENT HANDOUT Page 62 of 81


For inquiries visit www.topnotchboardprep.com.ph or email us at topnotchmedicalboardprep@gmail.com
TOPNOTCH MEDICAL BOARD PREP SURGERY SUPPLEMENT HANDOUT - Jules Lopez,MD-MBA,Teddy Carpio,MD-MBA
For inquiries visit www.topnotchboardprep.com.ph or email us at topnotchmedicalboardprep@gmail.com
***Remember, though often recommended, the use of early usually become clinically evident between the 5th &
ambulation and routine NG intubation has NOT been 10th postop
demonstrated to be associated with earlier resolution of initial signs: Fever, leukocytosis, prolonged ileus,
postoperative ileus. abdominal tenderness, and wound infection
(+) drainage of enteric material through the
abdominal wound or through existing drains:
D. INFLAMMATORY BOWEL SYNDROME: CROHN’S associated with intra-abdominal abscesses.
DISEASE VS ULCERATIVE COLITIS - Diagnosis
CT scan: most useful initial test
table 56. Inflammatory bowel syndrome small bowel series or enteroclysis examination: can
Crohn’s disease Ulcerative colitis be obtained to demonstrate the fistula's site of
description chronic, idiopathic Chronic inflammatory origin in the bowel.
transmural inflammatory disease affecting only the Fistulogram: greater sensitivity in localizing the
disease with a propensity colonic mucosa and
fistula origin.
to affect the distal ileum submucosa
Etiology & -more common in Higher chance of leading to - Treatment
epidemiology Ashkenazi jews & colorectal cancer Should follow orderly steps (done to maximize
caucasaians, females, has spontaneous closure)
a bimodal age Stabilization: fluid & electrolyte resuscitation,
distribution (3rd & 6th TPN, antibiotics,
decade), (+) strong Investigation: see diagnosis
pattern of family Decision to do perform surgery or do
inheritance, smokers &
conservative treatment
higher Socio-eco status
Pathology Focal transmural Inflammation is limited to Surgeons usually do 2 to 3 months of
inflammation, mucosa and submucosa conservative therapy before
aphthous ulcers only; lead pipe colon considering surgical intervention.
(earliest lesion of (lacks haustral markings); This approach is based on evidence that
Crohn’s), no granulomas 90% of fistulas that are going to close,
non casseating close within a 5-week interval
granulomas, - Definitive management: surgery (if failure of
cobblestoning,
spontaneous closure during time period or with
**fat wrapping
(encroachment of complications and risk factors)
mesenteric fat onto the - rehabilitation
serosal surface of the
bowel): pathognomonic MUST KNOW a
of crohn’s
Remember FRIEND (factors that inhibit spontaneous closure of
spares rectum,can occur fistulas):
anywhere in the GI tract, Primarily affects the colon
skip lesions, targets & rectum and is continous;
Foreign body within the fistula tract
terminal ileum can also manifest with Radiation enteritis
backwash ileitis Infection/Inflammation at the fistula origin Epithelialization of
fistula, the fistula tract
Neoplasm at the fistula origin
Distal obstruction of the intestine
S/Sx Inisiduous onset with bloody diarrhea and
waxing and waning crampy abdominal pain.
course of abdominal pain Proctitis may produce
(usually RLQ), nonbloody tenesmus; can proceed to
F. SMALL BOWEL NEOPLASMS
diarrhea & weight loss; fulminant colitis and toxic - Adenomas are the most common benign neoplasm of
megacolon the small intestine
(+) extraintestinal - Most common location for primary adenocarcinoma
manifestation: arthritis, and adenomas of the small bowel is DUODENUM
uveitis, iritis, eythema (EXCEPT in patient’s with Crohn’s disease, which is
nodosum, pyoderma found mostly in the ileum)
gangrenosum, primary - Primary small bowel cancers are rare; 1.1 to 2.4% of
sclerosing cholangitis,
nephrolithiasis
all GI malignancies
Diagnosis Endoscopy (skip lesions, Endoscopy & proctoscopy Adenocarcinomas: 35 - 50%
cobblestoning, abscess (earliest manifestation is Carcinoid tumors: 20 to 40%
formation and fistulas); mucosal edema; mucosal Lymphomas: 10 to 15 %
histology demonstrate friability ; ulceration; (+) GISTs: most common location is STOMACH (60-
granulomas; (+) (pANCA) Pus and mucus) 70%), 2nd most common location is small intestine
and anti–Saccharomyces (25-35%)
cerevisiae antibody - Pathophysiology: proposed explanations for the low
(ASCA
frequency of small intestinal neoplasms
Treatment Sulfasalazine + steroids; Similar to Crohn’s;
surgery if unresponsive colectomy after 15 years of
dilution of environmental carcinogens in the liquid
to aggressive medical Tx symptoms chyme present in the SI lumen
rapid transit of chime (limiting the contact time
E. INTESTINAL FISTULAS between carcinogens and the intestinal mucosa)
- abnormal communication between two epithelialized relatively low concentration of bacteria in small
surfaces intestinal chime (therefore, low concentration of
- can be internal (within GI tract or adjacent organs)or carcinogenic products of bacterial metabolism)
external (with communication to external environment) mucosal protection by secretory IgA and
- Kinds: hydrolases such as benzpyrene hydroxylase
low output fistulas - drain less than 200 mL of render carcinogens less active
fluid/day efficient epithelial cellular apoptotic mechanisms
high output fistulas - drain more than 500 mL of that serve to eliminate clones harboring genetic
fluid/day mutations.
- 80% of enterocutaneous fistulas are due to
iatrogenic complications
- Clinical presentation
TOPNOTCH MEDICAL BOARD PREP SURGERY SUPPLEMENT HANDOUT Page 63 of 81
For inquiries visit www.topnotchboardprep.com.ph or email us at topnotchmedicalboardprep@gmail.com
TOPNOTCH MEDICAL BOARD PREP SURGERY SUPPLEMENT HANDOUT - Jules Lopez,MD-MBA,Teddy Carpio,MD-MBA
For inquiries visit www.topnotchboardprep.com.ph or email us at topnotchmedicalboardprep@gmail.com
- Clinical presentation - Littre’s hernia: Meckel's diverticula found in an
Partial SBO is the most common mode of inguinal or femoral hernia sacs; when incarcerated,
presentation can cause intestinal obstruction
Only becomes symptomatic when it becomes large - Clinical presentation
- Diagnosis: Because of the absent or nonspecific most common presentations associated with
symptoms associated with most small intestinal symptomatic Meckel's diverticula: bleeding (most
neoplasms, these lesions rarely are diagnosed common in pediatric age), intestinal
preoperatively obstruction (most common in adults), and
- Treatment: surgical resection diverticulitis
- Diagnosis
G. RADIATION ENTERITIS Usually discovered incidentally on radiographic
- An undesired side effect of radiation therapy is imaging, during endoscopy, or at the time of
radiation-induced injury to the small intestine surgery.
- The SI is susceptible to radiation-induced injury CT scan: low sensitivity and specificity
because it has a high rate of rapidly proliferating Enteroclysis: has 75% accuracy but u not
cells compared to the other portions of the GI tract applicable during acute presentations
- Pathophysiology Radionuclide scans (99mTc-pertechnetate): positive
principal mechanism of radiation-induced cell only when the diverticulum contains associated
death is believed to be apoptosis resulting from ectopic gastric mucosa that is capable of uptake of
free-radical–induced breaks in double-stranded the tracer
DNA - Treatment: surgical
The intensity of injury is related to the dose of diverticulectomy
radiation administered If the indication for diverticulectomy is bleeding,
- Pathology segmental resection of ileum that includes both the
acute injury: villus blunting, dense infiltrate of diverticulum and the adjacent ileal peptic ulcer
leukocytes and plasma cells within the crypts, should be performed.
mucosal sloughing, ulceration, and hemorrhage Segmental ileal resection may also be necessary if
chronic injury: progressive occlusive vasculitis that the diverticulum contains a tumor or if the base of
leads to chronic ischemia and fibrosis that affects the diverticulum is inflamed or perforated.
all layers of the intestinal wall, rather than the The management of incidentally found
mucosa alone leading to strictures, abscesses, (asymptomatic) Meckel's diverticula is
and fistulas controversial.
- Clinical presentation
Acute: nausea, vomiting, diarrhea, and crampy I. ACQUIRED DIVERTICULUM
abdominal pain. - Considered as false diverticula (because their walls
Chronic: becomes evident within 2 years of consist of mucosa and submucosa but lack a complete
radiation administration, most commonly presents muscularis)
with partial small bowel obstruction with - more common in the duodenum, near the ampulla
nausea, vomiting, intermittent abdominal (periampullary, juxtapapillary, or peri-Vaterian
distention, crampy abdominal pain, and weight loss diverticula)
The terminal ileum is the most frequently affected - Diverticula in the jejunum tend to be large and
segment accompanied by multiple other diverticula, whereas
- Diagnosis those in the ileum tend to be small and solitary.
Enteroclysis: most accurate imaging test for - Pathophysiology
diagnosing chronic radiation enteritis, Due to acquired abnormalities of intestinal smooth
CT scan findings are neither very sensitive nor muscle or dysregulated motility leading to
specific for chronic radiation enteritis; should be herniation of mucosa and submucosa through
obtained to rule out the presence of recurrent weakened areas of muscularis.
cancer (because of overlap in clinical - Clinical presentation
manifestations) Acquired diverticula are asymptomatic unless
- Treatment: supportive therapy associated complications arise
Complications (6 to 10% of patients): intestinal
H. MECKEL’S DIVERTICULUM obstruction, diverticulitis, hemorrhage,
- most prevalent congenital anomaly of the GI tract perforation, and malabsorption.
- considered a true diverticula - Diagnosis
- location is usually found in the ileum within 100 cm of Most acquired diverticula are discovered
the ileocecal valve incidentally on radiographic imaging, during
- 60% of Meckel's diverticula contain heterotopic endoscopy, or at the time of surgery.
mucosa (most common: gastric mucosa – 60%; Enteroclysis is the most sensitive test for detecting
others: Pancreatic acini, Brunner's glands, pancreatic jejunoileal diverticula
islets, colonic mucosa, endometriosis, and hepatobiliary - Treatment
tissues). If asymptomatic, observe
If (+)complications, such as bleeding and
MUST KNOW a diverticulitis: segmental intestinal resection for
rule of TWOs of Meckel’s diverticulum: diverticula located in the jejunum or ileum.

2% prevalence J. MESENTERIC ISCHEMIA


2:1 female predominance
location 2 ft proximal to the ileocecal valve in adults Table 57: comparison of acute vs chronic mesenteric ischemia
one half of those who are symptomatic are under 2 years of age Acute mesenteric ischemia Chronic mesenteric ischemia
Causes: - results from atherosclerotic
-arterial embolus: most lesions in the main splanchnic
common cause; most common arteries (celiac, superior
- Pathophysiology source: heart; most common mesenteric, and inferior
Failure of the the omphalomesenteric location: SMA mesenteric arteries
(vitelline) duct to undergo obliteration during -arterial thrombosis: occur in -rarely leads to infarction
the 8th week of gestation proximal mesenteric arteries
-vasospasm (nonocclusive -Postprandial abdominal pain
mesenteric ischemia): is the most prevalent
diagnosed in critically ill symptom, producing a
TOPNOTCH MEDICAL BOARD PREP SURGERY SUPPLEMENT HANDOUT Page 64 of 81
For inquiries visit www.topnotchboardprep.com.ph or email us at topnotchmedicalboardprep@gmail.com
TOPNOTCH MEDICAL BOARD PREP SURGERY SUPPLEMENT HANDOUT - Jules Lopez,MD-MBA,Teddy Carpio,MD-MBA
For inquiries visit www.topnotchboardprep.com.ph or email us at topnotchmedicalboardprep@gmail.com
Acute mesenteric ischemia Chronic mesenteric ischemia events in cobalmin absorption are poorly
patients receiving vasopressor characteristic aversion to food characterized, but the intact complex probably
agents. ("food-fear") and weight loss enters enterocytes through translocation. Because
-venous thrombosis: involves (can be mistaken as a symptom each of these steps is necessary for cobalamin
the superior mesenteric vein of malignancy)
in 95% of cases; associated with
assimilation, gastric resection, gastric bypass and
heritable or acquired ileal resection can each result in Vitamin B 12
coagulation DO insufficiency.

Golden period: 3 hours – 2. Which of the following is the LAST to recover


intestinal sloughing; 6 hours: from postoperative ileus?
full thickness intestinal
infarction a. Stomach
Hallmark of acute mesenteric
b. Small Bowel
ischemia: Severe abdominal c. Colon
pain, out of proportion to the d. NONE of the above – recovery is simultaneous
degree of tenderness on
examination Answer: C
The return of normal motility generally follows a
K. INTUSSUCEPTION characteristic temporal sequence, with small
- refers to a condition where one segment of the intestine intestinal motility returning to normal within the 1st
becomes drawn in to the lumen of the proximal 24 hours after laparotomy and gastric and colonic
segment of the bowel motility returning to normal by 48 hours and 3 to 5
- usually is seen in the pediatric population days, respectively. Because small bowel motility is
- Adult intussusceptions are rare; usually with distinct returned before colonic and gastric motility, listening
pathologic lead point (which can be malignant) for bowel sounds is not a reliable indicator that ileus
- commonly present with a history of intermittent has fully resolved. Functional evidence of coordinated
abdominal pain and signs and symptoms of bowel GI motility in the form of passing flatus or bowel
obstruction movement is a more useful indicator.
- CT scan: diagnostic of choice
Finding: "target sign"
- Treatment: surgical resection of the involved segment
and the lead point, which needs to undergo pathologic APPENDIX
evaluation to rule out an underlying malignancy.
A. Anatomy
L. SHORT BOWEL SYNDROME B. Acute appendicitis
- presence of less than 200 cm of residual small bowel in C. Appendiceal tumors - Carcinoid
adult patients
- usually acquired (s/p intestinal resection)
- can result to malabsorptive symptoms: diarrhea,
A. ANATOMY
dehydration, and malnutrition,
- Function: immunologic organ; a GALT tossie that
- most common etiologies:
secrete immunoglobulins
adults: acute mesenteric ischemia, malignancy, and
- The base of the appendix can always be found at the
Crohn's disease
confluence of the taenia
pedia: intestinal atresias, volvulus, and necrotizing
- Tip of the appendix varies: retroceccal (most common),
enterocolitis
pelvic, subcecal, preileal, or right pericolic position
- Pathophysiology
***The location of the tip of the appendix
Normal: Resection of less than 50% of the small
determine the location of physical findings
intestine is generally well tolerated.
produced by irritation of parietal peritoneum
Symptomatic when greater than 50 to 80% of the
- the luminal capacity if the normal appendix is 0.1 cc.
small intestine has been resected.
secretion of as little as 0.5 cc of fluid distal to the
Malabsorption in patients who have undergone
obstruction raises intraluminal pressure to 60 cm H20.
massive small bowel resection is exacerbated by a
characteristic hypergastrinemia-associated
B.
ACUTE APPENDICITIS
gastric acid hypersecretion that persists for 1 to
-
Etiology
2 years postoperatively
Fecalith: most common cause of appendiceal
- Treatment:
obstruction; usually in adults
TPN & enteral nutrition
Lymphoid hyperplasia: most common cause of
Pharmacotherapy
appendiceal obstruction in patients of pediatric age
REVIEW QUESTIONS - Pathogenesis: Luminal obstruction bacterial
overgrowth, active mucosal secretion & increased
1. Vitamin B12 deficiency can occur after luminal pressure
- Natural history: rarely resolves; ultimately lead to
a. Gastrectomy gangrene and perforation
b. Gastric bypass - Clinical presentation
c. Ileal resection Hallmark of appendicitis: poorly localized pain (due
d. ALL OF THE ABOVE to distension stimulates visceral afferent pain
fibers) that is referred to the periumbilical region
Answer: D (via autonomic innervations) Right lower
Vitamin B12 (cobalamin) malabsorption can result quadrant pain and tenderness (via somatic
from a variety of surgical manipulations. The innervations due to involvement of the parietal
vitamin is initially bound by saliva-derived R peritoneum)
protein. In the duodenum, R protein is hydrolyzed Vomiting usually follows abdominal pain
by pancreatic enzymes, allowing free cobalamin to Anorexia is a constant symptom; if not present,
bind to gastric parietal cell-derived intrinsic factor. question diagnosis; usually precedes abdominal
The cobalamin-intrinsic factor complex is able to pain
escape hydrolysis by pancreatic enzymes, allowing Variations in the anatomic location of the
it to reach the terminal ileum, which expresses appendiceal tip account for the different
specific receptors for intrinsic factor. Subsequent manifestation of the abdominal pain
TOPNOTCH MEDICAL BOARD PREP SURGERY SUPPLEMENT HANDOUT Page 65 of 81
For inquiries visit www.topnotchboardprep.com.ph or email us at topnotchmedicalboardprep@gmail.com
TOPNOTCH MEDICAL BOARD PREP SURGERY SUPPLEMENT HANDOUT - Jules Lopez,MD-MBA,Teddy Carpio,MD-MBA
For inquiries visit www.topnotchboardprep.com.ph or email us at topnotchmedicalboardprep@gmail.com
Retrocecal: flank or back pain
Pelvis: findings maybe absent; painful DRE AP IN THE ELDERLY
exam - Usually with atypical presentation: fever, leukocytosis
- PE maneuvers: and RLQ pain maybe minimal or absent
Dumphy’s sign: increased pain during coughing or - Have 60-90% rupture rate
jumping - The atrophic omentum is less capable of walling off a
Rovsing’s sign: pain in the RLQ when pressure is perforated appendix diffuse peritonitis or distant
applied on the LLQ; this suggests peritoneal intra-abdominal abscess are expected
irritation - If patient is older than 60yo, always rule out cancer
Psoas sign: pain on extension of the right thigh because the definitive treatment for that is right
with the patient lying on the left side; this is due to hemicolectomy (if affecting the cecum)
the pain elicited by the stretched psoas muscle
irritating the inflamed appendix AP IN PREGNANCY
Obturator’s sign: pain with passive rotation of the - Most common surgical emergency in pregnancy
flexed right hip; suggests that the inflamed tip lies - In pregnancy, the gravid uterus pushes the appendix
in the appendix superiorly and the tip medially
- Diagnosis: usually based on history and physical - Most consistent sign of AP in pregnant women: pain in
examination even in the absence of laboratories and the right side of the abdomen
imaging - Common occurrence of abdominal pain, nausea and
Laboratory findings leukocytosis in the normal course of pregnancy makes
1. Moderate leukocytosis with diagnosis difficult
polymorphonuclear predominance (if above - Most cases occur during 2nd trimester
18,000 – suspect abscess or perforation) - Fetal mortality is 2-8.5%; increases to 35% with
2. Can also have normal WBC count (1/3 of rupture
patients)
3. Minimal albuminuria, (+) WBC and RBC in AP IN HIV OR AIDS PATIENTS
urine if appendix is retrocecal - Similar presentation to non-infected patients
4. Anemia in elderly should raise suspicion of - Risk of appendiceal rupture is higher for these patients
carcinoma of the cecum - DDx: CMV enteritis, typhilitis, fungal, protozoal and
Imaging mycobacterial infections
1. Plain abdominal films: fecalith, localized
ileus on the RLQ & loss of peritoneal fat Treatment: appendectomy
strip
2. UTZ: tubular, immobile and C. APPENDICEAL TUMORS - CARCINOID
noncompressible appendix,wall thickness of - most common location is appendix (50%), ileum (25%)
>2mm and outer diameter of at least 6 mm then rectum (20%)
are indicative of appendicitis - ileal carcinoid has the highest potential for metastasis
3. CT scan: thickened by more than 5- (arounf 35%) vs appendiceal carcinoid which has
7mm&fluid filled, periappendiceal lowest potential for metastasis (3%)
inflammation along with fat stranding, fluid - Gross appearance: small, firm, circumscribed, yellowish
collections & phlegmons tumor
- Differential diagnosis - Treatment:
Acute mesenteric adenitis: associated with URTI <2cm at distal appendix: appendectomy
and presents with a more diffused abdominal painl >2cm or at base: right hemicolectomy
also with generalized lymphadenopathy with
lymphocytosis REVIEW QUESTIONS
Acute gastroenteritis: crampy abdominal pain
with watery stools, nausea and vomiting 1. A patient suspected of having appendicitis underwent
Diverticulitis: of cecum or perforated carcinoma exploration, Crohn’s disease was found. Which of the
of cecum is difficult to distinguish clinically from following are true?
appendicitis; diagnosis is usually done a. The normal appendix should always be
intraoperatively removed
Epiploic apendagitis: infarction of the appendage b. All grossly involved bowel, including the
due to torsion; pain shift is unusal and patient is appendix, should be resected.
usually not ill c. An inflamed appendix, cecum and terminal
Gynecologic conditions: ruptured ectopic ileum, should be resected
pregnancy, PID, ruptured grafian follicle, twisted d. Perforated bowel and advanced Crohn’s
ovarian cyst disease with obstruction should be resected.
Intussusception: patient’s age, type of pain,
palpable mass in the lower quadrant and passage Answer: D
of currant jelly stool may help with diagnosis; If a normal appendix is found at the time of
barium enema offers both diagnostic and laparotomy, other causes should be sought. If
therapeutic option for intussusception. Crohn’s disease is encountered and the cecum and
base of the appendix are normal, an appendectomy
- Special conditions: should be performed. If the base is involved with
Crohn’s disease and the appendix is normal,
AP IN THE YOUNG appendectomy should not be performed. If the
- Diagnostic accuracy in these age group is lower finding of Crohn’s disease is uncomplicated by
compared to adults due to imprecise history and perforation or obstruction, ileal resection is not
nonspecific abdominal complaints indicated. However, in the case of perforation or
- Hx: vomiting, fever and diarrhea are common Crohn’s disease with obstruction, the involved
complaints bowel should be resected.
- PE: abdominal distention, maximal tenderness in the
right lower quadrant, the inability to walk or walking
with a limp, and pain with percussion, coughing, and
hopping
- Gangrene and rupture are more common in these age
group because of delays in diagnosis

TOPNOTCH MEDICAL BOARD PREP SURGERY SUPPLEMENT HANDOUT Page 66 of 81


For inquiries visit www.topnotchboardprep.com.ph or email us at topnotchmedicalboardprep@gmail.com
TOPNOTCH MEDICAL BOARD PREP SURGERY SUPPLEMENT HANDOUT - Jules Lopez,MD-MBA,Teddy Carpio,MD-MBA
For inquiries visit www.topnotchboardprep.com.ph or email us at topnotchmedicalboardprep@gmail.com

COLON, RECTUM, ANUS abdominal air, bowel gas patterns suggestive of small or
large bowel obstruction and volvulus
A. Embryology - contrast studies are useful for evaluationg obstructive
B. Diagnostic evaluation of Colon, Rectum and Anus symptoms, delineating fistulous tracts and diagnosing
C. Evaluation of Common symptoms small perforations or anastomotic leaks.
D. Diverticular disease - Gastrografin (water soluble contrast agent) is
E. Colorectal adenocarcinoma recommended if perforation or leak is suspected
F. Colorectal carcinoid tumors - Double contrast barium enema is 70-90% sensitive
G. Anal intraepithelial neoplasia (Bowen’s disease) for the detection of mass lesions greater than 1 cm in
H. Volvulus diameter
I. Colonic pseudoobstruction (Ogilvie’s syndrome) If a small, non obstructing lesion is considered,
J. Hemorrhoids colonoscopy is the preferred imaging modality of
K. Anal fissure choice
L. Anorectal abcess
M. Fistula in ano 2. CT
- the utility of CT is in the detection of extraluminal
disease, such as intra-abdominal abscesses and
pericoloic inflammation and in staging colorectal
carcinoma (because of its sensitivity in detecting
A. EMBRYOLOGY
hepatic metastasis)
- Embryonic GI tract begins developing during 4th week
REMEMBER: a standard CT scan is INSENSITIVE
of gestation
for detection of intraluminal lesions
Table 57. Embryology of GI tract - If considering a perforation / anastomotic leak:
FOREGUT MIDGUT HINDGUT check for extravasation of oral or rectal contrast
Esophagus, stomach, small intestine, distal transverse - Bowel wall thickening / mesenteric stranding
pancreas, liver, ascending colon, colon, descending suggests inflammatory bowel disease, enteritis/colitis
duodenum and proximal colon, rectum, and or ischemia
transverse colon proximal anus
Celiac artery SMA IMA 3. MRI
***distal anus is derived from the ectoderm; BS: internal pudendal - the main use of MRI in colorectal DO is in the evaluation
artey of pelvic lesions
- more sensitive than CT for detecting bony involvement
- The colon has 5 distinct layers: mucosa, submucosa,
or pelvic sidewall extension of rectal tumors.
inner circular muscle, outer longitudinal muscle, and
- Can be useful in the detection and delineation of
serosa
complex fistulas in ano.
MUST KNOW
4. Positron Emission Tomography
- useful for imaging tissues with high levels of anaerobic
Most common bacterium within the colon is B. fragilis
glycolysis, such as malignant tumors
followed by E. coli and Enterococcus sp.
- F-fluorodeoxyglucose is injected as a tracer its
metabolism results in positron emission
- Used as an adjunct to CT in staging colorectal cancer
B. DIAGNOSTIC EVALUATION OF COLON, RECTUM AND
ANUS
5. Angiography
- used for the detection of bleeding within the colon or
ENDOSCOPY:
small bowel
- to visualize hemorrhage angiographically, bleeding
1.Anoscopy
must be relatively brisk (0.5 to 1 cc per minute)
- useful instrument for the examination of the anal canal
- if extravasation of contrast is identified, infusion with
- not attempted without anesthesia if patient complains
vasopressin or angiographic embolization can be
of severe perianal pain and does not tolerate digital
therapeutic.
rectal examination
2. Proctoscopy
6. Endorectal and Endoanal UTZ
- useful for the examination of the rectum and distal
- is used primarily to evaluate the depth of invasion of
sigmoid colon
neoplastic lesions in the rectum and detecting
- can be both therapeutic and diagnostic
sphincter defects & outlining complex anal fistulas
- length: 25 cm
- normal rectal wall can be seen as a 5 layer structure
- 15-19 mm diameter proctoscope is useful for diagnostic
- UTZ can reliably differentiate benign polyps from
examination
invasive tumors based upon the integrity of the
- useful for polypectomy, electrocoagulation, detorsion of
submucosal layer.
sigmoid volvulus
- Accuracy in detecting depth of mural invasion is 81-
94%
3. Flexible sigmoidoscopy and colonoscopy
- provides excellent visualization of colon and rectum PHYSIOLOGIC AND PELVIC FLOOR INVESTIGATIONS:
- can be both diagnostic and therapeutic useful in the evaluation of patients with incontinence,
- length: constipation, rectal prolapse, obstructed defecation and
60 cm: sigmoidoscope other pelvic floor disorders
100-160 cm: colonoscope
- full length insertion: 1. Manometry
may allow visualization as far as splenic flexure: -
procedure: pressure-sensitive catheter is placed in the
sigmoidoscope lower rectum catheter is withdrawn through the anal
may allow visualization as far as terminal ileum: canal and pressures recorded
colonoscope - values:
resting pressure (normal: 40-80 mmHg): reflects
IMAGING:
the function of the internal anal sphincter
Squeeze pressure (normal: 40-80 mmHg above
1. Plain x-ray and contrast studies
resting pressure): maximum voluntary contraction
- plain x-rays of abdomen (upright, supine and
pressure minus resting pressure, reflects the
diaphragmatic views) are useful for detecting free intra-
function of the external anal sphincter
TOPNOTCH MEDICAL BOARD PREP SURGERY SUPPLEMENT HANDOUT Page 67 of 81
For inquiries visit www.topnotchboardprep.com.ph or email us at topnotchmedicalboardprep@gmail.com
TOPNOTCH MEDICAL BOARD PREP SURGERY SUPPLEMENT HANDOUT - Jules Lopez,MD-MBA,Teddy Carpio,MD-MBA
For inquiries visit www.topnotchboardprep.com.ph or email us at topnotchmedicalboardprep@gmail.com
High pressure zone (normal: 2-4 cm): estimates
the kength of the anal canal 4. Diarrhea and irritable bowel syndrome
- Absence of rectoanal inhibitory reflex is - Acute bloody diarrhea and pain can be due to infection
characteristic of Hirschsprung’s disease or inflammation
- chronic diarrhea has a more difficult diagnostic
2. Neurophysiology dilemma since causes are myriad (ulcerative colitis,
- Neurophysiologic testing assesses function of the crohn’s colitis, malabsorption, short-gut syndrome,
pudendal nerve and recruitment of puborectalis muscle carcinoid, islet cell tumors, etc)
fibers
5. Incontinence
LABORATORY STUDIES: - ranges in severity from occasional leakage of gas and
liquid stool to daily loss of solid stool
1. Fecal Occult Blood testing (FOBT) - can be neurogenic or anatomic
- is a screening test for colonic neoplasms in Neurogenic: diseases of CNS, spinal cord, pudendal
asymptomatic, average-risk individuals nerve injury
- occult bleeding from any GI source will produce a Anatomic: congenital abnormalities, procidentia,
positive result (since it is a non specific test for overflow incontinence secondary to impaction,
peroxidase contained in hemoglobin) neoplasm or trauma
- any positive FOBT mandates further investigation, - Most common traumatic cause of incontinence is injury
usually by colonoscopy to the anal sphincter during vaginal delivery

2. Stool studies D. DIVERTICULAR DISEASE


- helpful in the evaluation of etiology of diarrhea - Diverticulosis: presence of diverticula without
- wet mount examination: (+) fecal leukocytes indicate inflammation.
colonic inflammation or presence of invasive organisms Common in patients with low fiber diet
(such as E. coli or Shigella) Majority tend to occur after the age of 85
- Sudan red stain to stool sample: to evaluate steatorrhea Sigmoid colon: most common site of
diverticulosis
3. CEA: tumor marker Common symptom: massive LGIB
- elevated in 60-90% of patients with colorectal cancer; - Diverticulitis: inflammation and infection associated
however, not an effective screening agent for colorectal with diverticula.
CA 5% of complicated diverticulitis develop a fistula to
- serial monitoring used after curative-intent surgery is an adjacent organ. Most common of which is a
done colovesical fistula.
C. EVALUATION OF COMMON SYMPTOMS - More common is false diverticula type
1. Pain Only mucosa and muscularis mucosa have
- abdominal pain related to colon and rectum can result herniated (also called pulsion diverticula) through
from obstruction (inflammatory or neoplastic), the colonic wall, in between taenia coli (area of
inflammation, perforation or ischemia weakness)
- pelvic pain can originate from distal colon and rectum - True diverticula, comprises all layers of the bowel, is
or adjacent urogenital structures congenital and rare
tenesmus: due to proctitis or from rectal or - Clinical manifestations of diverticulitis:
rectrorectal mass Uncomplicated diverticulitis: left-sided abdominal
cyclical pain + menses + rectal bleeding: pain, with or without fever, mass and leukocytosis
endometriosis complicated diverticulitis: abscess, obstruction,
- anorectal pain is most often secondary to anal fissure, diffuse peritonitis (free perforation), or fistulas
perirectal abscess and/or fistula, or a thrombosed (most common is colovesical fistula)
hemorrhoids - Hinchey staging for complicated diverticulitis
Stage I: colonic inflammation with an associated
2. Lower GI bleeding pericolic abscess
- first goal in managing a patient with GI hemorrhage: Stage II: colonic inflammation with a retroperitoneal or
ADEQUATE RESUSCITATION pelvic abscess
- insert NGT (1st test that should be performed) since the Stage III: purulent peritonitis
most common cause of bleeding can either be Stage IV: fecal peritonitis.
esophageal, gastric or duodenal - Diagnosis: CT scan
if (+) return of bile suggests that bleeding is Appears as pericolic soft tissue stranding,
distal to the ligament of Treitz colonic wall thickening, and/or phlegmon
if bloody/non-bile secretions suggests an upper - Treatment:
intestinal source; do EGD right away Uncomplicated diverticulitis: outpatient therapy
- technetium-99-tagged RBC scan: highly sensitive (as with broad-spectrum oral antibiotics for 7-10 days
little as 0.1 cc/hour of bleeding can be detected); & low-residue diet; failure to improve within 48-72
however location is imprecise perform angiography hours indicates abscess formation
to localize bleeding If 2nd episode of uncomplicated diverticulitis or 1st
- if sharp, knife-like pain + bright red rectal bleeding with episode of complicated diverticulitis: elective
bowel movements anal fissure sigmoid colectomy is recommended
- if painless, bright red rectal bleeding secondary to Small abscesses (<2 cm diameter) may be treated
bowel movements internal hemorrhoids with parenteral antibiotics.
Larger abscesses are best treated with CT-guided
3. Constipation and obstructed defecation percutaneous drainage
- A very common problem
- rule out an underlying metabolic, pharmacologic, E. COLORECTAL ADENOCARCINOMA
endocrine, psychological and neurologic causes first - Most common malignancy of the GIT
before work up - Risk factors:
- a stricture or mass lesion should be excluded by 1. Aging: dominant risk factor for colorectal cancer;
colonoscopy or barium enema incidence increases after 50 yo
- once other causes have been ruled out, perform transit 2. Known FH of cancer: accounts for 20% of cases
studies 3. Diet: high in animal fat and low in fiber
- Medical management is the mainstay of treatment for 4. Inflammatory bowel syndrome
constipation (High fiber, increase fluids & laxatives) 5. Cigarette smoking
TOPNOTCH MEDICAL BOARD PREP SURGERY SUPPLEMENT HANDOUT Page 68 of 81
For inquiries visit www.topnotchboardprep.com.ph or email us at topnotchmedicalboardprep@gmail.com
TOPNOTCH MEDICAL BOARD PREP SURGERY SUPPLEMENT HANDOUT - Jules Lopez,MD-MBA,Teddy Carpio,MD-MBA
For inquiries visit www.topnotchboardprep.com.ph or email us at topnotchmedicalboardprep@gmail.com
Cowden syndrome: autosomal dominant
- Pathogenesis: genetic defects disorder with hamartomas of all three
embryonal cell layers; Facial
Figure 34. Schematic diagram showing progression from normal trichilemmomas, breast cancer, thyroid
colonic mucosa to carcinoma of colon disease, and GI polyps are typical of the
syndrome.
Neoplastic polyps
Tubular adenomas: most common type of
neoplastic polyps; asymptomatic,
pedunculated, less than 1 cm in size and occur
APC commonly in the rectosigmoid region
- Tumor suppressor gene located at chromosome 5 <1cm: rare chance for malignany
- Function: the protein product of APC is for maintain 1-2cm: 10% chance for malignancy
cellular adhesions and suppressing neoplastic growth >2 cm: 30% chance for malignancy
- APC inactivation leads to sporadic colorectal cancer Tubulovillous adenoma: mixed; 22% chance
- Mutated in individuals with familial adenomatous for malignancy
polyposis (FAP) Villous adenoma: sessile, larger and
symptomatic, can cause malignancy by 40-
K-RAS 50%; highest risk of cancer
- Proto-oncogene located in chromosome 12 Sessile adenomas are more likely to harbor
- Function: encodes for plasma membrane based protein malignancy compared to pedunculated ones
involved in transduction of growth and differential
signals - Inherited colorectal carcinoma
- Mutation leads to uncontrolled cell division Familial adenomatosis polyposis
- K-RAS activation leads to colorectal cancer rare autosomal dominant condition accounts
- for only about 1% of all colorectal
adenocarcinomas.
DCC Due to mutation in the APC gene, located on
- Tumor suppressor gene located at chromosome 18 chromosome 5q
- Function: encodes for a protein responsible for cell to Clinically, patients develop hundreds to
cell contact thousands of adenomatous polyps shortly
- Loss of DCC gene (or inactivation) tend to present in after puberty.
more advanced carcinomas The lifetime risk of colorectal cancer in FAP
- Present in 70% of colorectal carcinomas patients approaches 100% by age 50 years.
Flexible sigmoidoscopy of first-degree
P53 relatives of FAP patients beginning at age 10
- Tumor suppressor gene located at chromosome 17 to 15 years
- Mutations of this gene are the most common genetic FAP may be associated with extraintestinal
abnormality found in human cancer genes manifestations such as congenital hypertrophy
- Function: crucial for initiating apoptosis in cells with of the retinal pigmented epithelium, desmoid
irreparable genetic damage. tumors, epidermoid cysts, mandibular
- Mutations in p53 are present in 75% of colorectal osteomas (Gardner's syndrome), and central
cancers nervous system tumors (Turcot's syndrome).
- p53 inactivation leads to colorectal cancer HNPCC or Lynch syndrome
Rare autosomal dominant disorder arising
- Polyps: from errors in mismatch repair
Non-neoplastic polyps (no malignant potential) is characterized by the development of
Hyperplastic polyp: most common type of colorectal carcinoma at an early age (average
all polyps; usually small, multiple and sessile; age: 40 to 45 years).
occur frequently in the rectosigmoid region The risk of synchronous or metachronous
Pseudopolyps (or inflammatory polyps): colorectal carcinoma is 40%.
occur most commonly in the context of HNPCC also may be associated with
inflammatory bowel disease, amebic colitis, extracolonic malignancies, including
ischemic colitis, and schistosomal colitis; not endometrial (most common), ovarian,
premalignant, but they cannot be pancreas, stomach, small bowel, biliary, and
distinguished from adenomatous polyps based urinary tract carcinomas.
upon gross appearance & therefore should be Diagnosis: Amsterdam criteria for clinical
removed. diagnosis of HNPCC are three affected
Hamartomas: similar appearance to relatives with histologically verified
adenomatous polyps but is not considered to adenocarcinoma of the large bowel (one must
be premalignant be a 1st degree relative of one of the others) in
Familial juvenile polyposis: autosomal 2 successive generations of a family with 1
dominant DO in which patients develop patient diagnosed before age 50 years.
hundreds of polyps in the colon and Screening colonoscopy is recommended
rectum; degenerate into adenomas annually for at-risk patients beginning at
carcinoma either age 20 to 25 years or 10 years younger
Peutz Jeghers syndrome: characterized than the youngest age at diagnosis in the
by polyposis of the small intestine and, to family, whichever comes first.
a lesser extent, of the colon and rectum. ;
Characteristic melanin spots are noted on Familial colorectal cancer
the buccal mucosa and lips of these Nonsyndromic familial colorectal cancer
patients. accounts for 10 to 15% of patients with
colorectal cancer
Cronkite-Canada syndrome: GI
Screening colonoscopy is recommended every
polyposis + alopecia + cutaneous
5 years beginning at age 40 years or beginning
pigmentation + atrophy of the fingernails
10 years before the age of the earliest
and toenails; SSx: Diarrhea, vomiting,
diagnosed patient in the pedigree.
malabsorption, and protein-losing
enteropathy
- Routes of spread & natural history:
TOPNOTCH MEDICAL BOARD PREP SURGERY SUPPLEMENT HANDOUT Page 69 of 81
For inquiries visit www.topnotchboardprep.com.ph or email us at topnotchmedicalboardprep@gmail.com
TOPNOTCH MEDICAL BOARD PREP SURGERY SUPPLEMENT HANDOUT - Jules Lopez,MD-MBA,Teddy Carpio,MD-MBA
For inquiries visit www.topnotchboardprep.com.ph or email us at topnotchmedicalboardprep@gmail.com
Regional lymph node involvement is the most G. ANAL INTRAEPITHELIAL NEOPLASIA (BOWEN’S
common form of spread of colorectal carcinoma DISEASE)
and usually precedes distant metastasis - refers to squamous cell carcinoma in situ of the
T stage (depth of invasion) is the single most anus.
significant predictor of lymph node spread - precursor to an invasive squamous cell carcinoma
The number of lymph nodes with metastases (epidermoid carcinoma)
correlates with the presence of distant disease and - may appear as a plaque-like lesion, or may only be
inversely with survival: 4 or more involved lymph apparent with high-resolution anoscopy and
nodes predict a poor prognosis application of acetic acid or Lugol's iodine solution.
most common site of distant metastasis from - associated with HPV infection types 16 and 18.
colorectal cancer is the liver (via hematogenous - Incidence has increased dramatically among HIV-
spread to the portal venous system) positive, homosexual men.
- Treatment:
- Screening: Ablation
annual DRE at age 40 Topical immunomodulators such as imiquimod,
FOB at age 50 Topical 5-FU
Flexible signoidoscopy every 5 years at age 50
Colonoscopy if with risk factors H. VOLVULUS
- Clinical presentation: change in bowel habits, rectal - occurs when an air-filled segment of the colon twists
bleeding, melena, unexplained anemia, or weight about its mesentery
loss - clinical manifestations: similar to SBO; abdominal
- Staging and treatment distention, nausea, and vomiting; can rapidly progress
to generalized abdominal pain and tenderness; Fever
Table 58. Duke staging of colorectal cancer & treatment and leukocytosis are heralds of gangrene and/or
Stage description 5 year Treatment perforation
survival
Stage A Cancer limited 90% Wide resection Table 59. Comparison of sigmoid vs cecal volvulus
to mucosa & of colon with Sigmoid volvulus Cecal volvulus
submucosa sampling of LN -90% of cases -<20% of cases
Stage B Cancer invades 70% (to rule out ↑er -plain abdominal xray: bent inner -plain abdominal xray: kidney-
the muscularis disease stage); tube or coffee bean appearance shaped, air-filled structure in the
propria stage B can with the convexity of the loop lying LUQ
also employ in the RUQ (opposite site of
RT+CT obstruction)
Stage C Invasion of 30% Surgery + -gastrografin enema: bird’s beak
local LN chemotx (5- (pathognomonic)
fluorouracil + Initial management: fluid -surgical exploration once
leucovorin) + resuscitation followed by diagnosis is made; no room for
RT endoscopic detorsion (rigid endoscopic detorsion
Stage D Distant Limited palliative proctoscope); if suspecting
metastasis survival gangrene or perforation, perform
immediate surgical exploration
LAR: for tumors located 5-10 cm from anal verge
APR: tumors less than 5 cm from ananl verge and if
recurrent cancer at LAR site
MUST KNOW a
F. COLORECTAL CARCINOID TUMORS
- Result of a neuroendocrine tumor that secretes Bird’s beak is also seen in barium esophagogram of
neurotransmitters (serotonin, ACTH, histamine, achalasia!
dopamine, tryptophan, substance P, bradykinin)
- occur most commonly in the GI tract
- most common location is appendix (50%), ileum (25%)
then rectum (20%) I. COLONIC PSEUDOOBSTRUCTION (OGILVIE’S
- ileal carcinoid has the highest potential for metastasis SYNDROME)
- appendiceal has lowest potential for metastasis - Distention of the abdomen leading to colonic
- Carcinoid tumors in the proximal colon are less obstruction (even if there is no obvious signs of
common and are more likely to be malignant. obstruction)
- Can be part of MEN type I - most commonly occurs in hospitalized patients and is
- risk of malignancy increases with size (more than 60% associated with the use of narcotics, bedrest, and
of tumors greater than 2 cm in diameter are associated comorbid disease.
with distant metastases) - Due to a neurologic dysfunction, electrolyte
- clinical manifestation: abnormality and ↑age
triad of flushing (due to excess bradykinin), - Treatment: NGT, IV neostigmine, IV atropine (to
diarrhea (due to excess serotonin) & valculhar counter bradycardia as SE of neostigmine), exploratory
heart disease (primarily affects the mitral valve laparotomy during worst case scenario)
from excess serotonin)
others: hypotension, tachycardia, alcohol J. HEMORRHOIDS
intolerance - are cushions of submucosal tissue containing venules,
- diagnosis: arterioles, and smooth-muscle fibers that are located in
24 hour 5-HIAA collection (5-HIAA is a metabolite the anal canal
of serotonin) - Excessive straining, increased abdominal pressure, and
- treatment: hard stools lead to further prolapsed of hemorrhoids
Small carcinoids can be locally resected, either - Difference between internal and external hemorrhoids:
transanally or using transanal endoscopic
microsurgery.
Larger tumors or tumors with obvious invasion
into the muscularis require more radical resection
Medical: somatostatin analogues (octreotide)
RT

TOPNOTCH MEDICAL BOARD PREP SURGERY SUPPLEMENT HANDOUT Page 70 of 81


For inquiries visit www.topnotchboardprep.com.ph or email us at topnotchmedicalboardprep@gmail.com
TOPNOTCH MEDICAL BOARD PREP SURGERY SUPPLEMENT HANDOUT - Jules Lopez,MD-MBA,Teddy Carpio,MD-MBA
For inquiries visit www.topnotchboardprep.com.ph or email us at topnotchmedicalboardprep@gmail.com
Table 60: comparison of internal and external hemorrhoids - Signs of (+) fistula: nonhealing of an abscess wounds or
Internal hemorrhoids External hemorrhoids recurrence of an abscess at the same location
-Exaggerated submucosal vascular - are dilated veins of the inferior
cushions normally located above hemorrhoidal plexus located M. FISTULA IN ANO
dentate line; covered by below the dentate line and
- Classification (based on relationship to the anal
insensate transitional mucosa of covered by anoderm; can cause
the anal canal and not by significant pain sphincter complex)
anoderm; only become painful 1. Intersphincteric (most common) : tracks
when already thrombosed / through the distal internal sphincter and
necrosis intersphincteric space to an external opening near
the anal verge
-prolapsing hemorrhoids: are
internal hemorrhoids beyond the
2. Transsphincteric: often results from an ischiorectal
dentate line abscess and extends through both the internal and
external sphincters
3. Suprasphincteric: originates in the intersphincteric
- Types of internal hemorrhoids: graded according to plane and tracks up and around the entire external
extent of prolapsed sphincter
4. Extrasphincteric: originates in the rectal wall and
1st degree: bulge into the anal canal and may prolapse tracks around both sphincters to exit laterally,
beyond the dentate line on straining usually in the ischiorectal fossa
2nd degree: prolapse through the anus but reduce - Goodsall’s rule : states that, if the external opening
spontaneously is anterior to the imaginary line drawn between the
3rd degree: prolapse through the anal canal and require ischial tuberosities, the fistula runs directly into the
manual reduction anal canal. If the external opening is posterior, the
4th degree: prolapse but cannot be reduced and are at tract curves to the posterior midline.
risk for strangulation
- Combined internal and external hemorrhoids: ***EXCEPTION: if an anterior external opening is
straddle the dentate line; mixed characteristics greater than 3 cm from the anal margin, these
- Post partum hemorrhoids: result from straining fistulas usually track to the posterior midline!!!
during labor, which results in edema, thrombosis,
and/or strangulation. Figure 35. Goodsall’s rule
- Treatment:
Dietary fiber, stool softeners, ↑OFI, avoid straining:
for 1st & 2nd degree hemorrhoids
Rubber band ligation: for persistent 1st & 2nd
degree hemorrhoids & selected 3rd degree
hemorrhoids
If thrombosed hemorrhoids, perform excision
Most common complication of
hemorrhoidectomy: urinary retention

K. ANAL FISSURE
- is a tear in the anoderm distal to the dentate line
- 90% of fissures are located at the posterior midline, - Treatment: fistulotomy with adequate drainage or seton
an area where the anoderm is least supported by the placement
sphincter
- Fissures located laterally should arouse suspicion of
Crohn’s, UC, syphilis, TB,leukemia REVIEW QUESTIONS
- Clinical manifestation: tearing pain with defecation
and hematochezia; often too tender to tolerate DRE 1. Which of the following is important in maintaining
- Treatment the integrity of the colonic mucosa?
Initially, can be managed conservatively with
lubricants, warm sitz bath and bulk laxatives a. short-chain fatty acids
(treatment of choice) b. alanine
Surgery: lateral subQ partial internal c. medium-chain fatty acids
sphincterectomy d. glutamine
Posterior fissurectomy & sphincterectomy
can lead to keyhole defect & constant Answer: A
soiling Short chain fatty acids are produced by bacterial
fermentation of dietary carbohydrates. Short chain
L. ANORECTAL ABCESS fatty acids are an important source of energy for
- Perianal abscess: most common manifestation and the colonic mucosa, and metabolism by colonocytes
appears as a painful swelling at the anal verge provides energy for processes such as active
- Ischiorectal abscess: happens when there is Spread transport of sodium. Lack of a dietary source for
through the external sphincter below the level of the production of short chain fatty acids, or diversion of
puborectalis; may become extremely large and may not the fecal stream by an ileostomy or colostomy, may
be visible externally; DRE will reveal a painful swelling result in mucosal atrophy and diversion colitis.
laterally in the ischiorectal fossa
- Intersphincteric abscess: occur in the intersphincteric 2. Match the organs in the left hand column with the
space and are notoriously difficult to diagnose; causes location of their referred pain in the right hand
deep pain in the rectum without external manifestation column. (items in the right may be used more than
- Pelvic and superior levator abscess: rare; may result once)
from extension of an intersphincteric or ischiorectal
abscess upward, or extension of an intraperitoneal A. Gallbladder a. epigastrium
abscess downward B. Jejunum b. periumbilical
- Horseshoe abscess: bilateral ischiorectal, supralevator C. Rectum c. hypogastrium
or perianal abscesses that communicate; begins as a D. Pancreas d. shoulder
posterior midline infection E. Appendix
- Treatment: drainage with local anesthesia

TOPNOTCH MEDICAL BOARD PREP SURGERY SUPPLEMENT HANDOUT Page 71 of 81


For inquiries visit www.topnotchboardprep.com.ph or email us at topnotchmedicalboardprep@gmail.com
TOPNOTCH MEDICAL BOARD PREP SURGERY SUPPLEMENT HANDOUT - Jules Lopez,MD-MBA,Teddy Carpio,MD-MBA
For inquiries visit www.topnotchboardprep.com.ph or email us at topnotchmedicalboardprep@gmail.com
Answer: A – a,d; B – b; C – c; D – a; E – b E.Rocky davis Right lower quadrant incision or muscle splitting
incision incision for appendectomy
The visceral peritoneum is innervated by C fibers F.McBurney it begins 2 to 5 centimeters above the anterior
coursing with the autonomic ganglia. C fibers are incision superior iliac spine and continues to a point one-
third of the way to the umbilicus (McBurney's
unmyelinated, slow-conducting (0.5-5.0 m/s), point). Thus, the incision is parallel to the
polymodal nociceptors that travel bilaterally with the external oblique muscle
sympathetic and parasympathetic fibers. Visceral pain is G.Transverse Similar to kocher incision (subcostal incision).
a response to injury of the visceral peritoneum. incision Preferred for newborns and infants because more
Distension, stretch, traction, compression, torsion, abdominal exposure is gained per length of the
ischemia and inflammation trigger visceral pain incision compared to vertical exposure
fibers. Abdominal organs are insensate to heat, H.Pfannenstiel Pfannenstiel incision, used commonly for pelvic
cutting and electrical stimulation. incision procedures, is distinguished by transverse skin
and anterior rectus sheath incisions, followed by
rectus muscle retraction and longitudinal incision
Visceral pain is typically vague and crampy and is of the peritoneum.
perceived in the region of oprigin of the embryologically
derived autonomic ganglia. Foregut organs (proximal B. RECTUS ABDOMINIS DIASTASIS
to the ligament of treitz) refer pain to the celiac chain, - Other name: diastasis recti
and the pain is felt in the epigastrium. The organs of the - is a clinically evident separation of the rectus abdominis
midgut (small intestine, ascending colon) refer pain to muscle pillars resulting to a characteristic bulging of
the superior mesenteric chain (periumbilical chain) and the abdominal wall in the epigastrium (sometimes
those of the hindgut (transverse and descending colon, mistaken for a ventral hernia)
sigmoid colon and rectum) to the inferior mesenteric - may be congenital
ganglia and hypogastrium. - can be associated with advancing age, in obesity, or
after pregnancy
- In the postpartum setting, rectus diastasis tends to
ABDOMINAL WALL & HERNIAS occur in women who are of advanced maternal age, who
have a multiple or twin pregnancy, or who deliver a
A. Abdominal Incisions high-birth-weight infant.
B. Rectus abdominis diastasis - Diagnosis:
C. Rectus sheath hematoma CT scan: can differentiate rectus diastasis from a
D. Abdominal wall hernias true ventral hernia
E. Incisional hernias - Treatment: surgery
F. Retroperitoneal fibrosis
G. Inguinal hernias C. RECTUS SHEATH HEMATOMA
H. Femoral hernia - As a result of hemorrhage from any of the network of
collateralizing vessels (superior and inferior epigastric
arteries or veins) within the rectus sheath and muscles
- History: trauma, sudden contraction of the rectus
muscles with coughing, sneezing, or any vigorous
A. ABDOMINAL INCISIONS physical activity.
- Clinical manifestations: sudden onset of unilateral
Figure 36. Abdominal incisions abdominal pain that increases with contraction of the
rectus muscles; palpable tender mass
(+)Fothergill's sign: palpable abdominal mass
that remains unchanged with contraction of the
rectus muscles
- Diagnosis:
Abdominal UTZ may show a solid or cystic mass
within the abdominal wall
CT scan: most definitive study to establish the
correct diagnosis and to exclude other disorders
- Treatment: nonoperative; surgery is indicated in
instances of expensing hematoma and hemodyanamic
instability

D. ABDOMINAL WALL HERNIAS


- This is due to defects in the parietal abdominal wall
fascia and muscle through which intra-abdominal or
preperitoneal contents can protrude
- ACQUIRED HERNIAS
A.Midline -are used because of the flexibility offered by this
incision approach in establishing adequate exposure.
may develop through slow architectural
-the incision in the fused midline aponeurotic deterioration of the muscular aponeuroses or they
tissue (linea alba) is simple and requires no may develop from failed healing of an anterior
division of skeletal muscle. abdominal wall incision (incisional hernia).
B.paramedian -made longitudinally 3 cm off the midline, most common finding is a mass or bulge on the
incision through the rectus abdominis sheath structures, anterior abdominal wall, which may increase in
and have largely been abandoned in favor of size with a Valsalva maneuver
midline or nonlongitudinal access methods PE reveals a bulge on the anterior abdominal wall
C.right -Subcostal incisions on the right (Kocher incision
that may reduce spontaneously, with recumbency,
subcostal for cholecystectomy) or left (for splenectomy) are
incision archetypal muscle-dividing incisions that or with manual pressure
D.bilateral generally result in the transaction of some or all Treatment: if incarcerated (cannot be reduced) or
subcostal of the rectus abdominis muscle fibers and strangulated (BS is compromised) do surgical
investing aponeuroses. correction
-These incisions generally are closed in two - PRIMARY VENTRAL HERNIAS
layers (anterior aponeurotic sheath of the rectus
Non incisional or true ventral hernias
muscle medially, transitioning to external oblique
muscle and aponeurosis more laterally & Examples:
posterior, deeper layer consists of internal
oblique and 72ransverses abdominis muscle)
TOPNOTCH MEDICAL BOARD PREP SURGERY SUPPLEMENT HANDOUT Page 72 of 81
For inquiries visit www.topnotchboardprep.com.ph or email us at topnotchmedicalboardprep@gmail.com
TOPNOTCH MEDICAL BOARD PREP SURGERY SUPPLEMENT HANDOUT - Jules Lopez,MD-MBA,Teddy Carpio,MD-MBA
For inquiries visit www.topnotchboardprep.com.ph or email us at topnotchmedicalboardprep@gmail.com
Epigastric hernias: congenital due to defective
midline fusion of lateral abdominal wall; occurs in - Clinical manifestations: groin pain
multiples and are small; located in the midline - Diagnosis: usually employed for ambiguous diagnosis
between the xiphoid process and the umbilicus; (i.e. obese patients)
found to contain omentum or a portion of the - Treatment: definitive treatment is surgical repair
falciform ligament.
Bassini repair: anterior approach, nonprosthetic,
Umbilical hernias: due to a patent umbilical ring; hernia reduced and the defect oversewn, &
more common in premature infants;spontaneous reconstruction the site of weakness; disadvantage: (+)
closure can occur at age of 5, no closure by that tension on the reconstructed tissue
time, do elective surgical repair
Shouldice repair: anterior approach, nonprosthetic,
Spigelian hernias: occur anywhere along the multilayer (4-layer suture repair) reconstruction
length of the Spigelian line or zone—an distributes the tension, effectively resulting in a
aponeurotic band of variable width at the lateral tension-free repair; lowest recurrence rate
border of the rectus abdominis.
Pott’s repair: high ligation of the sac only, with no
E. RETROPERITONEAL FIBROSIS repair of the inguinal canal; used for indirect hernias
- class of disorders characterized by hyperproliferation of only
fibrous tissue in the retroperitoneum
- if primary, it is known as Ormond disease McVay repair: anterior approach, nonprosthetic; the
- may be secondary to inflammatory process, malignancy, conjoined tendon is sutured to the cooper’s ligament
or medication (methysergide, ergotamine, laterally; can be used for indirect, direct & femoral
hydralazine, methyldopa and B blockers) hernias
- Men are twice as likely to be affected as women ***problem with anterior non prosthetic
- primarily affects individuals in the 4th-6th decades of approaches: high recurrence rates
life.
- Clinical manifestations: Lichenstein tension free repair: addition of a mesh
Sx: insidious onset of dull, poorly localized prosthesis effected a reconstruction of the posterior
abdominal pain, unilateral leg swelling, inguinal canal, without placing tension on the floor itself
intermittent claudication, oliguria, hematuria, &
dysuria. Read-rives repair: anterior preperitoneal approach
PE: hypertension, the palpation of an abdominal or
flank mass, lower extremity edema (unilateral or Rives, stoppa, wanz repair: giant prosthetic
bilateral), or diminished lower extremity pulses reinforcement of the visceral sac; preperitoneal
(unilateral or bilateral). approach
- Diagnosis:
↑ESR, BUN & creatinine Kugel repair: maximize on the preperitoneal approach
Most definitive noninvasive diagnostic test: while minimizing on the length of the skin and fascia
intravenous pyelography incision
- Treatment: corticosteroids with or without surgery
(only indicated when renal function is compromised) Laparoscopic repair: uses preperitoneal approach
with small incisions; can asses and repair unilateral or
F. INGUINAL HERNIAS bilateral inguinal hernias
Emergent inguinal hernia repair: reserved for
Table 61. comparison of Inguinal hernias strangulated, incarcerated and sliding hernias
Indirect inguinal hernia Direct inguinal hernia
etiology - usually congenital; due -Usually acquired;
to patent processus weakness in the Figure 37: treatment algorithm for hernia repair
vaginalis abdominal wall
musculature
Risk Strenuous physical activity, obesity, ehler’s danlos,
factors: smoking
anatomy protrude lateral to the protrusions medial to the
inferior epigastric inferior epigastric
vessels, through the deep vessels, in Hesselbach's
inguinal ring triangle
PE: cough impulse is Cough impulse is
inguinal controlled; felt on the manifest; felt on the
occlusion dorsum of fingertip fingertip
test

MUST KNOW a

Hesselbach’s triangle:
Inferior: inguinal ligament
Medial: rectus abdominis
Superolateral border: inferior epigastric vessels

MUST KNOW a

Femoral hernias are more prevalent in females compared to


males but the most common type of groin hernia in females
is still indirect inguinal hernia.

TOPNOTCH MEDICAL BOARD PREP SURGERY SUPPLEMENT HANDOUT Page 73 of 81


For inquiries visit www.topnotchboardprep.com.ph or email us at topnotchmedicalboardprep@gmail.com
TOPNOTCH MEDICAL BOARD PREP SURGERY SUPPLEMENT HANDOUT - Jules Lopez,MD-MBA,Teddy Carpio,MD-MBA
For inquiries visit www.topnotchboardprep.com.ph or email us at topnotchmedicalboardprep@gmail.com
REVIEW QUESTIONS Pringle maneuver: used to clamp this ligament in
the event of injury to the right hepatic artery
1. Which of the following is the most important initial during cholecystectomy
therapy for a patient with portal hypertension, Relationship: CBD is located at the right of the
ascites, and a tense umbilical hernia? hepatic artery and anterior to the portal vein
From the right and deep to the porta hepatis is the
a. Primary repair with concurrent placement of a foramen of winslow (or epiploic foramen)
peritoneal venous shunt - Segmental anatomy
b. Emergency primary repair to avoid hernia rupture Cantlie’s line: plane from the gallbladder fossa to
c. Medical therapy to control ascites the IVC that separates the liver’s right and left
d. Transjugular intrahepatic portocaval shunt lobes grossly.
followed by umbilical hernia repair Falciform ligament: separates the left lateral and
left medial segments along the umbilical fissure
Answer: C and anchors the liver to the anterior abdominal
Treatment and control of the ascited with diuretic, wall; does not separate the liver to right and left
dietary management and paracentesis is the most lobes
appropriate initial therapy. Patients with refractory Couinaud’s segments: divides liver into 8
ascited may be candidates for transjugular intrahepatic segments, in clockwise direction with caudate lobe
portocaval shunting or eventual liver transplantation. as segment 1
Umbilical hernia repair should be deferred until after
the ascites is controlled.
Figure 38. Segmental anatomy of liver
2. In the setting of an equivocal examination, which
of the following has the greatest sensitivity in
diagnosing an inguinal hernia?

a. Repeat examination by a second surgeon


b. Ultrasound
c. CT scan
d. MRI

Answer: D
Although Ct scan is useful in ambiguous clinical Notes to figure
Segments part Corresponding side Venous drainage
presentations, little data exist to support its routine use Segment I Caudate lobe IVC
in diagnosis. The use of MRI in assessing groin hernias Segment II Left lateral superior segment Left lobe Left hepatic vein
Segment III Left lateral inferior segment Left lobe Left hepatic vein
was examined in a group of 41 patients scheduled to Segment IV Left medial segment (quadate Left lobe Middle hepatic
undergo laparoscopic inguinal hernia repair. lobe – outdated) vein
Segment V Right anterior inferior segment Right lobe Right & middle
Preoperatively, all patients underwent US and MRI. hepatic vein
Laparoscopic confirmation of the presence of inguinal Segment VI Right posterior inferior segment Right lobe Right hepatic vein
hernia was deemed as gold standard. Physical Segment Right posterior superior segment Right lobe Right hepatic vein
VII
examination was found to be the least sensitive. False Segment Right anterior superior segment Right lobe Right & middle
positives were low on physical examination and MRI VIII hepatic vein

(one finding), but higher with US (four findings). With


- The hepatic veins divides the liver into 4 sectors
further refinement of technology, radiologic techniques
- The liver has dual blood supply:
qill continue to improve sensitivity and specificity rates
hepatic actery: 25%
of diagnosis, thereby serving a supplementary role in
branch of celiac artery
cases of undertain diagnosis
most common variation: right hepatic artery
from SMA
portal vein: 75% (majority)
LIVER, PORTAL VENOUS SYSTEM & GALLBLADDER confluence of splenic vein and SMV
- normal pressure: 3-5 mmHg
A. Anatomy - communication of portal vein and systemic circulation
B. Liver function tests (important for location of varices & bleeding in portal
C. Radiographic evaluation hypertension): gastroesophageal junction, anal
D. Liver cirrhosis canal, falciform ligament, splenic venous bed and
E. Portal Hypertension left renal vein, and retroperitoneum
F. Budd-Chiari syndrome - Biliary tree
G. Infections of the liver Hepatic ducts follow arterial branching of the liver
H. Benign neoplasms of the liver Left hepatic duct has a longer extrahepatic course
I. Malignant tumors
J. Gallstone disease B. LIVER FUNCTION TESTS
K. Acute cholecystitis - Term used to frequently measure the levels of group of
L. Choledocholithiases serum markers for evaluation of liver dysfunction.
M. Cholangitis - A misnomer because the panel measures cell damage,
N. Biliary pancreatitis and not liver function
O. Acalculous cholecystitis
P. Biliary or choledochal cysts Table 62. Different components of liver function tests
Q. Sclerosing cholangitis Serum albumin, -Measures liver’s synthetic function
prothrombin - prothrombin time and INR: best test
time & clotting among the 3 to measure the liver’s synthetic
factors (except function
factor VIII) - PT is prolonged with conditions such as
A. ANATOMY vitamin K deficiency or warfarin therapy
(because vitamin K is involved in the Y-
Liver carboxylation of factors used to measure
- Largest organ, weighing approximately 1500 g prothrombin time)
- Hepatoduodenal ligament contains the porta ***factor VIII: not synthesized exclusively in the
liver; has the shortest half life; useful for
hepatis (portal vein, hepatic artery and common bile determining liver failure
duct) AST (formarly -Indicators of integrity of hepatocellular
TOPNOTCH MEDICAL BOARD PREP SURGERY SUPPLEMENT HANDOUT Page 74 of 81
For inquiries visit www.topnotchboardprep.com.ph or email us at topnotchmedicalboardprep@gmail.com
TOPNOTCH MEDICAL BOARD PREP SURGERY SUPPLEMENT HANDOUT - Jules Lopez,MD-MBA,Teddy Carpio,MD-MBA
For inquiries visit www.topnotchboardprep.com.ph or email us at topnotchmedicalboardprep@gmail.com
SGOT) & ALT membranes; increased levels reflect - if acute cholecystitis: (+) edema within the wall of the
(formarly SGPT) hepatocellular injury with leakage GB or between the GB and liver in association with
-AST: can also be found in liver, cardiac localized tenderness
muscle, skeletal muscle, kidney, brain, - if chronic cholecystitis: contracted thick-walled GB
pancreas, lungs, and red blood cells and thus is
less specific
- Extrahepatic ducts are well visualized using UTZ
-ALT: more specific for liver disease (except for retroduodenal portion)
-AST:ALT ratio of >2:1 alcoholic liver - Dilation of the ducts + stones in the GB + jaundiced
disease patient think extrahepatic obstruction
moderate increases: viral hepatitis Periampullary tumors can be difficult to diagnose
-in the thousands ischemia, toxin ingestion
on UTZ
(acetaminophen), fulminant hepatitis
Indirect -elevations point to intrahepatic cholestasis,
UTZ is useful for evaluating tumor invasion and
(unconjugated) hemolytic disorders (hemolytic anemia, flow in the portal vein – an important guideline in
bilirubin resoprtion of hematomas), bilirubin defects in the resectability of periampullary and pancreatic
hepatic uptake or conjugation (acquired or head tumors
inherited)
Direct -elevations point to extrahepatic or obstructive ORAL CHOLECYSTOGRAPHY
(conjugated) cholestasis, inherited or acquired disorders of - Considered as a diagnostic procedure of choice for
bilirubin intrahepatic excretion or extrahepatic gallstones but it largely replaced now by UTZ.
obstruction - Mechanism: oral administration of radiopaque
Alkaline -found in liver and bones;
compound that is absorbed and excreted by the liver,
phosphatase -indicative of biliary obstruction
- since half life of AP is 7 days, it may take passed into the GB stones are noted on a film as a
several days for the levels to normalize even filling defect in a visualized, opacified GB
after resolution of biliary obstruction
GGTP -early marker and sensitive test for
hepatobiliary disease BILIARY RADIONUCLIDE SCANNING (HIDA SCAN)
-nonspecific; can also be elevated in overdose
of certain medications, alcohol abuse,
pancreatic disease, myocardial infarction,
- Provides a noninvasive evaluation of the liver, GB, bile
renal failure, & obstructive pulmonary disease ducts and duodenum with both anatomic and functional
- interpret GGTP elevations with other enzyme information
abnormalities - Mechanism: Technetium-labeled derivatives of
dimethyl iminodiacetic acid (HIDA) are injected IV
BIOCHEMISTRY cleared by Kuppfer cells in the liver excreted in
the bile
AST (aspartate transaminase): an enzyme in gluconeogenesis 10 minutes: time it takes for the liver to detect it
that transfers amino groups from aspartic acid to ketoglutaric 60 minutes: time it takes for the GB, bile ducts and
acid to produce oxaloacetate. duodenum to detect it
- the primary use of biliary scintigraphy is the
ALT (alanine transaminase): an enzyme in gluconeogenesis that diagnosis of acute cholecystitis
transfers amino groups from alanine to ketoglutaric acid to appearance: nonvisualized GB, with prompt filling
produce pyruvic acid of the common bil duct and duodenum, biliary
obstruction
sensitivity & specificity: 95%
C. RADIOGRAPHIC EVALUATION - can also detect obstruction of the ampulla
appearance: filling of the GB and CBD with delayed
ULTRASOUND and absent filling of the duodenum
- can also be used for detection of biliary leaks as a
Liver complication of GB surgery
- Useful initial test imaging test of the liver because it
is inexpensive, involves no radiation exposure, and is
well tolerated by patients COMPUTED TOMOGRAPHY
- It is excellent for diagnosing biliary pathology and liver Liver
lesions. - Contrast medium is routinely used for liver evaluation
- Limitations: because of the similar densities of most pathologic
Incomplete imaging: dome or beneath the ribs on liver masses and normal hepatic parenchyma.
the surface, lesion boundaries are not as visualized Uses dual or triple phase bolus of IV contrast
Obesity Exploits the dual blood supply of the liver: most
Overlying gas bowels liver tumors receive their blood supply from
- If a mass is detected, further evaluation by CT or MRI is the hepatic artery and normal hepatic
required since UTZ has lower sensitivity and specificity parenchyma from portal vein
- Intraoperative ultrasound: 2 phases:
Gold standard for diagnosing liver lesions 1. Arterial dominant phase (20 to 30
Useful for tumor staging, visualization of seconds after beginning of contrast
intrahepatic vascular structures, guidance of injection) – the phase where hepatic tumors
resection plane by assessment of relationship of and other hypervascular lesions are well
mass to vessels, for biopsy of tumors and tumor delineated.
ablation 2. Venous or portal dominant phase (60 to
70 seconds after contrast injection) – the
Gallbladder phase where there is optimal enhancement
- UTZ is the initial investigation used for any patient of normal liver parenchyma and
suspected of disease in the biliary tree. hypovascular lesions (will appear
- UTZ will show gallbladder stones with sensitivity and attenuated in contrast with brighter normal
specificity of >90% liver parenchyma)
Appearance of GB Stones: (+) acoustic shadow,
move with changes in position (vs polyps: may gallbladder
also have a shadow but does not move with - It is the test of choice in evaluating patients with
changes in position) suspected malignancy of the GB, extrahepatic
If a stone obstructs the neck of the GB: large GB but biliary system or nearby organs, in particular, the
thin walled head of pancreas
TOPNOTCH MEDICAL BOARD PREP SURGERY SUPPLEMENT HANDOUT Page 75 of 81
For inquiries visit www.topnotchboardprep.com.ph or email us at topnotchmedicalboardprep@gmail.com
TOPNOTCH MEDICAL BOARD PREP SURGERY SUPPLEMENT HANDOUT - Jules Lopez,MD-MBA,Teddy Carpio,MD-MBA
For inquiries visit www.topnotchboardprep.com.ph or email us at topnotchmedicalboardprep@gmail.com
- Abdominal CT scan is inferior to UTZ in diagnosing Macronodular cirrhosis: frequently has septa and
gallstones regenerative nodules (irregularly sized
hepatocytes with large nuclei and cell plates of
varying thickness)
PERCUTANEOUS TRANSHEPATIC CHOLANGIOGRAPHY Mixed cirrhosis: present when regeneration is
- Useful in patients with bile duct strictures and occurring in a micronodular liver and over time
tumors, as it defines the anatomy of the biliary tree converts to a macronodular pattern
proximal to the affected segment - Etiology: viral, autoimmune, drug-induced, cholestatic,
- Mechanism: intrahepatic ducts are accessed and metabolic diseases
percutaneously with a small needle under fluoroscopic - Clinical manifestation
guidance catheter is placed cholangiogram Fat stores and muscle mass are reduced
performed can do therapeutic interventions as resting energy expenditure is increased
(+) Muscle cramps: respond to administration of
well (biliary drain insertion, stent placement)
quinine sulfate and human albumin
- Very little role in management of uncomplicated
increased CO & HR
gallstone disease
Prone to infections due to impaired phagocytic
activity of the RES
MAGNETIC RESONANCE IMAGING
- Diagnosis
Liver mild normocytic normochromic anemia.
- Also uses contrast agent, just like in CT scan, to Decreased WBC & PC
differentiate normal and pathologic lesion in the liver bone marrow: macronormoblastic
- Types: prothrombin time is prolonged & does not respond
Gadopentate dimeglumine – behaves in a manner to vitamin K tx
similar to iodine in CT serum albumin level is decreased
Feruxomide – excretion of kuppfer cells serum levels of bilirubin, transaminases, and
Iminoacetic acid-derivative radionuclide – alkaline phosphatase are all elevated
secretion in bile by hepatocytes
- CHILD-TURCOTTE-PUGH SCORE: evaluate the risk of
Gallbladder portocaval shunt procedures secondary to portal
- MRI with MRCP (magnetic resonance hypertension and also useful in predicting surgical
cholangiopancreatography) offers a single risks of other intra-abdominal operations
noninvasive test for the diagnosis of bliary tract and performed on cirrhotic patients
pancreatic disease
Table 63. Child-Turcotte-Pugh Score
variable 1 point 2 points 3 points
Bilirubin < 2 mg/dL 2-3 mg/dL >3 mg/dL
ENDOSCOPIC RETROGRADE CHOLANGIOGRAPHY (ERCP) Albumin >3.5 g/dL 2.8-3.5 g/dL <2.8 g/dL
- It is the diagnostic and therapeutic procedure of INR <1.7 1.7-2.2 >2.2
choice for stones in the CBD associated with Encephalopathy none controlled uncontrolled
obstructive jaundice, cholangitis and gallstone Ascites none controlled uncontrolled
pancreatitis Child-Turcotte-Pugh Class & overall surgical mortality rates
- Provides direct visualization of the bilary and Class A = 5-6 points 10%
pancreatic ducts, particularly the ampullary region and Class B= 7-9 points 30%
distal common bile duct Class C= 10-15 points 75-80%
- Therapeutic interventions include sphincterotomy,
stone extraction if indicated E.
PORTAL HYPERTENSION
-
definition: direct portal venous pressure that is >5
mmHg greater than the IVC pressure, a splenic pressure
POSITRON EMISSION TOMOGRAPHY of >15 mmHg, or a portal venous pressure measured at
liver surgery of >20 mmHg
- PET offers functional imaging of tissues with high - normal portal venous pressure: 5 to 10 mmHg
metabolic activity, including most types of metastatic at this pressure, very little blood is shunted from
tumors the portal venous system into the systemic
- With high value for colorectal cancer with liver circulation
metastases as portal venous pressure increases, the
20% of patients with colorectal cancer present communication with the systemic circulation dilate
initially with liver metastasis Large amount of blood is shunted around the
presence of extrahepatic disease is a poor liver and into the systemic circulation
prognosticator and precludes surgical intervention complications
valuable tool for the diagnostic work up of A portal pressure of >12 mmHg is necessary for
patient with potentially resectable hepatic varices to form and subsequently bleed
disease - Etiology: most common cause is cirrhosis
must be combined with CT to improve (intrahepatic)
diagnostic accuracy - Clinical manifestation
Most significant clinical finding:
D. LIVER CIRRHOSIS gastroesophageal varices
- final sequela of chronic hepatic insult, is characterized Major BS of GE varices: anterior branch of the
by the presence of fibrous septa (due to left gastric or coronary vein
accumulation to ECM matrix or scar tissue) May present with splenomegaly, hemorrhoids,
throughout the liver subdividing the parenchyma into ascites, caput medusa & upper GI bleeding due to
hepatocellular nodules variceal bleeding (leading cause of morbidity
- 2 consequences: hepatocellular failure and portal and mortality)
hypertension - Diagnosis: most accurate method of determining portal
- Classification hypertension is hepatic venography
Micronodular cirrhosis: characterized by thick - Management
regular septa, small uniform regenerative nodules, Prevention of variceal bleeding: improve liver
and involvement of virtually every hepatic lobule function (avoid alcohol), avoid aspirin & NSAID,
beta blockers
TOPNOTCH MEDICAL BOARD PREP SURGERY SUPPLEMENT HANDOUT Page 76 of 81
For inquiries visit www.topnotchboardprep.com.ph or email us at topnotchmedicalboardprep@gmail.com
TOPNOTCH MEDICAL BOARD PREP SURGERY SUPPLEMENT HANDOUT - Jules Lopez,MD-MBA,Teddy Carpio,MD-MBA
For inquiries visit www.topnotchboardprep.com.ph or email us at topnotchmedicalboardprep@gmail.com
Management of acute variceal bleeding - definitive imaging: hepatic venography
Specifics: - initial treatment: anticoagulation
ICU admission: must!
Blood resuscitation: goal is Hgb of G. INFECTIONS OF THE LIVER
8g/dL and above
FFP and platelets for patients with PYOGENIC LIVER ABSCESS
severe coagulopathy - most common liver abscesses seen in the United
Short term prophylactic antibiotics: States.
ceftriaxone 1g/day (proven to decrease - Risk factors: IV drug abuse, teeth cleaning, diverticulitis,
the rate of bacterial infections and Crohn's disease, subacute bacterial endocarditis, (+)
increase survival) infected indwelling catheters & immunocompromised
Vassopressin at 0.2 to 0.8 units/min IV states
for vasoconstriction (most potent) - may be single or multiple
Octreotide/somatostatin for splanchnic - more frequently found in the right lobe of the liver
vasoconstriction - causative organisms:
Endoscopic variceal ligation (EVL) monomicrobial: 40% ; polymicrobial: 40%;
culture negative: 20%
Balloon tamponade using sengstaken-
most common: gram-negative organisms
blakemore tube
(Escherichia coli – 2/3; Streptococcus faecalis,
Shunt therapy (surgical shunts or
Klebsiella, and Proteus vulgaris are also common)
TIPS)
Anaerobic organisms (ex. Bacteroides fragilis) are
Even with aggressive pharmacologic and
also seen frequently
endoscopic therapy, 10-20% of patients
If (+) endocarditis / indwelling catheter: think
with variceal bleeding will continue to
Staphylococcus and Streptococcus
rebleed
- Clinical manifestations: RUQ pain, fever & jaundice (1/3
Shunt therapy (surgical shunt or TIPS), of patients)
on the other hand, has been shown to
- Diagnosis:
control refractory variceal bleeding in
Leucocytosis, ↑ESR & AP (most common
>90% of treated individuals
laboratory findings)
Surgical shunt: CTP class A Blood cultures reveal the causative organism in
TIPS: CTP class B & C approximately 50% of cases.
Balloon tamponade using sengstaken- Liver UTZ: round or oval hypoechoic lesions
blakemore tube can control refractory with well-defined borders and a variable
bleeding in >80% of patients number of internal echoes.
Complication: aspiration, esophageal CT scan: highly sensitive in the localization;
perforation appear as hypodense mass with air-fluid levels
(indicating a gas-producing organisms) &
Table.64 Comparison of Surgical shunts vs TIPS peripheral enhancement
Surgical shunts (can be selective TIPS (Transjugular Intrahepatic
- Treatment: cornerstones of treatment include
or non selective shunts) Portosystemic Shunt)
-aim: reduce portal venous -considered as a nonselective
correction of the underlying cause, percutaneous needle
pressure, maintain total hepatic shunt aspiration, and IV antibiotic therapy
and portal blood flow and avoid a -involves implantation of a metallic Initial antibiotic therapy needs to cover gram-
high incidence of complicating stent between an intrahepatic negative as well as anaerobic organisms; must
hepatic encephalopathy branch of the portal vein and a be continued for at least 8 weeks.
hepatic vein radical If aspiration and IV antibiotics fail, undergo
-non-selective shunt (ex. -TIPS can control variceal surgical therapy (either laparoscopic or open
portacaval shunt or eck fistula: bleeding in >90% of cases
drainage)
joins the portal vein to the IVC in refractory to medical treatment
an end-to-side fashion & disrupts -disadvantages: bleeding either
Anatomic surgical resection is reserved for patients
portal vein flow to the liver, or intra-abdominally or via the biliary with recalcitrant abscesses.
joins it in a side-to-side fashion tree, infections, renal failure, Always rule out necrotic hepatic malignancy
and maintains partial portal decreased hepatic function, and
venous flow to the liver; non ↑er hepatic encephalopathy
selective; rarely performed now (because it is a non selective AMEBIC ABSCESS
because it has a higher incidence shunt) -
most common type of liver abscesses worldwide.
of hepatic encephalopathy and -
Causative agent: Entamoeba histolytica
decreased liver function resulting
-
can be single or multiple
from the reduction of portal
perfusion; controls bleeding -
most commonly located in the superior-anterior
effectively aspect of the right lobe of the liver near the diaphragm
- Gross: necrotic central portion that contains a thick,
-selective shunt (ex. Warren reddish brown, pus-like material (anchovy paste or
shunt – distal splenorenal & left chocolate sauce)
gastric caval shunt) have ↓er - Clinical manifestation: RUQ pain + fever + hepatomegaly
incidence of hepatic + travel to an endemic area
encephalopathy
- Diagnosis:
most common biochemical abnormality: ↑ AP
F. BUDD-CHIARI SYNDROME
level.
- uncommon congestive hepatopathy characterized by
(+) Leukocytosis
the obstruction of hepatic venous outflow due to
↑transaminase levels and jaundice are unusual.
endoluminal venous thrombosis (primary) or
(+) fluorescent antibody test for E. histolytica
compressive lesion external to the veins (secondary)
Ultrasound and CT scanning: very sensitive but
- risk factors: coagulopathies, thrombotic disease
nonspecific for the detection of amebic abscesses
- most patients are women
Appears to be as a well-defined low-density round
- mean age of diagnosis: 30 yo
lesions that have enhancement of the wall, ragged
- clinical manifestations: abdominal pain (RUQ), ascites,
in appearance with a peripheral zone of edema; has
and hepatomegaly or long standing portal hypertension
a central cavity with septations & fluid levels
- diagnosis
- Treatment
- abdominal UTZ: initial investigation of choice
Metronidazole 750 mg tid for 7 to 10 days is the
check for absence of hepatic vein flow, spider web
treatment of choice and is successful in 95% of
hepatic veins & collateral circulation
cases.
TOPNOTCH MEDICAL BOARD PREP SURGERY SUPPLEMENT HANDOUT Page 77 of 81
For inquiries visit www.topnotchboardprep.com.ph or email us at topnotchmedicalboardprep@gmail.com
TOPNOTCH MEDICAL BOARD PREP SURGERY SUPPLEMENT HANDOUT - Jules Lopez,MD-MBA,Teddy Carpio,MD-MBA
For inquiries visit www.topnotchboardprep.com.ph or email us at topnotchmedicalboardprep@gmail.com
Defervescence usually occurs in 3 to 5 days. fibrous tissue and small blood vessels which eventually
Time of resolution of abscess: 30 to 300 days from grow
presentation - most common solid benign masses that occur in the
Aspiration of the abscess is rarely needed and liver
should be reserved for patients with large - more common in women
abscesses, abscesses that do not respond to - clinical manifestation:
medical therapy, abscesses that appear to be most common symptom is abdominal pain
superinfected, and abscesses of the left lobe of the can be asymptomatic as well
liver that may rupture into the pericardium - diagnosis:
biphasic contrast CT scan: asymmetrical nodular
peripheral enhancement that is isodense with
HYDATID DISEASE large vessels and exhibit progressive centripetal
- due to the larval or cyst stage of infection by the enhancement fill-in over time
tapeworm Echinococcus granulosus (causative agents) MRI: hypointense on T1-weighted images and
intermediate hosts: Humans, sheep, and cattle hyperintense on T2-weighted images
definitive host: dogs Caution should be exercised in ordering a liver
- commonly involve the right lobe of the liver, usually biopsy if the suspected diagnosis is hemangioma
the anterior-inferior or posterior-inferior segments because of the risk of bleeding from the biopsy site
- clinical manifestations: dull RUQ or abdominal - treatment: Surgical resection (enucleation or
distention; can be clinically silent; if ruptured, may lead formal hepatic resection) only if symptomatic;
to an allergic or anaphylactic reaction. observation if asymptomatic
- Diagnosis:
(+)ELISA for echinococcal antigens; maybe (-) if
cyst has not leaked or does not contain scolices, or HEPATIC ADENOMA
if the parasite is no longer viable - benign solid neoplasms of the liver
Eosinophilia of >7% is found is approximately 30% - most commonly seen in young women (aged 20-40)
of infected patients. - typically solitary
UTZ & CT scan of the abdomen: sensitive for - risk factors: Prior or current use of estrogens (oral
detecting hydatid cysts. contraceptives)
hydatid cysts: appear as well-defined - Gross appearance: soft and encapsulated and are tan to
hypodense lesions with a distinct wall; (+) light brown.
Ring-like calcifications of the pericysts - Histology: does not contain Kuppfer cells
(present in 20 to 30% of cases); healing occurs - (+) risk of malignant transformation to a well-
the entire cyst calcifies densely, and a differentiated HCC
lesion with this appearance is usually dead or - Clinical manifestation:
inactive. Daughter cysts: occur in a peripheral carry a significant risk of spontaneous rupture
location & are slightly hypodense compared with intraperitoneal bleeding.
with the mother cyst. The clinical presentation may be abdominal pain
- Treatment: - Diagnosis:
Unless the cysts are small or the patient is not a CT scan: with sharply defined borders; can be
suitable candidate for surgery, treatment of confused with metastatic tumors
hydatid disease is surgically based (laparoscopic venous phase contrast: hypodense or isodense
or open complete cyst removal + instillation of (in comparison with background liver
scolicidal agent) arterial phase contrast: subtle hypervascular
caution must be exercised to avoid rupture of enhancement
the cyst with release of protoscolices into the MRI: hyperintense on T1-weighted images and
peritoneal cavity. enhance early after gadolinium injection.
Peritoneal contamination can result in an acute nuclear imaging: "cold”; no uptake of
anaphylactic reaction or peritoneal radioisotope
implantation of scolices with daughter cyst - Treatment: surgical resection
formation and inevitable recurrence
Medical treatment of choice: albendazole - initial
treatment for small, asymptomatic cysts. FOCAL NODULAR HYPERPLASIA
- A benign, solid neoplasm of the liver
H. BENIGN NEOPLASMS OF THE LIVER - more common in women of childbearing age
- FNH lesions usually do not rupture spontaneously
HEPATIC CYST and have no significant risk of malignant
- most common benign lesion found in the liver is the transformation.
congenital or simple cyst - diagnosis:
- female:male ratio is approximately 4:1 biphasic CT scan: well circumscribed with a
- Clinical manifestation: asymptomatic if small; Large typical central scar
simple cysts may cause abdominal pain, epigastric Arterial phase contrast: intense homogeneous
fullness, and early satiety. Occasionally the affected enhancement
patient presents with an abdominal mass. Venous phase contrast: isodense or invisible
- Diagnosis: appear as thin-walled, homogeneous, MRI scans: hypointense on T1-weighted images &
fluid-filled structures with few to no septations. isointense to hyperintense on T2-weighted images
- Treatment: After gadolinium administration, lesions are
Observation if asymptomatic hyperintense but become isointense on
If symptomatic, perform UTZ- or CT-guided delayed images.
percutaneous cyst aspiration followed by nuclear imaging: (+) uptake by Kupffer cells.
sclerotherapy - Treatment: surgical resection only if symptomatic
excised cyst wall is sent for pathologic analysis to
rule out carcinoma, and the remaining cyst wall BILE DUCT HAMARTOMA
must be carefully inspected for evidence of - small liver lesions (2 - 4 mm)
neoplastic change. - usually visualized on the surface of the liver at
laparotomy.
HEMANGIOMA - Gross appearance: firm, smooth, and whitish yellow in
- consist of large endothelial-lined vascular spaces and appearance.
represent congenital vascular lesions that contain
TOPNOTCH MEDICAL BOARD PREP SURGERY SUPPLEMENT HANDOUT Page 78 of 81
For inquiries visit www.topnotchboardprep.com.ph or email us at topnotchmedicalboardprep@gmail.com
TOPNOTCH MEDICAL BOARD PREP SURGERY SUPPLEMENT HANDOUT - Jules Lopez,MD-MBA,Teddy Carpio,MD-MBA
For inquiries visit www.topnotchboardprep.com.ph or email us at topnotchmedicalboardprep@gmail.com
- can be difficult to differentiate from small metastatic - Treatment: surgery is the only curative option for
lesions gallbladder cancer
- excisional biopsy often is required to establish the reoperation for an incidental finding of gallbladder
diagnosis cancer after cholecystectomy (central liver
resection, hilar lymphadenectomy, and evaluation
I. MALIGNANT TUMORS of cystic duct stump)
reoperation should be considered for all patients
HEPATOCELLULAR CARCINOMA (HCC) who have T2 or T3 tumors or for whom the
- 5th most common malignancy worldwide accuracy of staging is in question
- Risk factors: viral hepatitis (B or C), alcoholic cirrhosis, radical resection in patients with advanced disease
hemochromatosis, and nonalcoholic steatohepatitis usually with dismal results if already with (+)
- HCCs are typically hypervascular with blood supplied hilar LN
predominantly from the hepatic artery
- Most common site of metastasis is lungs METASTATIC COLORECTAL CANCER
- Clinical manifestations: jaundice, pruritus, - Over 50% of patients diagnosed with colorectal cancer
hepatosplenomegaly, bleeding diathesis, cachexia, will develop hepatic metastases during their lifetime.
encephalopathy, asterixis, ascites and varices - Resection is the preferred treatment for liver
- Diagnosis: metastases from colorectal CA, provided that patient
CT scan: appears hypervascular during the arterial has adequate liver reserve, no extrahepatic metastases,
phase of CT studies & relatively hypodense during total hepatic involvement and advanced crirhosis, vena
the delayed phases due to early washout of the cava or portal vein invasion
contrast medium by the arterial blood. - volume of future liver remnant and the health of the
MRI: HCC is variable on T1-weighted images and background liver, and not actual tumor number, as the
usually hyperintense on T2-weighted images; HCC primary determinants in selection for an operative
enhances in the arterial phase after gadolinium approach.
injection because of its hypervascularity and
becomes hypointense in the delayed phases due to J. GALLSTONE DISEASE
contrast washout - Prevalence and incidence
(+) thrombus in portal vein is highly suggestive most common problems affecting the digestive
of HCC tract
↑AST,ALT,AFP Women are 3x more likely to develop gallstones
- treatment options for liver cancer than men
hepatic resection: reserved for patients without risk factors: Obesity, pregnancy, dietary factors,
cirrhosis & Child's class A cirrhosis with preserved Crohn's disease, terminal ileal resection, gastric
liver function and no portal hypertension surgery, hereditary spherocytosis, sickle cell
liver transplantation: if with poor liver function disease, and thalassemia
and the HCC meets the Milan criteria (one nodule - Natural history
<5 cm, or two or three nodules all <3 cm, no gross Most patients will remain asymptomatic
vascular invasion or extrahepatic spread) prophylactic cholecystectomy in asymptomatic
Chemoembolization can also be of benefit persons with gallstones is rarely indicated
5 year survival after complete resection: 30% cholecystectomy is advisable for the ff
asymptomatic patients:
1. elderly patients with diabetes
CHOLANGIOCARCINOMA 2. individuals isolated from medical care for
- 2nd most common primary malignancy within the liver extended periods of time
- It is the adenocarcinoma of the bile ducts that forms in 3. in populations with increased risk of
the biliary epithelial cells gallbladder cancer (porcelain gallbladder
- Most commonly occurs at the bifurcation of the – premalignant lesion)
common hepatic duct 4. symptomatic Cholesterolosis:
- Subclassification: accumulation of cholesterol in macrophages
peripheral (intrahepatic) bile duct cancer in the gallbladder mucosa, either locally or
tumor mass is within the lobe or peripheral of as polyps; produces the classic macroscopic
the liver appearance of a "strawberry gallbladder."
less common that extrahepatic bile duct 5. symptomatic Adenomyomatosis or
Cancer cholecystitis glandularis proliferans:
central (extrahepatic) bile duct cancer characterized on microscopy by
if it is proximally located, referred to as a hilar hypertrophic smooth muscle bundles and
cholangiocarcinoma (Klatskin's tumor). by the ingrowths of mucosal glands into the
presents with obstructive and painless muscle layer (epithelial sinus formation)
jaundice rather than an actual liver mass 6. symptomatic granulomatous polyps
- treatment: - Gallstone formation
surgical resection is the treatment of choice Cholesterol stones (80% of gallstones)
hilar cholangiocarcinoma + primary multiple, variable size, may be hard and
sclerosing cholangitis: surgical resection has faceted or irregular, mulberry-shaped, and
no role & transplantation provided dismal soft; colors range from whitish yellow and
results green to black
neoadjuvant chemoradiation has a role Most cholesterol stones are radiolucent
formation is due to supersaturation of bile
with cholesterol
GALLBLADDER CANCER Pigment stones (15-20% of gallstones)
- rare aggressive tumor with a very poor prognosis. dark because of the presence of calcium
- Cholithiasis is the most important risk factor for bilirubinate
gallbladder carcinoma Black pigment stones: small, brittle, black,
- 80-90% of gallbladder tumors are adenocarcinomas and sometimes speculated; In Asian countries
- signs and symptoms of GB carcinoma are such as Japan, black stones account for a much
indistinguishable from cholecystitis and cholelithiasis higher percentage of gallstones than in the
- sensitivity of UTZ in detecting GB carcinoma ranges Western hemisphere; typically occur in
from 70-100%. patients with cirrhosis and hemolysis

TOPNOTCH MEDICAL BOARD PREP SURGERY SUPPLEMENT HANDOUT Page 79 of 81


For inquiries visit www.topnotchboardprep.com.ph or email us at topnotchmedicalboardprep@gmail.com
TOPNOTCH MEDICAL BOARD PREP SURGERY SUPPLEMENT HANDOUT - Jules Lopez,MD-MBA,Teddy Carpio,MD-MBA
For inquiries visit www.topnotchboardprep.com.ph or email us at topnotchmedicalboardprep@gmail.com
Brown pigment stones: <1 cm in diameter, mild elevation of alkaline phosphatase,
brownish-yellow, soft, and often mushy; they transaminases, and amylase.
are formed usually due to secondary to - diagnosis:
bacterial infection (ex. E. coli)caused by UTZ: most useful radiologic test for diagnosing
bile stasis.; associated with stasis secondary acute cholecystitis
to parasite infection Is 95% sensitive and specific
- Clinical presentation Appears as thickening of the gallbladder wall
Abdominal pain: epigastrium or RUQ, constant, and (+) pericholecystic fluid
increasing in severity, episodic, usually after a fatty (+) sonographic murphy’s sign
meal, nausea, vomiting Biliary radionuclide scanning (HIDA scan):
Hydrops of gallbladder: manifests as a palpable most accurate in the diagnosis of acute
nontender gallbladder cholecystitis
Usually due to impacted stone without - Treatment
cholecystis (pathophysio: bile gets absorbed, IV fluids
but the gallbladder epithelium continues to Antibiotics: should cover Gram (-) aerobes +
secrete mucus, and the gallbladder becomes anaerobes - 3rd generation cephalosporin or 2nd
distended with mucinous material) generation cephalosporin + metronidazole
Is usually an indication for cholecystectomy Analgesia
- Diagnosis Cholecystectomy: definitive treatment
Abdominal UTZ: standard diagnostic test for Laparoscopic cholecystectomy: procedure of choice
gallstones
Presence of hyperechoic intraluminal focus L. CHOLEDOCHOLITHIASES
Shadowing posterior to the focus - Common bile duct stones
Movement of the focus with positional changes - Common over the age of 60
of the patient - clinical manifestations: may be silent or incidental; if
- Management: Patients with symptomatic gallstones symptomatic, may cause pain, nausea and vomiting with
should be advised to have elective laparoscopic mild epigastric or RUQ tenderness + mild icterus
cholecystectomy - diagnosis:
↑ of serum bilirubin, alkaline phosphatase, and
K. ACUTE CHOLECYSTITIS transaminases
- Pathogenesis: UTZ: dilated common bile duct (>8 mm in
Acute cholecystitis is secondary to gallstones in 90 diameter)
to 95% of cases Endoscopic cholangiography: gold standard for
In <1% of acute cholecystitis, the cause is a tumor diagnosing CBD stones; can be therapeutic as well
obstructing the cystic duct (leads to gallbladder IOC can be done to evaluate CBD stones
distention, inflammation, and edema of the - Treatment: sphincterotomy and ductal clearance of the
gallbladder wall) stones is appropriate, followed by a laparoscopic
Gross appearance: gallbladder wall is grossly cholecystectomy
thickened & reddish with subserosal hemorrhages;
(+) pericholecystic fluid often; mucosal hyperemia M. CHOLANGITIS
& patchy necrosis - Complication of choledochal stones
When the gallbladder remains obstructed and - Gallstones are the most common cause of obstruction in
secondary bacterial infection supervenes an cholangitis
- Normal: bile is sterile
acute gangrenous cholecystitis develops
- Causative organisms: E. coli, Klebsiella pneumoniae,
abscess or empyema forms within the gallbladder;
Streptococcus faecalis, Enterobacter, and
can also lead to perforation of ischemic areas
Bacteroidesfragilis
emphysematous gallbladder : (+) gas may be seen
in the gallbladder lumen and in the wall of the - Clinical manifestations :
gallbladder on abdominal radiographs and CT most common presentation is fever, epigastric or
scans due to gas-forming organisms as part of the right upper quadrant pain, and jaundice
secondary bacterial infection (Charcot's triad)
- clinical manifestations: charcot’s triad + septic shock + mental status
unremitting epigastric or RUQ pain, may persist for changes reynaud’s pentad
several days, may radiate to the right upper part of - diagnosis:
the back or the interscapular area; febrile, Leukocytosis, hyperbilirubinemia, and elevation of
anorexia, nausea, and vomiting, reluctant to move, alkaline phosphatase and transaminases are seen
(+) focal tenderness and guarding on the RUQ, (+) UTZ: (+)gallbladder stones, dilated ducts
Murphy's sign (an inspiratory arrest with deep ERC: Definitive diagnosis
palpation in the right subcostal area) is - Treatment
characteristic IV antibiotics: initial management; cover for
mirizzi’s syndrome: Severe jaundice due to gram (-)
common bile duct stones or obstruction of the bile Fluid resuscitation: initial management
ducts by severe pericholecystic inflammation Emergency biliary decompression: if failed to
secondary to impaction of a stone in the improve with IV antibiotics and resuscitation
infundibulum of the gallbladder that mechanically measures
obstructs the bile duct
in elderly patients and in those with diabetes N. BILIARY PANCREATITIS
mellitus, acute cholecystitis may have a subtle - Obstruction of the pancreatic duct by an impacted stone
presentation resulting in a delay in diagnosis. or temporary obstruction by a stone passing through
- Laboratory diagnosis: the ampulla leads to this condition
A mild to moderate leukocytosis (12,000 to 15,000 - Diagnosis: UTZ of biliary tree
cells/mm3) - Treatment: ERC with sphincterotomy and stone
if high WBC (above 20,000): suggests a extraction + cholecystectomy (upon resolution of
complicated form of cholecystitis such as pancreatitis during same admission)
gangrenous cholecystitis, perforation, or
associated cholangitis. O. ACALCULOUS CHOLECYSTITIS
mild elevation of serum bilirubin, <4 mg/mL - develops in critically ill patients in ICU (patients on
parenteral nutrition with extensive burns, sepsis, major
TOPNOTCH MEDICAL BOARD PREP SURGERY SUPPLEMENT HANDOUT Page 80 of 81
For inquiries visit www.topnotchboardprep.com.ph or email us at topnotchmedicalboardprep@gmail.com
TOPNOTCH MEDICAL BOARD PREP SURGERY SUPPLEMENT HANDOUT - Jules Lopez,MD-MBA,Teddy Carpio,MD-MBA
For inquiries visit www.topnotchboardprep.com.ph or email us at topnotchmedicalboardprep@gmail.com
operations, multiple trauma, or prolonged illness with emen
multiple organ system failure) REVIEW QUESTIONS
- histopathology: reveals edema of the serosa and
muscular layers, with patchy thrombosis of arterioles 1. A patient presents with biliary colic. On ultrasound
and venules there are multiple small gallstones in the
- clinical manifestations: gallbladder and the common bile duct measures
alert patient: right upper quadrant pain and 9mm in diameter. No stone is visualized in the
tenderness, fever, and leukocytosis common bile duct. Which of the following is the
sedated or unconscious patient: fever and elevated most reasonable next step?
WBC count, as well as elevation of alkaline
phosphatase and bilirubin a. Repeat UTZ in 24-48 hours
- diagnosis: b. MRCP with contrast
UTZ: diagnostic test of choice; appears as distended c. Percutaneous cholangiography
gallbladder with thickened wall, biliary sludge, d. Laparoscopic cholecystectomy and intraoperative
pericholecystic fluid, and (+) abscess formation cholangiography
- Treatment of choice: Percutaneous ultrasound- or CT-
guided cholecystostomy Answer: D
For patients with symptomatic gallstones and
P. BILIARY or CHOLEDOCHAL CYSTS suspected CBD stones, either preoperative endoscopic
- congenital cystic dilatations of the extrahepatic and/or cholangiography or an intraoperative cholangiogram
intrahepatic biliary tree will document the bile duct stones. If an endoscopic
- rare cholangiogram reveals stones, sphincterotomy and
- more common in women ductal clearance of the stones is appropriate, followed
- more frequently diagnosed during childhood by a laparoscopic cholecystectomy. An intraoperative
- types: cholangiogram at the time of cholecystectomy will also
type I: cystic dilatation of the extrahepatic bile document the presence or absence of bile duct stones.
duct; most common type Laparoscopic common bile duct exploration via the
type II: diverticulum of the CBD cystic duct or with formal choledochotomy allows the
type III: a “choledochocele” extending from the distal stones to be retrieved in the same setting. If the
duct into the duodenum expertise and/or the instrumentation for laparoscopic
type IV: combined intrahepatic and extrahepatic cysts common bile duct exploration are not available, a drain
type V: cystic disease confined to intrahepatic ducts shuld be left adjacent to the cystic duct and the patient
scheduled for endoscopic sphincterotomy the following
- clinical manifestations: jaundice or cholangitis (for day. An open common bile duct exploration is an option
adults);less than ½ of patients present with the classic of the endoscopic method has already been tried or is,
clinical triad of abdominal pain, jaundice, and a mass for some reason, not feasible.
- diagnosis: Ultrasonography or CT scanning will confirm
the diagnosis, but endoscopic, transhepatic, or MRC is 2. Which hepatic cells provides the primary defense
required to assess the biliary anatomy and to plan the against lipopolysaccharide (LPS)?
appropriate surgical treatment
- treatment: complete cyst excision with roux-en-Y a. Hepatocytes
hepaticojejunostomy b. Kuppfer cells
c. Bile duct epithelial cells
Q. SCLEROSING CHOLANGITIS d. Intrahepatic endothelial cells
- is a progressive disease that eventually results in
secondary biliary cirrhosis characterized by Answer: B
inflammatory strictures involving the intrahepatic and The complications of Gram negative sepsis is
extrahepatic biliary tree initiated by the endotoxin LPS. The liver is the
- associated with ulcerative colitis, Riedel's thyroiditis main organ in the clearance of LPS in the
and retroperitoneal fibrosis bloodstream and plays a critical role in the
- increased risk for developing cholangiocarcinoma. identification and processing of LPS. Kuppfer cells
- mean age of presentation is 30 to 45 years are the resident macrophages in the liver and have
- men are affected twice as commonly as women been shown to participate in LPS clearance.
- clinical manifestations: jaundice, fatigue, weight loss,
pruritus, and abdominal pain; usually with cyclic
remissions and excacerbations
- diagnosis:
elevated ALP & bilirubin
ERCP: confirmatory test
multiple dilatations and strictures
(beading) of both the intra- and
extrahepatic biliary tree

TOPNOTCH MEDICAL BOARD PREP SURGERY SUPPLEMENT HANDOUT Page 81 of 81


For inquiries visit www.topnotchboardprep.com.ph or email us at topnotchmedicalboardprep@gmail.com

You might also like